SlideShare a Scribd company logo
Hỗ trợ trực tuyến
Fb www.facebook.com/DayKemQuyNhon
Mobi/Zalo 0905779594
Tài liệu chuẩn tham khảo
Phát triển kênh bởi
Ths Nguyễn Thanh Tú
Đơn vị tài trợ / phát hành / chia sẻ học thuật :
Nguyen Thanh Tu Group
C H I N H P H Ụ C K Ì T H I V À O
L Ớ P 1 0 M Ô N T O Á N
Ths Nguyễn Thanh Tú
eBook Collection
CHINH PHỤC KÌ THI VÀO LỚP 10 THPT
MÔN TOÁN NĂM 2024 CÁC CHUYÊN ĐỀ
HAY VÀ KHÓ, 20 ĐỀ LUYỆN TẬP CÓ
HƯỚNG DẪN CHI TIẾT
WORD VERSION | 2023 EDITION
ORDER NOW / CHUYỂN GIAO QUA EMAIL
TAILIEUCHUANTHAMKHAO@GMAIL.COM
vectorstock.com/28062405
Liên hệ tài liệu word toán SĐT (zalo): 521
Website:
Môc lôc
Trang
PHẦN 1 Các chuyên đề hay và khó
Chương 1 Phương trình
Chương 2 Hệ phương trình
Chương 3 Bất đẳng thức
PHẦN 2 20 đề luyện tập có hướng dẫn chi tiết
PHẦN 3 10 đề tự luyện
Liên hệ tài liệu word toán SĐT (zalo): 1
Website:
Chuyên đề 1. PHƯƠNG TRÌNH
I. TÓM TẮT LÝ THUYẾT
Đối với phương trình vô tỷ (tức là phương trình có chứa ẩn trong dấu căn), điều cần lưu ý nhất là tính
không thuận nghịch của các phép toán. Chẳng hạn nếu trong một phương trình nào đó, bạn thay .
A B
(với A và B là các biểu thức nào đó của x ) bởi .
A B thì tập xác định của phương trình rất có thể bị
mở rộng, bởi vì .
A B chỉ xác dịnh khi 0
A ≥ và 0
B ≥ trong khi .
A B xác định ngay cả khi 0
A <
và 0
B < . Vậy bạn chỉ thu được một phương trình hệ quả. Ngược lại, nếu thay thế .
A B bởi .
A B
thì tập xác định có thể bị thu hẹp lại, do đó bạn rất dễ bị bỏ sót nghiệm. Điều đó cảnh báo rằng khi thực
hiện một phép tính về căn thức, để biến đổi một phương trình thì nói chung bạn không được phương
trình tương đương. Để tránh các sai sót kiểu như thế, người ta dùng một trong các cách sau:
Cách 1: Nếu chắc chắn phép biến đổi chỉ cho phương trình hệ quả thì ở bước cuối cùng, ta dùng phép
thử trực tiếp vào phương trình để loại bỏ nghiệm ngoại lai.
Ví dụ:
Giải phương trình 2 1 3 3
x x
− + + =
Giải:
Phương trình đã cho, suy ra:
( ) ( )( )
2
2 1 3 9 2 2 1 3 7 3
x x x x x
− + + = ⇒ − + = −
( ) ( )
2
2 2 1
4 2 5 3 7 3 62 61 0
61
x
x x x x x
x
=

⇒ + − = − ⇒ − + = ⇒  =

Thử trực tiếp vào phương trình, ta thấy 1
x = thỏa mãn, còn 61
x = không thỏa mãn.
Vậy phương trình có nghiệm là: 1
x = .
Cách 2: Ghi nhớ tập xác định của phương trình và các điều kiện cần thiết khác trước khi biến đổi
phương trình. Nếu phép biến đổi dẫn đến phương trình hệ quả thì nghiệm ngoại lai chính là các giá trị
của ẩn không nằm trong tập xác định hoặc không thỏa mãn các điều kiện đã nêu. Đôi khi, chính tập xác
định và các điều kiện ấy sẽ đem lại những gợi ý hữu ích cho bạn trong quá trình giải phương trình.
Ví dụ:
Giải phương trình ( ) ( )
2
3 2 1
x x x x x
+ −= +
Liên hệ tài liệu word toán SĐT (zalo): 2
Website:
Giải:
Điều kiện
( )
( )
0
3 0 1
2
2 1 0
3
x
x x
x
x x
x
=


− ≥

 
⇔ ≤ −


+ ≥


 ≥

+ Xét 0
x = , thỏa mãn phương trình
+ Xét
1
2
x ≤ − phương trình đã cho tương đương với
. . 3 . 2 1 3 2 1
x x x x x x x x x
− − + − − + = − − − ⇔ − + − + = − −
( ) ( )
2
3 2 1 2 3 4
x x x x x
⇔ − + − + =
− − ⇔ − − + =
− (vô nghiệm vì giá trị của căn thức không thể
bằng một số âm)
+ Xét 3
x ≥ , phương trình đã cho tương đương với
. . 3 . 2 1 3 2 1
x x x x x x x x x
+ − = + ⇔ + − = +
( ) ( )
2
2 4
3 2 1 2 3 4 3 4 0
1
x
x x x x x x x
x
=

⇔ + − = + ⇔ − = ⇔ − − = ⇔  = −

Nhận thấy 1
x = − không thỏa mãn 3
x ≥ nên bị loại.
Vậy phương trình đã cho có nghiệm: 0; 4
x x
= =
Cách 3
Chú ý đến các điều kiện xác định của phương trình, các điều kiện để thực hiện các phép biến đổi đồng
nhất hay biến đổi tương đương phương trình và đặt các điều kiện đó cùng với phương trình trong một hệ
hỗn hợp (cả phương trình và bất phương trình). Hệ này sẽ tương đương với phương trình đã cho.
Nhưng dù theo cách nào thì bạn cũng phải chú ý đến các điều kiện nảy sinh trong quá trình biến đổi
phương trình, đặc biệt là sự thay đổi tập xác định của phương trình. Điều đó sẽ giúp bạn có những quyết
định đúng đắn khi giải phương trình.
Dưới đây là một số đồng nhất thức có điều kiện thường gặp:
Đồng nhất thức Điều kiện
( )
2
A A
= 0
A ≥
. .
A B A B
= 0
A ≥ và 0
B ≥
A A
B
B
= 0
A ≥ và 0
B >
2
.
A B A B
= 0
A ≥ và 0
B ≥
2
.
A B A B
= − 0
A ≤ và 0
B ≥
Liên hệ tài liệu word toán SĐT (zalo): 3
Website:
* DẠNG 1: A B
=
Phương pháp: Sử dụng
công thức của định nghĩa căn bậc hai số học 2
0
B
A B
A B
≥

= ⇔ 
=

Chú ý: Sau khi tìm nghiệm của bài toán xong, chúng ta nên thử lại nghiệm để tránh sai sót trong tính
toán.
Ví dụ 1:
Giải phương trình: 2
4 3 2 5
x x x
− + − = −
Giải:
Phương trình tương đương với
( )
2
2
2 5 0
4 3 2 5
x
x x x
− ≥



− + − = −


2
5
5 2
14
2
2
5
5 24 28 0 14
5
x
x
x
x
x x
x

≥


≥ 

⇔ ⇔ ⇔ =
=

 

 
− + =
 
 =


Vậy phương trình có nghiệm là:
14
5
x =
Ví dụ 2:
Giải phương trình: 2
2 3 5 2 2
x x x
+ − = −
Giải:
Phương trình tương đương với
( )
2
2
2 2 0
2 3 5 2 2
x
x x x
− ≥



+ − = −


( )( )
1
1
1 1
9
1 2 9 0 9
2
2
x
x
x x
x x x
x
≥

=


≥

  =
 
⇔ ⇔ ⇔
  
− − = =

  = 


.
A
B A B
B
= 0
A ≤ và 0
B >
.
A
B A B
B
= − 0
A ≤ và 0
B <
Liên hệ tài liệu word toán SĐT (zalo): 4
Website:
Vậy phương trình có nghiệm là:
9
1;
2
x x
= =
Liên hệ tài liệu word toán SĐT (zalo): 5
Website:
BÀI TẬP TƯƠNG TỰ
1) Giải phương trình: 2
3 4 3 1
x x x
+ + − =
Đáp số:
3 105
16
x
− +
=
2) Giải phương trình: 2
2 6 2
x x x
+ − = −
Đáp số:
5
3
x =
3) Giải phương trình: 2
2 3
x x x
+ + + =
Đáp số: 1
x =
4) Giải phương trình: 2
2 3 1 0
x x x
+ + + + =
Đáp số: 3
x = −
* DẠNG 2: A B
=
Phương pháp: Phương trình tương đương với
( )
0 0
A B
A B
≥ ≥



=


Ví dụ :
Giải phương trình: 2
3
x x x
− = −
Giải:
Phương trình tương đương với 2
3 0
3
x
x x x
− ≥


− = −

2
3
3
3
3 3
x
x
x
x x
≤
≤ 
 
⇔ ⇔ ⇔ =
±
 
= = ±

 
BÀI TẬP TƯƠNG TỰ:
1) Giải phương trình: 2 5 1
x x
+ = −
Đáp số:
4
3
x =
2) Giải phương trình: 2
2 3 4 3
x x
− = −
Đáp số: 2
x =
3) Giải phương trình: 2
6 3
x x x
− − = −
Đáp số: 3
x =
Liên hệ tài liệu word toán SĐT (zalo): 6
Website:
* DẠNG 3: A B C
+ =
Phương pháp: Bình phương 2 vế của phương trình ta được
( )
2
.
2
C A B
A B C A B
− −
+ =
⇔ = (quay về dạng 1)
Chú ý: Chỉ bình phương khi 2 vế của phương trình đều không âm.
Ví dụ 1:
Giải phương trình: 3 1 2 3
x x
+ + − =
Giải:
Điều kiện:
1
2
3
x
− ≤ ≤
Phương trình tương đương với: ( )( )
2 3 2 3 1 2 9
x x x
+ + + − =
2
2 2
3 0
3 5 2 3
3 5 2 6 9
x
x x x
x x x x
− >

⇔ − + + = − ⇔ 
− + + = − +

2
1
4 11 7 0 7
4
x
x x
x
=


⇔ − + = ⇔
 =

Đối chiếu với điều kiện ta thu được nghiệm:
7
1;
4
x x
= =
Nhận xét:
- Phương trình dạng: ( ) ( ) ( ) ( )
( )
2
2
0
f x g x m f x g x m
+ = > ⇔ + =
( ) ( ) ( ) ( ) ( ) ( ) ( ) ( )
2 2
2 2
f x g x f x g x m f x g x m f x g x
⇔ + + = ⇔ = − −
( ) ( )
( ) ( ) ( ) ( )
2
1
2
2
2
4
m f x g x x x
x x
f x g x m f x g x
 ≥ + =


⇔ ⇒
  =
 
= − − 
  

- Phương trình trên có cách giải khác như sau:
( ) ( ) ( ) ( )
f x g x m f x m g x
+ =
⇔ =
−
( )
( ) ( ) ( )
( )
( ) ( ) ( )
2 2
2 2
m f x m f x
f x m m g x g x m g x g x m f x
≥ ≥
 
 
⇔ ⇔
 
= − + = + −
 
 
Liên hệ tài liệu word toán SĐT (zalo): 7
Website:
( ) ( ) ( )
( ) ( ) ( )
( )
2
1
2
2 2
2
;
4
m f x g x m f x x x
x x
m g x g x m f x
 ≥ + ≥ =


⇔ ⇒
  =
= + − 


Ý tưởng: Đây là một bài phương trình cơ bản, dạng toán một vế chưa hai căn thức vế còn lại là một
hằng số thì phương pháp nâng lũy thừa hai vế là một phương pháp tối ưu nhất.
Ví dụ 2:
Giải phương trình: 5 1 2 3 14 7
x x x
+ + +
= +
Giải:
Điều kiện:
5 1 0
1
2 3 0
5
14 7 0
x
x x
x
+ ≥


+ ≥ ⇔ ≥ −

 + ≥

Phương trình tương đương với: ( )
2
5 1 2 3 14 7
x x x
+ + − = +
7 3
7 4 2 5 1. 2 3 14 7 5 1. 2 3
2
x
x x x x x x
+
⇔ + + + + = + ⇔ + + =
( )( )
2 2
2
7 3 49 42 9
5 1 2 3 10 17 3
2 4
x x x
x x x x
+ + +
 
⇔ + +
= ⇔ + +
=
 
 
2
3
9 26 3 0 1
9
x
x x
x
=


⇔ − − = ⇔
 = −

Vậy phương trình có nghiệm:
1
; 3
9
x x
=
− =
Nhận xét: Ở đây, khi Giải phương trình
7 3
5 1. 2 3
2
x
x x
+
+ + = chúng ta không cần đặt điều kiện
2
0
B
A B
≥


=

(tức
7 3
0
2
x +
≥ ) vì khi
1
5
x ≥ − thì
7 3
0
2
x +
> .
Nhưng nếu chúng ta không nhận xét được vế phải thì chúng ta vẫn phải đặt điều kiện bình thường như ở
dạng 1
Ví dụ 3:
Giải phương trình: 3 3 5 2 4
x x x
− − − = −
Giải:
Liên hệ tài liệu word toán SĐT (zalo): 8
Website:
Điều kiện
3 3 0
5 0 2 5
2 4 0
x
x x
x
− ≥


− ≥ ⇔ ≤ ≤

 − ≥

Phương trình tương đương với: 3 3 2 4 5
x x x
− = − + −
( )
2
3 3 2 4 5 3 3 1 2 2 4. 5
x x x x x x x
⇔ − = − + − ⇔ − = + + − −
( ) ( )( )
2
2 2 4. 5 2 2 4 5
x x x x x x
⇔ − = − − ⇔ − = − −
2 2
3 18 24 0
4
x
x x
x
=

⇔ − + = ⇔  =

Vậy phương trình có nghiệm: 2; 4
x x
= =
Nhận xét: Ở đây, khi giải phương trình: 2 2 4. 5
x x x
− = − − chúng ta không cần đặt điều kiện
2
0
B
A B
≥


=

(tức 2 0
x − ≥ ) vì khi 2 5
x
≤ ≤
Nhưng nếu chúng ta không nhận xét được vế trái thì chúng ta vẫn phải đặt điều kiện bình thường như ở
dạng 1.
BÀI TẬP TƯƠNG TỰ
1) Giải phương trình: 11 3 1 4 2 5
x x x
+ − +
= −
Đáp số: 3
x =
2) Giải phương trình: 5 1 3 2 1
x x x
− − − = −
Đáp số: 2
x =
3) Giải phương trình: 2 3 1 1 2 2 1
x x x
+ − −
= −
Đáp số: 5
x =
4) Giải phương trình: 3 1 1 8
x x
+ + + =
Đáp số: 8
x =
5) Giải phương trình: 7 4 1 3
x x
+ − + =
Đáp số: 3
x =
* DẠNG 4: ( )
2 2
2 2 0
x a x b a b x a x b a b cx d a
+ − + − + − − + − = + >
Phương pháp: Đặt ( )
0
t x b t
= − ≥ , suy ra 2
x t b
= + .
Phương trình trở thành: ( )
2 2 2 2 2
2 2
t at a t at a c t b d
+ + + − + = + +
Liên hệ tài liệu word toán SĐT (zalo): 9
Website:
( ) ( )
2 2
t a t a c t b d t a t a c t b d
⇔ + + − = + + ⇔ + + − = + +
Sau đó, sử dụng định nghĩa trị tuyệt đối:
( )
( )
0
0
A A
A
A A
≥

= 
− <


hoặc sử dụng phương pháp chia khoảng để
giải.
Ví dụ 1:
Giải phương trình: 1 2 2 1 2 2 1
x x x x
− + − − − − − =
Giải:
Điều kiện 2
2 0 2
2
6 9 0
1 2 2 0
x x
x
x x
x x
− ≥ ≥
 

⇔ ⇔ ≥
 
− + ≥
− − − ≥
 

Đặt: ( )
2 0
t x t
= − ≥ , suy ra 2
1 1
x t
− = + .
Phương trình trở thành: 2 2
1 2 1 2 1
t t t t
+ + − + − =
( ) ( )
2 2 1 1
1 1 1 1 1 1 1
1 2
t t
t t t t t t t
t t
− =

+ − − = ⇔ + − − = ⇔ − = ⇔ ⇔ =
 − =
−

1 9
2
2 4
x x
⇔ − = ⇔ =
Vậy phương trình có nghiệm:
9
4
x =
Ví dụ 2:
Giải phương trình: 2 1 2 1 2
x x x x
+ − − − − =
Giải:
Điều kiện 2
1 0 1
1
4 4 0
2 1 0
x x
x
x x
x x
− ≥ ≥
 

⇔ ⇔ ≥
 
− + ≥
− − ≥
 

Đặt ( )
1 0
t x t
= − ≥ , suy ra 2
1
x t
= +
Phương trình trở thành: 2 2
1 2 1 2 2
t t t t
+ + − + − =
( ) ( )
2 2
1 1 2 1 1 2 1 1 1 0
t t t t t t t
+ − − = ⇔ + − − = ⇔ − = − ⇔ − ≥
1 1 2
x x
⇔ − ≥ ⇔ ≥
Vậy phương trình có nghiệm với mọi 2
x ≥
Liên hệ tài liệu word toán SĐT (zalo): 10
Website:
BÀI TẬP TƯƠNG TỰ
1) Giải phương trình: 14 49 14 49 14
x x x x
+ − + − − =
Đáp số:
7
; 7
2
x
 
∈  
 
.
2) Giải phương trình:
3
2 1 2 1
2
x
x x x x
+
+ − + − − =
Đáp số: 1; 5
x x
= =
3) Giải phương trình: 4 4 4 4 4
x x x x
+ − + − − =
Đáp số: [ ]
4; 8
x ∈
* DẠNG 5: ( ) ( )
. .
a f x b c f x d e
+ + + =
Phương pháp:
Đặt ( )
t f x
= , Phương trình trở thành: . .
a t b c t d e
+ + + =
Sau đó bình phương hai vế đưa về dạng A B
=
Ví dụ:
Giải phương trình: 2 2
3 6 2 6 5 9
x x x x
+ + + + + =
Giải:
Điều kiện x ∈  .
Phương trình tương đương với: ( ) ( )
2 2
3 6 2 3 5 9
x x x x
+ + + + + = (1)
Đặt ( )
2 5
3 *
2
t x x t
 
= + ≥ −
 
 
Phương trình (1) trở thành: 6 2 5 9
t t
+ + + =
( )
2
6 2 5 81 3 11 2 6. 2 5 81
t t t t t
+ + + = ⇔ + + + + =
( )( ) ( )
2
70 3 0
2 6. 2 5 70 3
4 6 2 5 70 3
t
t t t
t t t
− ≥


⇔ + + = − ⇔ 
+ + = −


( )( ) ( )
2 2 2
70 70
3 3
8 68 120 4900 420 9
4 6 2 5 70 3
t t
t t t t
t t t
 
≤ ≤
 
⇔ ⇔
 
  + + = − +
⇔ + + = − 

Liên hệ tài liệu word toán SĐT (zalo): 11
Website:
2
70
70
3
10
3
478
488 4780 0
10
t
t
t
t
t t
t

≤
 
≤
 
⇔ ⇔ ⇔ =
  =

 ⇔
− + = 
  =


(thỏa mãn điều kiện (*))
Với 2 2
10 3 10
5
x
t x x
x
=

= ⇒ + = ⇔  = −

Vậy phương trình có nghiệm: 5; 2
x x
=
− =
.
* DẠNG 6: ( ) ( )
. .
a f x b c f x d
+
= +
Phương pháp: Đặt ( ) ( )
. 0
t c f x d t
= + ≥ , suy ra ( )
2
t d
f x
c
−
=
Phương trình trở thành:
2
2
0
t d
a b t at ct ad bc
c
 
−
+ = ⇔ − − + =
 
 
.
Giải phương trình: này và sau đó thế lại tìm ẩn x
Ví dụ:
Giải phương trình: 2 2
2 2 4 8 20
x x x x
+ = + + +
Giải:
Điều kiện x ∈  .
Đặt ( )
2
2 4 8 0
t x x t
= + + ≥ , suy ra
2
2 8
2
2
t
x x
−
+ =
Phương trình trở thành:
2
8
20
2
t
t
−
= +
2 6
2 48 0
8
t
t t
t
= −

⇔ − − = ⇔  =

+ Với 6
t = − , không thỏa mãn điều kiện.
+ Với 8
t = , thỏa mãn điều kiện nên ta có 2
2 4 8 8
x x
+ + =
2 2
2 4 8 64 2 4 56 0 1 29
x x x x x
⇔ + + = ⇔ + − =⇔ =
− ±
Vậy phương trình có nghiệm: 1 29
x =− ±
* DẠNG 7: ( )( )
a x b x c a x b x d
+ + − + + − =
Liên hệ tài liệu word toán SĐT (zalo): 12
Website:
Phương pháp: Đặt ( )
0
t a x b x t
= + + − ≥ , suy ra
2
.
2
t a b
a x b x
− −
= + − .
Phương trình trở thành:
2
.
2
t a b
t c d
− −
+ =
2
2 2 0
ct t ca cb d
⇔ + − − − =
Giải phương trình: này và sau đó thế lại tìm ẩn x
Ví dụ:
Giải phương trình: ( )( )
1 3 1 3 2
x x x x
+ + − − + − =
Giải:
Điều kiện
1 0
1 3
3 0
x
x
x
+ ≥

⇔ − ≤ ≤

− ≥

Đặt ( )
1 3 0
t x x t
= + + − ≥ , suy ra
2
4
1. 3
2
t
x x
−
+ − = .
Phương trình trở thành:
2
4
2
2
t
t
−
− =
2 0
2 0
2
t
t t
t
=

⇔ − + = ⇔  =

.
+ Với
1 0
0 1 3 0
3 0
x
t x x
x
+ =

= ⇔ + + − = ⇔ 
− =

(vô nghiệm).
+ Với
1
2 1. 3 0
3
x
t x x
x
= −

= ⇔ + − = ⇔  =

(thỏa mãn điều kiện)
Vậy phương trình có nghiệm: 1; 3
x x
=
− =
* DẠNG PHỨC TẠP
Ví dụ 1:
Giải phương trình: ( )( )
2
1 1 2 2 1 8
x x x
+ + − + − =
Giải:
Điều kiện 1 1
x
− ≤ ≤
Đặt: 1 1
t x x
= + + −
Liên hệ tài liệu word toán SĐT (zalo): 13
Website:
( )( )
2 2
1 2 1 1 1 2 2 1
t x x x x x
⇒ = + + + − + − = + − .
Khi đó phương trình đã cho trở thành 2
. 8
t t =
3 2 2
8 2 2 2 1 4 1 1 0
t t x x x
⇔ = ⇔ = ⇔ + − = ⇔ − = ⇔ =
Vậy phương trình đã cho có nghiệm duy nhất là 0
x = .
Nhận xét: Này toán sử dụng phương pháp đặt ẩn phụ Giải phương trình
* NHẮC LẠI KIẾN THỨC VÀ PHƯƠNG PHÁP
- Hằng đẳng thức quen thuộc: ( )
2 2 2
2
u v u uv v
+ = + + .
- Với ( ) [ ; ]
f x a b
∈ − , đặt ( ) ( )
t a f x b f x
= + + − , khi đó
( )
( ) ( )
( ) ( )
( ) ( )
( )
2
2
2
2
t a b
t a b a f x b f x a f x b f x
− −
=
+ + + − ⇔ + − =
Ý tưởng: Nhận thấy ở hai căn thức, ta có tổng bình phương của chúng là một hằng số, mặt khác tích của
chúng có liên quan đến biểu thức còn lại trong phương trình.
- Ta có: ( ) ( ) ( )( )
2 2
2
1 1 1 1 2; 2 1 2 1 1
x x x x x x x
+ + − = + + − = − = + −
- Do đó: ( ) ( ) ( )( )
2 2
2
2 2 1 1 1 2 1 1
x x x x x
+ − = + + − + + −
( )
2
1 1 .
x x
= + + −
- Đặt 1 1
t x x
= + + − , phương trình đã cho trở thành: 3
8
t =
2 2
2 2 2 1 4 1 1 0
t x x x
⇔ = ⇔ + − = ⇔ − = ⇔ =
Bài toán kết thúc.
BÀI TẬP TƯƠNG TỰ
1) Giải phương trình: ( )
2
6 2 4 3 2 2
x x x
+ − = + + −
Đáp số: 2
x = ± .
2) Giải phương trình: ( )( )
7 2 2 4 3 2 2 4 4 3
x x x x
+ − + = − + +
Đáp số:
5 4 3
4
x
±
= −
Ví dụ 2:
Giải phương trình: ( )
2
2 2 2 4 2 3
x x x x
+ + − + − = −
Liên hệ tài liệu word toán SĐT (zalo): 14
Website:
Giải:
Điều kiện 2
x ≥ .
Đặt: 2 2 0
t x x
= + + − >
2 2 2
2 2 2 4 2 2 4
t x x x x x
⇒ =
+ + − + − =+ −
Phương trình đã cho tương đương: 2
2 2 2 2 4 6
x x x x
+ + − + + − =
2
6 2
t t t
⇔ + − ⇔ = hoặc 3
t = − (loại).
Với 2
t = ta có 2 2 2
x x
+ + − =
Do điều kiện 2
x ≥ , ta có: 2 2 4 0 2
x x
+ + − ≥ + =
Suy ra phương trình có nghiệm duy nhất 2
x = .
Nhận xét: Bài toán sử dụng phương pháp đặt ẩn phụ, đưa phương trình ban đầu về phương trình bậc hai
tìm ẩn, sau đó dùng phương pháp nâng lũy thừa tìm nghiệm của phương trình ban đầu.
* NHẮC LẠI KIẾN THỨC VÀ PHƯƠNG PHÁP
- Cách giải phương trình bậc hai tổng quát: 2
. . 0
a t b t c
+ + =
- Hằng đẳng thức: ( )( )
2 2
a b a b a b
− = − + và ( )
2 2 2
2
a b a ab b
+ = + +
- Phương trình có dạng: ( ) ( )
f x g x m
+ =
, với m là số thực dương thì có hai cách nâng lũy thừa
như sau:
Cách 1. Bình phương hai vế của phương trình, ta có:
( ) ( )
( ) ( )
( ) ( ) ( ) ( ) 2
0; 0
2 .
f x g x
f x g x m
f x g x f x g x m
≥ ≥


+ =
⇔ 
+ + =


( ) ( )
( ) ( ) ( ) ( )
1
2
2
2
0; 0
.
4 .
f x g x x x
x x
f x g x m f x g x
≥ ≥
 =


⇔ ⇒
  =
 
= − − 
  

Cách 2. Chuyển ( )
g x sang VP rồi bình phương, ta có:
( ) ( )
( )
( ) ( ) ( )
2
2
m g x
f x m g x
f x m m g x g x
 ≥

=
− ⇔ 
=
− +


( )
( ) ( ) ( )
( )
( ) ( ) ( )
2
2 2 2
.
2 4
m g x
m g x
m g x m f x g x m g x m f x g x

 ≥
≥
 
⇔ ⇔
 
 
= − + = − +
   
 
Chú ý:
- Nếu ( ) ( )
f x g x k
− =
; k là hằng số thì ta có thể sử dụng cách liên hợp như sau:
Liên hệ tài liệu word toán SĐT (zalo): 15
Website:
( ) ( )
f x g x m
+ =
(i)
( ) ( )
( ) ( ) ( )
( ) ( ) ( )
( )
f x g x f x g x m f x g x
⇔ + − = −
( ) ( ) ( ) ( )
( ) ( ) ( )
k
f x g x m f x g x f x g x
m
⇔ − = − ⇔ − = (ii)
Lấy (i) + (ii), ta được ( ) ( )
2
2 4
k k
f x m f x m
m m
 
= + ⇔ = +
 
 
.
- Nếu c a b
≥ ≥ và x c
≥ suy ra x a x b c a c b
+ + + ≥ + + + .
Dấu “=” xảy ra khi và chỉ khi x c
= .
Ý tưởng: Bài toán xuất hiện ba căn thức bậc hai ở VT là: 2, 2
x x
+ − và 2
4
x − , áp dụng hằng
đẳng thức ( )( )
2 2
a b a b a b
− = − + dễ thấy được rằng 2
4 2. 2
x x x
− = − + , hay nói cách khác căn
thức cuối cùng chính là tích của hai căn thức còn lại. Đồng thời nếu chuyển ( )
2 3 x
− từ VP sang VT thì
sẽ xuất hiện 2x , mà ( ) ( )
2 2
2 2 2 2 2
x x x x x
= + + − = + + − do đó VT của phương trình ban đầu có:
( ) ( )
2 2
2 2 2. 2. 2 2 2 6 0
x x x x x x
+ + − + − + + + + − − =
( )
2
2 2 2 2 6 0
x x x x
⇔ + + − + + + − − =
(*)
- Đặt 2 2 0
t x x
= + + − > thì phương trình (*) được viết lại thành:
( )( )
2
0
0
2
2 3 0
6 0
t
t
t
t t
t t
>
> 
 
⇔ ⇔ =
 
− − =
+ − = 
 
- Với 2
t = suy ra 2
2
2 2 2
2 2 4 4
x
x x
x x
≥


+ + − = ⇔ 
+ − =


2
2
2
4 2
x
x
x x
≥


⇔ ⇔ =

− = −


Đến đây có thể đánh giá như lời giải là: 2 2 2 4 0 2 2
x x x x
≥ ⇒ + + − ≥ + = ⇒ = .
- Vì ( )
2 2 4
x x
+ − − = nên giải phương trình 2 2 2
x x
+ + − = theo chú ý như sau:
( )
2 2 2 4 2 2
x x x x
+ + − = ⇔ = + + −
2 2 2 2 2 4 2
x x x x
⇔ + − = ⇒ + = ⇔ =
Bài toán kết thức.
BÀI TẬP TƯƠNG TỰ
Liên hệ tài liệu word toán SĐT (zalo): 16
Website:
1) Giải phương trình: 2
2 1 2 1 2 1 3 2
x x x x
+ + − + − = −
Đáp số: phương trình vô nghiệp thực.
2) Giải phương trình: 2 2
2 5 2 2 5 1 5 2
x x x x x
+ + + − = − + +
Đáp số:
17
16
x =
Ví dụ 3:
Giải phương trình: 2
3 1 3 1 1
x x x x
+ + − = + + −
Giải:
Điều kiện 1 1
x
− ≤ ≤ .
Phương trình tương đương với:
( )
2
2
2 1 1 1 1 2 1
x x x x x
+ + + − = + + − + +
( ) ( )
1 1 2 1 2 1 1 1
x x x x x x
⇔ + + − + + = + + + + −
( )( )
1 1 2 1 1 0
x x x
⇔ + + − − + − =
+ Giải 2 2
1 1 2 2 2 1 4 1 1 0
x x x x x
+ + − = ⇔ + − = ⇔ − = ⇔ =
+ Giải 1 1 0
x x
+ = ⇔ =
Đáp số 0
x =
Nhận xét: Bài toán sử dụng phương pháp nhóm nhân tử chung, sau đó nâng lũy thừa bậc hai để tìm
nghiệm của phương trình.
* NHẮC LẠI KIẾN THỨC VÀ PHƯƠNG PHÁP
- Giải phương trình: ( ) ( )
( )
( )
0
. 0
0
f x
f x g x
g x
=

= ⇔ 
=


.
- Giải phương trình: ( ) ( )
a f x a f x b
− + + =
( )
( )
( )
( ) ( )
2
2
2 2 2 2 2
.
2 2 4 2
a f x a
a f x a
b
a a f x a f x b a
≥ ≥ −

≥ ≥ −

 
⇔ =
⇔
 
 
+ − − =−
   
 
Ý tưởng: Bài toán xuất hiện ba căn thức, nhưng có điều đặc biệt ở đây là căn thức còn lại là tích của hai
căn thức kia. Mặt khác 2
1 , 1
x x
− − có sự đồng nhất hệ số, do đó ta sẽ nhóm hai căn này lại nên ta
được nhân tử chung như sau: ( )
2
1 1 1 1 1
x x x x
− − − = − + − . Và ta mong muốn biểu thức
Liên hệ tài liệu word toán SĐT (zalo): 17
Website:
3 3 1
x x
+ − + sẽ phân tích được biểu thức có chứa 1 1
x + − . Thật vậy nếu coi ( ) 3 3 1
h x x x
= + − +
là một phương trình bậc hai ẩn 1
x + ta sẽ thấy: ( ) ( )( )
1 3 1 2 1 1 1 2
h x x x x x
= + − + + = + − + − .
Chính vì thế bài toán của ta được giải quyết như sau:
2
3 1 3 1 1
x x x x
+ + − = + + −
( )( ) ( )
1 1 1 2 1 1 1 0
x x x x
⇔ + − + − + − + − =
( )( )
1 1
1 1 1 1 2 0
1 1 2
x
x x x
x x
 + =
⇔ + − + + − − = ⇔ 
+ + − =


.
Phần còn lại chỉ là việc bình phương các phương trình và tìm nghiệm như ở trên đã nêu. Ta được
nghiệm của phương trình là 0
x =
Bài toán kết thúc.
BÀI TẬP TƯƠNG TỰ
1) Giải phương trình: 2
4 4 3 2 2
x x x x
+ + − = + + −
Đáp số: 1
x =
2) Giải phương trình: 2
2 3 1 4 3 2 1 1 2
x x x x
+ + − = + + −
Đáp số: 0
x =
Ví dụ:
Giải phương trình: 3 3 1 1
x x x
+ + + = −
Giải:
Phương trình tương đương với: 2 3 2 3 1 2 2
x x x
+ + + = − .
Đặt 2 2
3
2 2
3 1
u x
v u x
v x

= +

⇒ − = −

= +


với ; 0
u v > , ta được
( ) ( )( )
2 2
2 2 0
u v v u v u v u
+ = − ⇔ + − − =
2 3 1 3 2 3 1 7 4 3
v u x x x x x
⇔ = + ⇔ + = + + ⇔ + = + + +
( ) 2
3
2 6 4 3 2 3 3
4 3 6 9
x
x x x x
x x x
≥


⇔ − = + ⇔ + = − ⇔ 
+ = − +


2
3 5 28
5 28
10 3 0 3
x x
x
x x x
≥ 
 = ±

⇔ ⇔ ⇔ = +
 
− − = ≥

 
Nhận xét: Bài toán sử dụng phương pháp đặt ẩn phụ để đưa phương trình từ phức tạp về đơn giản hóa,
sau đó dùng phương pháp nâng lũy thừa để tìm nghiệm của phương trình.
Liên hệ tài liệu word toán SĐT (zalo): 18
Website:
* NHẮC LẠI KIẾN THỨC VÀ PHƯƠNG PHÁP:
- Hằng đẳng thức cơ bản: ( )( )
2 2
a b a b a b
− = − + .
- Cách giải phương trình vô tỷ dạng: ( ) ( )
f x g x m
= +
( ) ( )
( ) ( )
( )
( ) ( )
( ) ( ) ( )
2
2
; 0 ; 0
2
f x g x f x g x
f x g x m m g x
f x g x m
 ≥ ≥

 
⇔ ⇔
 
− − =
= +
 


( ) ( )
( ) ( ) ( )
1
2
2 2
2
; 0
.
4
f x g x x x
x x
f x g x m m g x
≥
 =


⇔ ⇔
  =
 
− − = 
 

Ý tưởng: Một bài toán đẹp, quan sát VT của phương trình có xuất hiện hai căn bậc hai riêng biệt đồng
thời trong căn chứa các biểu thức bậc nhất, cũng như VP của phương trình cũng là một biểu thức bậc
nhất, nên vậy ta có thể nâng lũy thừa để đưa phương trình ban đầu về phương trình bậc bốn. Nhưng nếu
tinh ý một chút, ta có ( ) ( ) ( )
3 1 3 2 2 2 1
x x x x
+ − + = − = − ,
Do vậy đặt
3
3 1
u x
v x

= +


= +


với ; 0
u v ≥ , suy ra: ( )
2 2
2 2 2 1
v u x x
− = − = − . Khi đó, phương trình đã cho
tương đương với:
( ) ( )( )
2 2
2 2 0
v u v u v u v u
− = + ⇔ + − − = (i)
- Vì ; 0
u v ≥ nên phương trình (i)
0
2
u v
v u
= =

⇔  = +

, với 0
u v
= = phương trình vô nghiệm nên ta chỉ cần
giải phương trình: 2
v u
= + .
- Với 2
v u
= + , ta có: 3 1 3 2 3 1 7 4 3
x x x x x
+ = + + ⇔ + = + + +
( ) 2
3
2 6 4 3 2 3 3
4 3 6 9
x
x x x x
x x x
≥


⇔ − = + ⇔ + = − ⇔ 
+ = − +


2
3 5 28
5 28
10 3 0 3
x x
x
x x x
≥ 
 = ±

⇔ ⇔ ⇔ = ±
 
− − = ≥

 
Bài toán kết thúc.
BẢI TẬP TƯƠNG TỰ
1) Giải phương trình: 4 1 2 2 2 1
x x x
+ + + = −
Đáp số: 2 5
x= +
2) Giải phương trình: 3 5 2 1 4
x x x
+ + + = −
Liên hệ tài liệu word toán SĐT (zalo): 19
Website:
Đáp số:
5 87
8
x
−
=
Ví dụ 5:
Giải phương trình:
( )
2
3 7
2 1
x
x
x x
+
+ =
+
Giải:
Điều kiện 0
x >
Phương trình tương đương: ( ) 2
3
2 1 7
x x x
x
+ + = +
Chia hai vế cho 0
x ≠ , ta được:
1 3 7 3 1 3 4 3 3 2
2 1 2 1 0 2 0
x x x x x x
x x x x x x x x x x
  
     
+ + = + ⇔ + − + + + = ⇔ + − + − =
  
        
        
+ Giải: 2 1
3 3
2 4 4 3 0
3
x
x x x x
x
x x
=

+ = ⇔ + = ⇔ − + = ⇔  =

+ Giải: 2
2
3 2 3 4
3 4 0
x x x x
x x x x
+ = ⇔ + = ⇔ + − =
( )( )
2
1 4 0 1
x x x x
⇔ − + + = ⇔ =
Đáp số 1; 3
x x
= = .
Nhận xét: Bài toán sử dụng phương pháp ẩn phụ không hoàn toàn, sau đó nâng lũy thừa tìm nghiệm của
phương trình ban đầu.
* NHẮC LẠI KIẾN THỨC VÀ PHƯƠNG PHÁP
- Phương pháp đặt ẩn phụ không hoàn toàn: xét một phương trình bậc hai có dạng
( ) ( )
2
. . 0
mf x t ng x t k
+ + =(*), trong đó t là ẩn phụ được biểu diễn dưới dạng ( )
t h x
= . Khi đó, ta có:
( ) ( )
2
4
t ng x kmf x
= −
 
 
Δ , với t
Δ bắt buộc là một số chính phương. Do đó, tìm được nghiệm của (*),
đó là
( )
( )
( )
( )
( )
( )
;
t t
ng x ng x
t h x t h x
mf x mf x
− + − −
= = = =
Δ Δ
- Cách giải phương trình: ( ) ( )
( ) ( )
( ) ( )
2
; 0
.
f x g x
f x g x
f x g x
≥


= ⇔ 
=


.
Liên hệ tài liệu word toán SĐT (zalo): 20
Website:
Ý tưởng: Trước hết, ta cần quy đồng mẫu số bài toán, như vậy ta sẽ được phương trình có dạng
( ) ( ) ( )
.
f x g x h x
= và nếu nâng lũy thừa hai vế, ta sẽ thu được một phương trình bậc 5. Và phương
trình bậc 5 nếu không có nghiệm nguyên thì sẽ rất khó để giải quyết. Vậy nên ta cần nghĩ đến hướng tư
duy khác, đó là bài toán xuất hiện căn thức
3
x
x
+ nên ta mong muốn sẽ tạo được lượng
2
3
k x
x
 
+
 
 
 
để có thể đưa về phương trình bậc hai, sau đó đặt
3
t x
x
= + để sử dụng phương pháp ẩn phụ không an
toàn.
- Ta có:
( )
( )
2
2
3 7 3
2 1 7
2 1
x
x x x x
x x x
+
+ = ⇔ + + = +
+
Chia cả hai vế của phương trình cho x, ta có:
1 3 7 3 1 3 4
2 1 2 1 0
x x x x
x x x x x x x
   
+ + = + ⇔ + − + + + =
   
   
(*)
- Đặt
3
0
t x
x
= + > , khi đó ta có ( ) 2 1 4
* 2 1 0
t t
x x
 
⇔ − + + =
 
 
.
Có:
2 2
1 4 1
1 1
t
x x x
   
= + − = −
   
   
Δ nên suy ra được
( )
( )
3
1 1
2
2
1 1
2
1 1 3 2
1 1
x i
t
t
x
x x
t
t x x ii
x x x x

 + =
=
= + + −  

 
⇔ ⇔

 
=
 =+ + − +  + =


 
- Giải (i), ta có (i) 3 1
3
4 4 3 0
3
x
x x x
x
x
=

⇔ + = ⇔ − + = ⇔  =

- Giải (ii), ta có (ii) 3
2
3 4
3 4 0 1
x x x x
x x
⇔ + = ⇔ + − = ⇔ =
Bài toán kết thức.
BÀI TẬP TƯƠNG TỰ:
1) Giải phương trình: ( ) ( )( )
3 4 12 28
x x x x
+ − + = −
Đáp số: ( )
4 2 1 ; 31 3
x x
= − = −
2) Giải phương trình: 3 2
2 2
x x x x
− = − −
Đáp số:
1 5 1 65
; .
2 8
x x
+ −
= =
Liên hệ tài liệu word toán SĐT (zalo): 21
Website:
Ví dụ 6:
Giải phương trình: ( )( )
9 2012 6 2012 9 6 .
x x x x
+ + += + + +
Giải:
Điều kiện 6
x ≥ −
Phương trình đã cho tương đương với: ( )( )
9 2012 6 1 0
x x
+ − + − =
+ Giải ( )
2
9 2012 0 2012 9 4048135
x x
+ − = ⇔ = − =
+ Giải 6 1 0 5
x x
+ − = ⇔ =
−
Vậy phương trình có hai nghiệm: 4048135; 5
x x
= = −
Nhận xét: Bài toán sử dụng phương pháp nhóm nhân tử chung và nâng lũy thừa tìm nghiệm của phương
trình.
* NHẮC LẠI KIẾN THỨC VÀ PHƯƠNG PHÁP:
- Các giải phương trình dạng:
( )
( ) ( )
( )
( )
( )
( )
( )
2
2
. 0 .
f x m f x m
f x m g x n
g x n
g x n
 =  =

− − = ⇔ ⇔ 
 =

= 

Ý tưởng: Bài toán cho hết sức đơn giản, với sự xuất hiện của hai căn thức
( )( )
9; 6; 9 6
x x x x
+ + + + nên không khó để nhóm được nhân tử chung như sau:
( )( )
9 2012 6 2012 9 6
x x x x
+ + += + + +
( )( )
9 9 6 2012 6 2012 0
x x x x
⇔ + − + + + + − =
( ) ( )
9 1 6 2012 1 6 0
x x x
⇔ + − + − − + =
( )( )
1 6 9 2012 0
x x
⇔ − + + − =
Bài toán kết thúc.
BÀI TẬP TƯƠNG TỰ
1) Giải phương trình: 2
2 6 7 2 13 42
x x x x
+ + + = + + +
Đáp số: 3; 5
x x
=
− =
−
2) Giải phương trình: 2
4 2 3 2 7 12
x x x x
+ + + = + + +
Đáp số: 0; 2
x x
= = −
Liên hệ tài liệu word toán SĐT (zalo): 22
Website:
Ví dụ 7:
Giải phương trình: 8 2 2 1
x x
+ − − =
Giải:
Điều kiện 8 2
x
− ≤ ≤
+ Nếu 1 2 8 9 3
x x
< ≤ ⇒ + > =
2 2 1 1 8 2 2 3 2 1
x x x
− < − = ⇒ + − − > − = ⇒ loại.
+ Nếu 8 1
x
− ≤ < , tương tự ta có: 8 2 2 9 2 1 1
x x
+ − − < − ==> loại
Với 1
x = , thỏa mãn phương trình.
Đáp số 1
x = .
Nhận xét: Bài toán sử dụng phương pháp nhẩm nghiệm và đánh giá theo miền nghiệm để chứng minh
nó có nghiệm duy nhất.
Ý tưởng: Đây một bài toán đơn giản nhưng đòi hỏi đi theo phương pháp đánh giá thì cần đoán trước
nghiệm của phương trình. Đầu tiên ta sẽ ưu tiên nghiệm nguyên trước, với nghiệm nguyên thì các biểu
thức chứa căn phải là một số chính phương, tức là
2
2
8
2
x k
x h
 + = ∈


− = ∈




.
Với điều kiện chặn của [ ]
8; 2
x ∈ − ] thì ta sẽ thử một vài giá trị nguyên của x và thấy rằng tại 1
x =
thỏa mãn phương trình. Công việc còn lại là ta sẽ đi chứng minh nó là nghiệm duy nhất. Nó là nghiệm
duy nhất nếu với [ 8; 1) (1; 2]
x ∈ − ∪ thì phương trình bài cho vô nghiệm. Đi xét từng trường hợp ta có:
- Với [ 8; 1)
x ∈ − suy ra 8 2 2 9 2 1 1
x x
+ − − < − =.
- Với (1; 2]
x ∈ suy ra 8 2 2 9 2 1 1
x x
+ − − > − =.
Cả hai trường hợp trên đều chứng minh 1
x = là nghiệm duy nhất của phương trình.
Bài toán kết thúc.
Liên hệ tài liệu word toán SĐT (zalo): 23
Website:
BÀI TẬP TƯƠNG TỰ
1) Giải phương trình: 7 8 3 4
x x x
+ − = +
2) Giải phương trình: 9 2 1 1
x x
+ − − =
Ví dụ 8:
Giải phương trình: ( )( )
3 1 1 1
x x x
+ − − + =
Giải:
Điều kiện 0 1
x
≤ ≤
Phương trình tương đương với:
( ) ( )
3
1 1 1 3 1 1 3
3
x x x x
x x
− + = ⇔ − + = + +
+ +
Nếu ( )
0 1 3 1 1 3
x x
≤ ≤ ⇒ − + > , đồng thời 3 1 4 3
x x
+ + < + =, suy ra VT > VP (loại).
Thử lại ta thấy 1
x = là nghiệm.
Nhận xét: Bài toán kết hợp giữa phương pháp nhân liên hợp và phương pháp đáng giá để tìm nghiệm
của phương trình.
* NHẮC LẠI KIẾN THỨC VÀ PHƯƠNG PHÁP
- Biểu thức liên hợp: ( )( )
x m x x m x x m x
+ − = + − + +
m
x m x
x m x
+ − =
+ +
với 0; 0
x x m
≥ + ≥ .
- Đánh giá: ( ) ( ) ( )
m f x n g x h x
− +
= + với
( ) 0
2
m f x
n m
≥ ≥



=


.
Ta có: ( )
( )
( ) ( )
0
2
m f x n n
f x m
g x h x m
 − + >

≤ < ⇒ 
+ <


, suy ra phương trình vô nghiệm.
Vậy x m
= là nghiệm của phương trình đã cho
Ý tưởng: Bài toán xuất hiện ba căn thức nằm trong một tích, sẽ rất khó để định hình ra hướng giải, ẩn
phụ sẽ rất phức tạp. Nhưng nếu xét hai căn thức đầu tiên ta thấy ( ) ( )
2 2
3 3
x x
− − =
. Vì thế ta sẽ nghĩ
ngay đến chuyện dùng hằng đẳng thức dạng: ( )( )
2 2
a b a b a b
− = − + . Khi đó phương trình đã cho
tương đương với:
Liên hệ tài liệu word toán SĐT (zalo): 24
Website:
( )
3
1 1 1 3 1 3 3
3
x x x x
x x
− + = ⇔ − + = + +
+ +
(i)
Với phương trình (i), ta sẽ đi nhẩm một vài giá trị nghiệm đẹp thỏa mãn các yêu cầu là 1 0
x
≥ ≥ và các
biểu thức trong căn thức là số chính phương vì thế ta khẳng định nó có nghiệm duy nhất 1
x = , đồng
thời 1 lại là miền chặn của biến do đó ta sẽ đi đánh giá phương trình (i). Tức là với 0 1
x
≤ < , ta sẽ
chứng minh (i) vô nghiệm như sau:
3 1 3 3
0 1
3 3
x
x
x x
 − + >

≤ < ⇒ 
+ + <


=> VT > VP => (i) vô nghiệm
Vậy ta kết luận 1
x = là nghiệm của phương trình đã cho.
Bài toán kết thúc.
BÀI TẬP TƯƠNG TỰ
1) Giải phương trình: ( )( )
2 1 2 1 1
x x x
+ − − − + =
Đáp số: 2
x =
2) Giải phương trình: ( )( )
1 2 3 1 1
x x x
+ − − − + =
Ví dụ 9:
Giải phương trình: ( )( )
4 2 4 2 2
x x x
+ − − + =
Giải:
Điều kiện 4 4
x
− ≤ ≤
Phương trình đã cho tương đương với: ( )
4 2 2
4 2
x
x x
x
− + =
+ +
.
+ Với 0
x = là nghiệm.
+ Giải: ( )
4 2 2 4 2
x x
− += + +
Đặt 4; 4
u x v x
= + = − ta thu được
( )
2
2 2
2 2
2 2
2 2 8 5 8 4 0
8
v u
u u u u
u v
= +

⇒ + + = ⇔ + − =

+ =

( )
2 14
; 2 96
5 5 4
5 25
2
u v
x x
u l

= =

⇔ ⇒ + = ⇔ =
−

= −


(thỏa mãn)
Vậy phương trình có hai nghiệm:
96
0;
25
x x
= = − .
Nhận xét: Sử dụng phương pháp nhân liên hợp, sau đó đặt ẩn phụ tìm nghiệm của bài toán.
Liên hệ tài liệu word toán SĐT (zalo): 25
Website:
NHẮC LẠI KIẾN THỨC VÀ PHƯƠNG PHÁP
- Hằng đẳng thức ( )( ) a b
a b a b a b a b
a b
−
− = − + ⇔ + =
−
.
- Giải phương trình: tổng quát dạng ( ) ( )
f x g x m
= +
( ) ( )
( ) ( ) ( )
( ) ( )
( ) ( ) ( )
2
2
2 2
; 0
2 4
f x g x m
f x g x
f x g x m g x m f x g x m mg x
 ≥ +
≥

 
⇔ ⇔
 
= + +  
− − =
 
  

Ý tưởng: Không khó để nhận thấy, phương trình có một nghiệm là 0
x = .
Đồng thời vế trái của phương trình có xuất hiện biểu thức 4 2
x + − , dễ thấy rằng
( ) ( )( )
2
2
4 2 4 2 4 2
x x x x
= + − = + − + + . Vì thế, phương trình đã cho tương đương với:
( )( ) ( )( )
4 2 0 0
4 2 4 2 2 4 2 4 2
4 2 4 2
x x
x x x x
x x
 + − = ⇔ =
+ − − + = + − + + ⇔ 
− = + +


Phương trình còn lại có thể giải bằng cách tổng quát nêu ở trên, hoặc có thể giải quyết bằng cách đặt ẩn
phụ như sau: ( )
4
; 0
4
v x
u v
u x

= −

≥

= +


. Ta có hệ phương trình
2 2
8 2 14 2 96
; 4
5 5 5 25
2 2
v u
u v x x
v u
 + =
⇔ = = ⇒ + = ⇔ =
−

= +

.
Bài toán kết thúc.
BÀI TẬP TƯƠNG TỰ
1) Giải phương trình: ( )( )
1 1 1 1 2
x x x
+ − − + = .
Đáp số:
24
0;
25
x x
= = −
2) Giải phương trình: ( )( )
9 1 9 1 3
x x x
+ − − + =
Đáp số: 0
x =
Ví dụ 10:
Giải phương trình:
1 1
4 1 5
2 1 2
x x
x x
+ +
= +
+ +
Giải:
Liên hệ tài liệu word toán SĐT (zalo): 26
Website:
Phương trình tương đương với:
1 1
5 4 1
2 1 2
x x
x x
− = − −
+ +
.
Giả sử:
1 1
1 2 1 2
2 1 2
x x x
x x
> ⇒ + > + ⇒ <
+ +
=> 0 5 4 1 1
VT x x x
< ⇒ < − ⇒ < (mâu thuẫn).
Giả sử 0 1
x
≤ < , lập luận tương tự thu được 1
x > (mâu thuẫn). Thử lại thu được nghiệm duy nhất 1
x =
Nhận xét: Bài toán sử dụng phương pháp đánh giá miền nghiệm để tìm nghiệm của phương trình.
Ý tưởng: Quan sát bài toán, ta thấy có vẻ nó hơi rắc rối một chút vì xuất hiện tới bốn căn thức mà căn
lại còn ở dưới mẫu thì sẽ rất khó khăn trong việc giải bằng các công cụ mà ta thường làm như ẩn phụ,
nâng lũy thừa...Và hướng cuối cùng ta nghĩ đến là đánh giá. Để đánh giá được nó, ta cần tìm nghiệm
trước, vẫn là ưu tiên hướng nghiệm nguyên đồng thời cân bằng căn thức với căn thức, phân thức với
phân thức ta có:
1 1
1; 4 1 5 1
2 1 2
x x x x
x x
= ⇔ = + = ⇔ =
+ +
Và bây giờ, ta sẽ đi chứng minh 1
x = là nghiệm duy nhất của phương trình. Hay nói cách khác, với
1
x > hoặc 1
x < nó sẽ vô nghiệm. Biến đổi phương trình đã cho về dạng:
( )( )
1 1 2 2 1
4 1 5 5 4 1
2 1 2 2 1 2
x x
x x x x
x x x x
+ − +
+ += + ⇔ = − +
+ + + +
* Với 1
x > , ta có
2 2 1 2 2 1 0
5 4 1 5 4 1 0
x x x x
x x x x

+ < + + − − <
 
⇔
 
> + − + >
 

, suy ra phương trình vô nghiệm.
* Với 1 0
x
> ≥ , ta có:
2 2 1 2 2 1 0
5 4 1 5 4 1 0
x x x x
x x x x

+ > + + − + >
 
⇔
 
< + − + <
 

, suy ra phương trình vô nghiệm.
Vậy 1
x = là nghiệm duy nhất của phương trình.
Bài toán kết thúc.
BÀI TẬP TƯƠNG TỰ
1) Giải phương trình:
1 1
3 1 4
2 1 2 1
x x
x x
+ +
= +
+ +
2) Giải phương trình:
2 2
1 1
3 1 4
2 1 2
x x
x x x x
+ +
= +
+ + + +
Ví dụ 11:
Giải phương trình: 2 3
8 3 8
x x
+= +
Liên hệ tài liệu word toán SĐT (zalo): 27
Website:
Giải:
Điều kiện 2
x ≥ −
Đặt ( )
2
2
; 0
2 4
a x
a b b
b x x

= +

≥ >

= − +


.
Ta có 2 2 2 2
2 2 4 2 4 8
a b x x x x
+ = + + − + = + , suy ra ( )( )
2 2
2 3 2 0
a b ab a b a b
+ = ⇔ − − =
+ TH1: 2 2 1
2 2 4 3 2 0
2
x
a b x x x x x
x
=

= ⇔ + = − + ⇔ − + = ⇔  =

.
+ TH2: ( ) 2 2
2 4 2 2 4 6 4 0 3 13
a b x x x x x x
= ⇔ + = − + ⇔ − − = ⇔ = ± .
Nhận xét: Bài toán sử dụng phương pháp đặt ẩn phụ rồi đưa về phương trình đẳng cấp bậc hai, tìm mối
quan hệ giữa hai ẩn phụ sau đó nâng lũy thừa tìm nghiệm của phương trình ban đầu.
NHẮC LẠI KIẾN THỨC CŨ VÀ PHƯƠNG PHÁP:
- Hằng đẳng thức: ( )( )
2 2 2 2
u v u v u uv v
+ = + − +
- Cách giải phương trình bậc hai dạng: 2 2
. . . 0
a x b xy c y
+ + = (*).
Làm nháp: Chia cả hai vế của phương trình (*) cho 2
y ( vì là làm nháp nên ta cứ coi là 0
y ≠ ), khi đó
(*)
2
. . 0
x x
a b c
y y
 
⇔ + + =
 
 
.
Coi đây là phương trình bậc hai ẩn
x
t
y
= (đã biết cách giải) thì nghiệm t tìm được sẽ biểu hiện mối
quan hệ giữa ,
x y . Cụ thể là 1
2
.
x t y
x t y
=

 =

Ý tưởng: Bài toán này có thể giải bằng cách lũy thừa hai vế, đưa về phương trình bậc bốn và vấn đề ta
gặp phải sẽ là nghiệm của phương trình bậc bốn. Ta đặt dấu hỏi cho nó? Nhưng nếu đế ý 3
8 2
= nên
trong căn thức sẽ xuất hiện ngay hằng đẳng thức ( )( )
3 3 3 2
8 2 2 2 4
x x x x x
+ = + = + − + . Vậy nên trong
căn bậc hai đã xuất hiện hai tích. Do đó để xuất hiện phương trình đẳng cấp thì ta cần làm công việc sau,
đó là đồng nhất hệ số của biểu thức dạng: ( ) ( )
2 2
1
1
2 4 2 8 2 0
2
4 2 8
a
a
a x x b x x b a
b
a b
=

=


− + + + = + ⇔ − = ⇔ 
 =

 + =

.
Đến đây, ta viết lại phương trình ban đầu như sau:
( ) ( ) ( )( )
2 3 2 2
8 3 8 2 4 2 2 3 2 4 2
x x x x x x x x
+ = + ⇔ − + + + = − + + (i)
Liên hệ tài liệu word toán SĐT (zalo): 28
Website:
* Đặt
2
2 4 0
2 0
u x x
v x
 = − + >


= + ≥


nên phương trình trên trở thành:
(i) ( )( )
2 2
3 2 0 2 0
2
u v
u uv v u v u v
u v
=

⇔ − + = ⇔ − − = ⇔  =

* Hoặc nhận thấy ( )
2
2
2 4 1 3 0
x x x
− + = + + ≠ nên phương trình (i) tương đương với:
2 2
2 2
2. 3 1 0
2 4 2 4
x x
x x x x
+ +
− + =
− + − +
. Có thể đặt căn thức là ẩn phụ để Giải phương trình: bậc hai.
* Hai hướng trên đều cho ta:
TH1. Với 2
2
2 1
2 4 2
2
3 2 0
x x
x x x
x
x x
≥ − =
 
− + = + ⇔ ⇔
 
=
− + = 

.
TH2. Với 2
2
2
2 4 2 2 3 13
6 4 0
x
x x x x
x x
≥ −

− + = + ⇔ ⇔ = ±

− − =

.
Bài toán kết thúc.
BÀI TẬP TƯƠNG TỰ
1) Giải phương trình: 2 3 2
3 5 2 6 4
x x x x x
+ −
= + − + .
Đáp số:
1 13
2
x
− +
=
2) Giải phương trình: 2 4 2
3
3 1 1
3
x x x x
− + =
− + +
Đáp số: 1
x =
II. BÀI TẬP
Bài 1:
Giải phương trình: 3 1 2 3
x x
+ + − =.
Giải
Điều kiện:
1
2
3
x
− ≤ ≤ .
Phương trình đã cho tương đương với ( )( )
2 3 2 3 1 2 9
x x x
+ + + − =
2
2 2
3 0
3 5 2 3
3 5 2 6 9
x
x x x
x x x x
− >

⇔ − + + = − ⇔ 
− + + = − +

Liên hệ tài liệu word toán SĐT (zalo): 29
Website:
2
1
4 11 7 0 7
4
x
x x
x
=


⇔ − + = ⇔
 =

Đối chiếu với điều kiện ta được được nghiệm:
7
1;
4
x x
= = .
Nhật xét: Bài toán sử dụng phương pháp nâng lũy thừa (bình phương) hai vế tìm nghiệm của phương
trình.
NHẮC LẠI KIẾN THỨC VÀ PHƯƠNG PHÁP:
- Phương trình dạng ( ) ( ) ( ) ( )
( )
2
2
0
f x g x m f x g x m
+ =
> + =
( ) ( ) ( ) ( ) ( ) ( ) ( ) ( )
2 2
2 2
f x g x f x g x m f x g x m f x g x
⇔ + + = ⇔ = − −
( ) ( )
( ) ( ) ( ) ( )
2
1
2
2
2
4
m f x g x x x
x x
f x g x m f x g x
 ≥ + =


⇔ ⇒
  =
 
= − − 
  

- Phương trình trên có cách giải khác như sau:
( ) ( ) ( ) ( )
f x g x m f x m g x
+ =
⇔ =
−
( )
( ) ( ) ( )
( )
( ) ( ) ( )
2 2
2 2
m f x m f x
f x m m g x g x m g x g x m f x
≥ ≥
 
 
⇔ ⇔
 
= − + = + −
 
 
( ) ( ) ( )
( ) ( ) ( )
( )
2
1
2
2 2
2
;
4
m f x g x m f x x x
x x
m g x g x m f x
 ≥ + ≥ =


⇔ ⇒
  =
= + − 


Ý tưởng: Đây là một bài phương trình cơ bản, dạng toán một vế chưa hai căn thức vế còn lại là một
hằng số thì phương pháp nâng lũy thừa hai vế là một phương pháp tối ưu nhất.
Bài toán kết thúc.
BÀI TẬP TƯƠNG TỰ
1) Giải phương trình: 3 1 1 8
x x
+ + + =
Đáp số: 8
x =
2) Giải phương trình: 7 4 1 3
x x
+ − + =
Đáp số: 3
x = .
Bài 2:
Giải phương trình: ( )( )
2
1 1 2 2 1 8
x x x
+ + − + − =
Liên hệ tài liệu word toán SĐT (zalo): 30
Website:
Giải
Điều kiện: 1 1
x
− ≤ ≤
Đặt: 1 1
t x x
= + + −
( )( )
2 2
1 2 1 1 1 2 2 1
t x x x x x
⇒ = + + + − + − = + −
Khi đó phương trình đã cho trở thành: 2
. 8
t t =
3 2 2
8 2 2 2 1 4 1 1 0
t t x x x
⇔ = ⇔ = ⇔ + − = ⇔ − = ⇔ =
Vậy phương trình đã cho có nghiệm duy nhất là 0
x = .
Nhận xét: Bài toán sử dụng phương pháp đặt ẩn phụ giải phương trình
NHẮC LẠI KIẾN THỨC VÀ PHƯƠNG PHÁP:
- Hằng đẳng thức quen thuộc: ( )
2 2 2
2
u v u uv v
+ = + + .
- Với ( ) [ ; ]
f x a b
∈ − , đặt ( ) ( )
t a f x b f x
= + + − , khi đó
( )
( ) ( )
( ) ( )
( ) ( )
( )
2
2
2
2
t a b
t a b a f x b f x a f x b f x
− −
=
+ + + − ⇔ + − =
Ý tưởng: Nhận thấy ở hai căn thức, ta có tổng bình phương của chúng là một hằng số, mặt khác tích của
chúng có liên quan đến biểu thức còn lại trong phương trình.
- Ta có: ( ) ( ) ( )( )
2 2
2
1 1 1 1 2; 2 1 2 1 1
x x x x x x x
+ + − = + + − = − = + −
- Do đó: ( ) ( ) ( )( )
2 2
2
2 2 1 1 1 2 1 1
x x x x x
+ − = + + − + + −
( )
2
1 1 .
x x
= + + −
- Đặt 1 1
t x x
= + + − , phương trình đã cho trở thành: 3
8
t =
2 2
2 2 2 1 4 1 1 0
t x x x
⇔ = ⇔ + − = ⇔ − = ⇔ =
Bài toán kết thúc.
BÀI TẬP TƯƠNG TỰ
1) Giải phương trình: ( )
2
6 2 4 3 2 2
x x x
+ − = + + −
Đáp số: 2
x = ±
2) Giải phương trình: ( )( )
7 2 2 4 3 2 2 4 4 3
x x x x
+ − + = − + +
Đáp số: .
5 4 3
4
x
±
= −
Liên hệ tài liệu word toán SĐT (zalo): 31
Website:
Bài 3:
Giải phương trình: ( )
2
2 2 2 4 2 3
x x x x
+ + − + − = −
Giải:
Điều kiện 2
x ≥ .
Đặt: 2 2 0
t x x
= + + − >
2 2 2
2 2 2 4 2 2 4
t x x x x x
⇒ =
+ + − + − =+ −
Phương trình đã cho tương đương: 2
2 2 2 2 4 6
x x x x
+ + − + + − =
2
6 2
t t t
⇔ + − ⇔ = hoặc 3
t = − (loại).
Với 2
t = ta có 2 2 2
x x
+ + − =
Do điều kiện 2
x ≥ , ta có: 2 2 4 0 2
x x
+ + − ≥ + =
Suy ra phương trình có nghiệm duy nhất 2
x = .
Nhận xét: Bài toán sử dụng phương pháp đặt ẩn phụ, đưa phương trình ban đầu về phương trình bậc hai
tìm ẩn, sau đó dùng phương pháp nâng lũy thừa tìm nghiệm của phương trình ban đầu.
* NHẮC LẠI KIẾN THỨC VÀ PHƯƠNG PHÁP
- Cách giải phương trình bậc hai tổng quát: 2
. . 0
a t b t c
+ + =
- Hằng đẳng thức: ( )( )
2 2
a b a b a b
− = − + và ( )
2 2 2
2
a b a ab b
+ = + +
- Phương trình có dạng: ( ) ( )
f x g x m
+ =
, với m là số thực dương thì có hai cách nâng lũy thừa
như sau:
Cách 1. Bình phương hai vế của phương trình, ta có:
( ) ( )
( ) ( )
( ) ( ) ( ) ( ) 2
0; 0
2 .
f x g x
f x g x m
f x g x f x g x m
≥ ≥


+ =
⇔ 
+ + =


( ) ( )
( ) ( ) ( ) ( )
1
2
2
2
0; 0
.
4 .
f x g x x x
x x
f x g x m f x g x
≥ ≥
 =


⇔ ⇒
  =
 
= − − 
  

Cách 2. Chuyển ( )
g x sang VP rồi bình phương, ta có:
( ) ( )
( )
( ) ( ) ( )
2
2
m g x
f x m g x
f x m m g x g x
 ≥

=
− ⇔ 
=
− +


Liên hệ tài liệu word toán SĐT (zalo): 32
Website:
( )
( ) ( ) ( )
( )
( ) ( ) ( )
2
2 2 2
.
2 4
m g x
m g x
m g x m f x g x m g x m f x g x

 ≥
≥
 
⇔ ⇔
 
 
= − + = − +
   
 
Chú ý:
- Nếu ( ) ( )
f x g x k
− =
; k là hằng số thì ta có thể sử dụng cách liên hợp như sau:
( ) ( )
f x g x m
+ =
(i)
( ) ( )
( ) ( ) ( )
( ) ( ) ( )
( )
f x g x f x g x m f x g x
⇔ + − = −
( ) ( ) ( ) ( )
( ) ( ) ( )
k
f x g x m f x g x f x g x
m
⇔ − = − ⇔ − = (ii)
Lấy (i) + (ii), ta được ( ) ( )
2
2 4
k k
f x m f x m
m m
 
= + ⇔ = +
 
 
.
- Nếu c a b
≥ ≥ và x c
≥ suy ra x a x b c a c b
+ + + ≥ + + + .
Dấu “=” xảy ra khi và chỉ khi x c
= .
Ý tưởng: Bài toán xuất hiện ba căn thức bậc hai ở VT là: 2, 2
x x
+ − và 2
4
x − , áp dụng hằng
đẳng thức ( )( )
2 2
a b a b a b
− = − + dễ thấy được rằng 2
4 2. 2
x x x
− = − + , hay nói cách khác căn
thức cuối cùng chính là tích của hai căn thức còn lại. Đồng thời nếu chuyển ( )
2 3 x
− từ VP sang VT thì
sẽ xuất hiện 2x , mà ( ) ( )
2 2
2 2 2 2 2
x x x x x
= + + − = + + − do đó VT của phương trình ban đầu có:
( ) ( )
2 2
2 2 2. 2. 2 2 2 6 0
x x x x x x
+ + − + − + + + + − − =
( )
2
2 2 2 2 6 0
x x x x
⇔ + + − + + + − − =
- Đặt 2 2 0
t x x
= + + − > thì phương trình (*) được viết lại thành:
( )( )
2
0
0
2
2 3 0
6 0
t
t
t
t t
t t
>
> 
 
⇔ ⇔ =
 
− − =
+ − = 
 
- Với 2
t = suy ra 2
2
2 2 2
2 2 4 4
x
x x
x x
≥


+ + − = ⇔ 
+ − =


2
2
2
4 2
x
x
x x
≥


⇔ ⇔ =

− = −


Đến đây có thể đánh giá như lời giải là: 2 2 2 4 0 2 2
x x x x
≥ ⇒ + + − ≥ + = ⇒ = .
- Vì ( )
2 2 4
x x
+ − − = nên giải phương trình 2 2 2
x x
+ + − = theo chú ý như sau:
( )
2 2 2 4 2 2 2
x x x x
+ + − = ⇔ = + + −
Liên hệ tài liệu word toán SĐT (zalo): 33
Website:
2 2 2 2 2 4 2
x x x x
⇔ + − = ⇒ + = ⇔ =
Bài toán kết thức.
BÀI TẬP TƯƠNG TỰ
1) Giải phương trình: 2
2 1 2 1 2 1 3 2
x x x x
+ + − + − = −
Đáp số: phương trình vô nghiệm thực.
2) Giải phương trình: 2 2
2 5 2 2 5 1 5 2
x x x x x
+ + + − = − + +
Đáp số:
17
16
x =
Bài 4:
Giải phương trình: 2
3 1 3 1 1
x x x x
+ + − = + + −
Giải:
Điều kiện 1 1
x
− ≤ ≤ .
Phương trình tương đương với:
( )
2
2
2 1 1 1 1 2 1
x x x x x
+ + + − = + + − + +
( ) ( )
1 1 2 1 2 1 1 1
x x x x x x
⇔ + + − + + = + + + + −
( )( )
1 1 2 1 1 0
x x x
⇔ + + − − + − =
+ Giải 2 2
1 1 2 2 2 1 4 1 1 0
x x x x x
+ + − = ⇔ + − = ⇔ − = ⇔ =
+ Giải 1 1 0
x x
+ = ⇔ =
Đáp số 0
x =
Nhận xét: Bài toán sử dụng phương pháp nhóm nhân tử chung, sau đó nâng lũy thừa bậc hai để tìm
nghiệm của phương trình.
* NHẮC LẠI KIẾN THỨC VÀ PHƯƠNG PHÁP
- Giải phương trình: ( ) ( )
( )
( )
0
. 0
0
f x
f x g x
g x
=

= ⇔ 
=


.
- Giải phương trình: ( ) ( )
a f x a f x b
− + + =
( )
( )
( )
( ) ( )
2
2
2 2 2 2 2
.
2 2 4 2
a f x a
a f x a
b
a a f x a f x b a
≥ ≥ −

≥ ≥ −

 
⇔ =
⇔
 
 
+ − − =−
   
 
Liên hệ tài liệu word toán SĐT (zalo): 34
Website:
Ý tưởng: Bài toán xuất hiện ba căn thức, nhưng có điều đặc biệt ở đây là căn thức còn lại là tích của hai
căn thức kia. Mặt khác 2
1 , 1
x x
− − có sự đồng nhất hệ số, do đó ta sẽ nhóm hai căn này lại nên ta
được nhân tử chung như sau: ( )
2
1 1 1 1 1
x x x x
− − − = − + − . Và ta mong muốn biểu thức
3 3 1
x x
+ − + sẽ phân tích được biểu thức có chứa 1 1
x + − . Thật vậy nếu coi ( ) 3 3 1
h x x x
= + − +
là một phương trình bậc hai ẩn 1
x + ta sẽ thấy: ( ) ( )( )
1 3 1 2 1 1 1 2
h x x x x x
= + − + + = + − + − .
Chính vì thế bài toán của ta được giải quyết như sau:
2
3 1 3 1 1
x x x x
+ + − = + + −
( )( ) ( )
1 1 1 2 1 1 1 0
x x x x
⇔ + − + − + − + − =
( )( )
1 1
1 1 1 1 2 0
1 1 2
x
x x x
x x
 + =
⇔ + − + + − − = ⇔ 
+ + − =


.
Phần còn lại chỉ là việc bình phương các phương trình và tìm nghiệm như ở trên đã nêu. Ta được
nghiệm của phương trình là 0
x =
Bài toán kết thúc.
BÀI TẬP TƯƠNG TỰ
1) Giải phương trình: 2
4 4 3 2 2
x x x x
+ + − = + + −
Đáp số: 1
x =
2) Giải phương trình: 2
2 3 1 4 3 2 1 1 2
x x x x
+ + − = + + −
Đáp số: 0
x =
Bài 5:
Giải phương trình: 3 3 1 1
x x x
+ + + = −
Giải:
Phương trình tương đương với: 2 3 2 3 1 2 2
x x x
+ + + = − .
Đặt 2 2
3
2 2
3 1
u x
v u x
v x

= +

⇒ − = −

= +


với ; 0
u v > , ta được
( ) ( )( )
2 2
2 2 0
u v v u v y v u
+ = − ⇔ + − − =
2 3 1 3 2 3 1 7 4 3
v u x x x x x
⇔ = + ⇔ + = + + ⇔ + = + + +
( ) 2
3
2 6 4 3 2 3 3
4 3 6 9
x
x x x x
x x x
≥


⇔ − = + ⇔ + = − ⇔ 
+ = − +


Liên hệ tài liệu word toán SĐT (zalo): 35
Website:
2
3 5 28
5 28
10 3 0 3
x x
x
x x x
≥ 
 = ±

⇔ ⇔ ⇔ = +
 
− − = ≥

 
Nhận xét: Bài toán sử dụng phương pháp đặt ẩn phụ để đưa phương trình từ phức tạp về đơn giản hóa,
sau đó dùng phương pháp nâng lũy thừa để tìm nghiệm của phương trình.
* NHẮC LẠI KIẾN THỨC CŨ VÀ PHƯƠNG PHÁP:
- Hằng đẳng thức cơ bản: ( )( )
2 2
a b a b a b
− = − + .
- Cách giải phương trình vô tỷ dạng: ( ) ( )
f x g x m
= +
( ) ( )
( ) ( )
( )
( ) ( )
( ) ( ) ( )
2
2
; 0 ; 0
2
f x g x f x g x
f x g x m m g x
f x g x m
 ≥ ≥

 
⇔ ⇔
 
− − =
= +
 


( ) ( )
( ) ( ) ( )
1
2
2 2
2
; 0
.
4
f x g x x x
x x
f x g x m m g x
≥
 =


⇔ ⇔
  =
 
− − = 
 

Ý tưởng: Một bài toán đẹp, quan sát VT của phương trình có xuất hiện hai căn bậc hai riêng biệt đồng
thời trong căn chứa các biểu thức bậc nhất, cũng như VP của phương trình cũng là một biểu thức bậc
nhất, nên vậy ta có thể nâng lũy thừa để đưa phương trình ban đầu về phương trình bậc bốn. Nhưng nếu
tinh ý một chút, ta có ( ) ( ) ( )
3 1 3 2 2 2 1
x x x x
+ − + = − = − ,
Do vậy đặt
3
3 1
u x
v x

= +


= +


với ; 0
u v ≥ , suy ra: ( )
2 2
2 2 2 1
v u x x
− = − = − . Khi đó, phương trình đã cho
tương đương với:
( ) ( )( )
2 2
2 2 0
v u v u v u v u
− = + ⇔ + − − = (i)
- Vì ; 0
u v ≥ nên phương trình (i)
0
2
u v
v u
= =

⇔  = +

, với 0
u v
= = phương trình vô nghiệm nên ta chỉ cần
giải phương trình: 2
v u
= + .
- Với 2
v u
= + , ta có: 3 1 3 2 3 1 7 4 3
x x x x x
+ = + + ⇔ + = + + +
( ) 2
3
2 6 4 3 2 3 3
4 3 6 9
x
x x x x
x x x
≥


⇔ − = + ⇔ + = − ⇔ 
+ = − +


2
3 5 28
5 28
10 3 0 3
x x
x
x x x
≥ 
 = ±

⇔ ⇔ ⇔ = ±
 
− − = ≥

 
Bài toán kết thúc.
BẢI TẬP TƯƠNG TỰ
Liên hệ tài liệu word toán SĐT (zalo): 36
Website:
1) Giải phương trình: 4 1 2 2 2 1
x x x
+ + + = −
Đáp số: 2 5
x= +
2) Giải phương trình: 3 5 2 1 4
x x x
+ + + = −
Đáp số:
5 87
8
x
−
=
Bài 6:
Giải phương trình:
( )
2
3 7
2 1
x
x
x x
+
+ =
+
Giải:
Điều kiện 0
x >
Phương trình tương đương: ( ) 2
3
2 1 7
x x x
x
+ + = +
Chia hai vế cho 0
x ≠ , ta được:
1 3 7 3 1 3 4 3 3 2
2 1 2 1 0 2 0
x x x x x x
x x x x x x x x x x
  
     
+ + = + ⇔ + − + + + = ⇔ + − + − =
  
        
        
+ Giải: 2 1
3 3
2 4 4 3 0
3
x
x x x x
x
x x
=

+ = ⇔ + = ⇔ − + = ⇔  =

+ Giải: 2
2
3 2 3 4
3 4 0
x x x x
x x x x
+ = ⇔ + = ⇔ + − =
( )( )
2
1 4 0 1
x x x x
⇔ − + + = ⇔ =
Đáp số 1; 3
x x
= = .
Nhận xét: Bài toán sử dụng phương pháp ẩn phụ không hoàn toàn, sau đó nâng lũy thừa tìm nghiệm của
phương trình ban đầu.
* NHẮC LẠI KIẾN THỨC VÀ PHƯƠNG PHÁP
- Phương pháp đặt ẩn phụ không hoàn toàn: xét một phương trình bậc hai có dạng
( ) ( )
2
. . 0
mf x t ng x t k
+ + =(*), trong đó t là ẩn phụ được biểu diễn dưới dạng ( )
t h x
= . Khi đó, ta có:
( ) ( )
2
4
t ng x kmf x
= −
 
 
Δ , với t
Δ bắt buộc là một số chính phương. Do đó, tìm được nghiệm của (*),
đó là
Liên hệ tài liệu word toán SĐT (zalo): 37
Website:
( )
( )
( )
( )
( )
( )
;
t t
ng x ng x
t h x t h x
mf x mf x
− + − −
= = = =
Δ Δ
- Cách giải phương trình: ( ) ( )
( ) ( )
( ) ( )
2
; 0
.
f x g x
f x g x
f x g x
≥


= ⇔ 
=


.
Ý tưởng: Trước hết, ta cần quy đồng mẫu số bài toán, như vậy ta sẽ được phương trình có dạng
( ) ( ) ( )
.
f x g x h x
= và nếu nâng lũy thừa hai vế, ta sẽ thu được một phương trình bậc 5. Và phương
trình bậc 5 nếu không có nghiệm nguyên thì sẽ rất khó để giải quyết. Vậy nên ta cần nghĩ đến hướng tư
duy khác, đó là bài toán xuất hiện căn thức
3
x
x
+ nên ta mong muốn sẽ tạo được lượng
2
3
k x
x
 
+
 
 
 
để có thể đưa về phương trình bậc hai, sau đó đặt
3
t x
x
= + để sử dụng phương pháp ẩn phụ không an
toàn.
- Ta có:
( )
( )
2
2
3 7 3
2 1 7
2 1
x
x x x x
x x x
+
+ = ⇔ + + = +
+
Chia cả hai vế của phương trình cho x, ta có:
1 3 7 3 1 3 4
2 1 2 1 0
x x x x
x x x x x x x
   
+ + = + ⇔ + − + + + =
   
   
(*)
- Đặt
3
0
t x
x
= + > , khi đó ta có ( ) 2 1 4
* 2 1 0
t t
x x
 
⇔ − + + =
 
 
.
Có:
2 2
1 4 1
1 1
t
x x x
   
= + − = −
   
   
Δ nên suy ra được
( )
( )
3
1 1
2
2
1 1
2
1 1 3 2
1 1
x i
t
t
x
x x
t
t x x ii
x x x x

 + =
=
= + + −  

 
⇔ ⇔

 
=
 =+ + − +  + =


 
- Giải (i), ta có (i) 3 1
3
4 4 3 0
3
x
x x x
x
x
=

⇔ + = ⇔ − + = ⇔  =

- Giải (ii), ta có (ii) 3
2
3 4
3 4 0 1
x x x x
x x
⇔ + = ⇔ + − = ⇔ =
Bài toán kết thức.
BÀI TẬP TƯƠNG TỰ:
1) Giải phương trình: ( ) ( )( )
3 4 12 28
x x x x
+ − + = −
Liên hệ tài liệu word toán SĐT (zalo): 38
Website:
Đáp số: ( )
4 2 1 ; 31 3
x x
= − = −
2) Giải phương trình: 3 2
2 2
x x x x
− = − −
Đáp số:
1 5 1 65
; .
2 8
x x
+ −
= =
Ví dụ 6:
Giải phương trình: ( )( )
9 2012 6 2012 9 6 .
x x x x
+ + += + + +
Giải:
Điều kiện 6
x ≥ −
Phương trình đã cho tương đương với: ( )( )
9 2012 6 1 0
x x
+ − + − =
+ Giải ( )
2
9 2012 0 2012 9 4048135
x x
+ − = ⇔ = − =
+ Giải 6 1 0 5
x x
+ − = ⇔ =
−
Vậy phương trình có hai nghiệm: 4048135; 5
x x
= = −
Nhận xét: Bài toán sử dụng phương pháp nhóm nhân tử chung và nâng lũy thừa tìm nghiệm của phương
trình.
* NHẮC LẠI KIẾN THỨC VÀ PHƯƠNG PHÁP:
- Các giải phương trình dạng:
( )
( ) ( )
( )
( )
( )
( )
( )
2
2
. 0 .
f x m f x m
f x m g x n
g x n
g x n
 =  =

− − = ⇔ ⇔ 
 =

= 

Ý tưởng: Bài toán cho hết sức đơn giản, với sự xuất hiện của hai căn thức
( )( )
9; 6; 9 6
x x x x
+ + + + nên không khó để nhóm được nhân tử chung như sau:
( )( )
9 2012 6 2012 9 6
x x x x
+ + += + + +
( )( )
9 9 6 2012 6 2012 0
x x x x
⇔ + − + + + + − =
( ) ( )
9 1 6 2012 1 6 0
x x x
⇔ + − + − − + =
( )( )
1 6 9 2012 0
x x
⇔ − + + − =
Bài toán kết thúc.
BÀI TẬP TƯƠNG TỰ
Liên hệ tài liệu word toán SĐT (zalo): 39
Website:
1) Giải phương trình: 2
2 6 7 2 13 42
x x x x
+ + + = + + +
Đáp số: 3; 5
x x
=
− =
−
2) Giải phương trình: 2
4 2 3 2 7 12
x x x x
+ + + = + + +
Đáp số: 0; 2
x x
= = −
Bài 8:
Giải phương trình: 8 2 2 1
x x
+ − − =
Giải:
Điều kiện 8 2
x
− ≤ ≤
+ Nếu 1 2 8 9 3
x x
< ≤ ⇒ + > =
2 2 1 1 8 2 2 3 2 1
x x x
− < − = ⇒ + − − > − = ⇒ loại.
+ Nếu 8 1
x
− ≤ < , tương tự ta có: 8 2 2 9 2 1 1
x x
+ − − < − ==> loại
Với 1
x = , thỏa mãn phương trình.
Đáp số 1
x = .
Nhận xét: Bài toán sử dụng phương pháp nhẩm nghiệm và đánh giá theo miền nghiệm để chứng minh
nó có nghiệm duy nhất.
Ý tưởng: Đây một bài toán đơn giản nhưng đòi hỏi đi theo phương pháp đánh giá thì cần đoán trước
nghiệm của phương trình. Đầu tiên ta sẽ ưu tiên nghiệm nguyên trước, với nghiệm nguyên thì các biểu
thức chứa căn phải là một số chính phương, tức là
2
2
8
2
x k
x h
 + = ∈


− = ∈




.
Với điều kiện chặn của [ ]
8; 2
x ∈ − ] thì ta sẽ thử một vài giá trị nguyên của x và thấy rằng tại 1
x =
thỏa mãn phương trình. Công việc còn lại là ta sẽ đi chứng minh nó là nghiệm duy nhất. Nó là nghiệm
duy nhất nếu với [ 8; 1) (1; 2]
x ∈ − ∪ thì phương trình bài cho vô nghiệm. Đi xét từng trường hợp, ta có:
- Với [ 8; 1)
x ∈ − suy ra 8 2 2 9 2 1 1
x x
+ − − < − =.
- Với (1; 2]
x ∈ suy ra 8 2 2 9 2 1 1
x x
+ − − > − =.
Cả hai trường hợp trên đều chứng minh 1
x = là nghiệm duy nhất của phương trình.
Bài toán kết thúc.
Liên hệ tài liệu word toán SĐT (zalo): 40
Website:
BÀI TẬP TƯƠNG TỰ
1) Giải phương trình: 7 8 3 4
x x x
+ − = +
Đáp số: x = 1
2) Giải phương trình: 9 2 1 1
x x
+ − − =
Đáp số: x = 0
Bài 9:
Giải phương trình: ( )( )
3 1 1 1
x x x
+ − − + =
Giải:
Điều kiện 0 1
x
≤ ≤
Phương trình tương đương với:
( ) ( )
3
1 1 1 3 1 1 3
3
x x x x
x x
− + = ⇔ − + = + +
+ +
Nếu ( )
0 1 3 1 1 3
x x
≤ ≤ ⇒ − + > , đồng thời 3 1 4 3
x x
+ + < + =, suy ra VT > VP (loại).
Thử lại ta thấy 1
x = là nghiệm.
Nhận xét: Bài toán kết hợp giữa phương pháp nhân liên hợp và phương pháp đáng giá để tìm nghiệm
của phương trình.
* NHẮC LẠI KIẾN THỨC VÀ PHƯƠNG PHÁP
- Biểu thức liên hợp: ( )( )
x m x x m x x m x
+ − = + − + +
m
x m x
x m x
+ − =
+ +
với 0; 0
x x m
≥ + ≥ .
- Đánh giá: ( ) ( ) ( )
m f x n g x h x
− +
= + với
( ) 0
2
m f x
n m
≥ ≥



=


.
Ta có: ( )
( )
( ) ( )
0
2
m f x n n
f x m
g x h x m
 − + >

≤ < ⇒ 
+ <


, suy ra phương trình vô nghiệm.
Vậy x m
= là nghiệm của phương trình đã cho
Ý tưởng: Bài toán xuất hiện ba căn thức nằm trong một tích, sẽ rất khó để định hình ra hướng giải, ẩn
phụ sẽ rất phức tạp. Nhưng nếu xét hai căn thức đầu tiên ta thấy ( ) ( )
2 2
3 3
x x
− − =
. Vì thế ta sẽ nghĩ
ngay đến chuyện dùng hằng đẳng thức dạng: ( )( )
2 2
a b a b a b
− = − + . Khi đó phương trình đã cho
tương đương với:
Liên hệ tài liệu word toán SĐT (zalo): 41
Website:
( )
3
1 1 1 3 1 3 3
3
x x x x
x x
− + = ⇔ − + = + +
+ +
(i)
Với phương trình (i), ta sẽ đi nhẩm một vài giá trị nghiệm đẹp thỏa mãn các yêu cầu là 1 0
x
≥ ≥ và các
biểu thức trong căn thức là số chính phương vì thế ta khẳng định nó có nghiệm duy nhất 1
x = , đồng
thời 1 lại là miền chặn của biến do đó ta sẽ đi đánh giá phương trình (i). Tức là với 0 1
x
≤ < , ta sẽ
chứng minh (I) vô nghiệm như sau:
3 1 3 3
0 1
3 3
x
x
x x
 − + >

≤ < ⇒ 
+ + <


=> VT > VP => (i) vô nghiệm
Vậy ta kết luận 1
x = là nghiệm của phương trình đã cho.
Bài toán kết thúc.
BÀI TẬP TƯƠNG TỰ
1) Giải phương trình: ( )( )
2 1 2 1 1
x x x
+ − − − + =
Đáp số: 2
x =
2) Giải phương trình: ( )( )
1 2 3 1 1
x x x
+ − − − + =
Ví dụ 9:
Giải phương trình: ( )( )
4 2 4 2 2
x x x
+ − − + =
Giải:
Điều kiện 4 4
x
− ≤ ≤
Phương trình đã cho tương đương với: ( )
4 2 2
4 2
x
x x
x
− + =
+ +
.
+ Với 0
x = là nghiệm.
+ Giải: ( )
4 2 2 4 2
x x
− += + +
Đặt 4; 4
u x v x
= + = − ta thu được
( )
2
2 2
2 2
2 2
2 2 8 5 8 4 0
8
v u
u u u u
u v
= +

⇒ + + = ⇔ + − =

+ =

( )
2 14
; 2 96
5 5 4
5 25
2
u v
x x
u l

= =

⇔ ⇒ + = ⇔ =
−

= −


(thỏa mãn)
Vậy phương trình có hai nghiệm:
96
0;
25
x x
= = − .
Nhận xét: Sử dụng phương pháp nhân liên hợp, sau đó đặt ẩn phụ tìm nghiệm của bài toán.
Liên hệ tài liệu word toán SĐT (zalo): 42
Website:
NHẮC LẠI KIẾN THỨC VÀ PHƯƠNG PHÁP
- Hằng đẳng thức ( )( ) a b
a b a b a b a b
a b
−
− = − + ⇔ + =
−
.
- Giải phương trình tổng quát dạng ( ) ( )
f x g x m
= +
( ) ( )
( ) ( ) ( )
( ) ( )
( ) ( ) ( )
2
2
2 2
; 0
2 4
f x g x m
f x g x
f x g x m g x m f x g x m mg x
 ≥ +
≥

 
⇔ ⇔
 
= + +  
− − =
 
  

Ý tưởng: Không khó để nhận thấy, phương trình có một nghiệm là 0
x = .
Đồng thời vế trái của phương trình có xuất hiện biểu thức 4 2
x + − , dễ thấy rằng
( ) ( )( )
2
2
4 2 4 2 4 2
x x x x
= + − = + − + + . Vì thế, phương trình đã cho tương đương với:
( )( ) ( )( )
4 2 0 0
4 2 4 2 2 4 2 4 2
4 2 4 2
x x
x x x x
x x
 + − = ⇔ =
+ − − + = + − + + ⇔ 
− = + +


Phương trình còn lại có thể giải bằng cách tổng quát nêu ở trên, hoặc có thể giải quyết bằng cách đặt ẩn
phụ như sau: ( )
4
; 0
4
v x
u v
u x

= −

≥

= +


. Ta có hệ phương trình
2 2
8 2 14 2 96
; 4
5 5 5 25
2 2
v u
u v x x
v u
 + =
⇔ = = ⇒ + = ⇔ =
−

= +

.
Bài toán kết thúc.
BÀI TẬP TƯƠNG TỰ
1) Giải phương trình: ( )( )
1 1 1 1 2
x x x
+ − − + = .
Đáp số:
24
0;
25
x x
= = −
2) Giải phương trình: ( )( )
9 1 9 1 3
x x x
+ − − + =
Đáp số: 0
x =
Ví dụ 10:
Giải phương trình:
1 1
4 1 5
2 1 2
x x
x x
+ +
= +
+ +
Giải:
Liên hệ tài liệu word toán SĐT (zalo): 43
Website:
Phương trình tương đương với:
1 1
5 4 1
2 1 2
x x
x x
− = − −
+ +
.
Giả sử:
1 1
1 2 1 2
2 1 2
x x x
x x
> ⇒ + > + ⇒ <
+ +
=> 0 5 4 1 1
VT x x x
< ⇒ < − ⇒ < (mâu thuẫn).
Giả sử 0 1
x
≤ < , lập luận tương tự thu được 1
x > (mâu thuẫn). Thử lại thu được nghiệm duy nhất 1
x =
Nhận xét: Bài toán sử dụng phương pháp đánh giá miền nghiệm để tìm nghiệm của phương trình.
Ý tưởng: Quan sát bài toán, ta thấy có vẻ nó hơi rắc rối một chút vì xuất hiện tới bốn căn thức mà căn
lại còn ở dưới mẫu thì sẽ rất khó khăn trong việc giải bằng các công cụ mà ta thường làm như ẩn phụ,
nâng lũy thừa...Và hướng cuối cùng ta nghĩ đến là đánh giá. Để đánh giá được nó, ta cần tìm nghiệm
trước, vẫn là ưu tiên hướng nghiệm nguyên đồng thời cân bằng căn thức với căn thức, phân thức với
phân thức ta có:
1 1
1; 4 1 5 1
2 1 2
x x x x
x x
= ⇔ = + = ⇔ =
+ +
Và bây giờ, ta sẽ đi chứng minh 1
x = là nghiệm duy nhất của phương trình. Hay nói cách khác, với
1
x > hoặc 1
x < nó sẽ vô nghiệm. Biến đổi phương trình đã cho về dạng:
( )( )
1 1 2 2 1
4 1 5 5 4 1
2 1 2 2 1 2
x x
x x x x
x x x x
+ − +
+ += + ⇔ = − +
+ + + +
* Với 1
x > , ta có
2 2 1 2 2 1 0
5 4 1 5 4 1 0
x x x x
x x x x

+ < + + − − <
 
⇔
 
> + − + >
 

, suy ra phương trình vô nghiệm.
* Với 1 0
x
> ≥ , ta có:
2 2 1 2 2 1 0
5 4 1 5 4 1 0
x x x x
x x x x

+ > + + − + >
 
⇔
 
< + − + <
 

, suy ra phương trình vô nghiệm.
Vậy 1
x = là nghiệm duy nhất của phương trình.
Bài toán kết thúc.
BÀI TẬP TƯƠNG TỰ
1) Giải phương trình:
1 1
3 1 4
2 1 2 1
x x
x x
+ +
= +
+ +
2) Giải phương trình:
2 2
1 1
3 1 4
2 1 2
x x
x x x x
+ +
= +
+ + + +
Bài 12:
Giải phương trình: 2 3
8 3 8
x x
+= +
Liên hệ tài liệu word toán SĐT (zalo): 44
Website:
Giải:
Điều kiện 2
x ≥ −
Đặt ( )
2
2
; 0
2 4
a x
a b b
b x x

= +

≥ >

= − +


.
Ta có 2 2 2 2
2 2 4 2 4 8
a b x x x x
+ = + + − + = + , suy ra ( )( )
2 2
2 3 2 0
a b ab a b a b
+ = ⇔ − − =
+ TH1: 2 2 1
2 2 4 3 2 0
2
x
a b x x x x x
x
=

= ⇔ + = − + ⇔ − + = ⇔  =

.
+ TH2: ( ) 2 2
2 4 2 2 4 6 4 0 3 13
a b x x x x x x
= ⇔ + = − + ⇔ − − = ⇔ = ± .
Nhận xét: Bài toán sử dụng phương pháp đặt ẩn phụ rồi đưa về phương trình đẳng cấp bậc hai, tìm mối
quan hệ giữa hai ẩn phụ sau đó nâng lũy thừa tìm nghiệm của phương trình ban đầu.
NHẮC LẠI KIẾN THỨC CŨ VÀ PHƯƠNG PHÁP:
- Hằng đẳng thức: ( )( )
2 2 2 2
u v u v u uv v
+ = + − +
- Cách giải phương trình bậc hai dạng: 2 2
. . . 0
a x b xy c y
+ + = (*).
Làm nháp: Chia cả hai vế của phương trình (*) cho 2
y ( vì là làm nháp nên ta cứ coi là 0
y ≠ ), khi đó
(*)
2
. . 0
x x
a b c
y y
 
⇔ + + =
 
 
.
Coi đây là phương trình bậc hai ẩn
x
t
y
= (đã biết cách giải) thì nghiệm t tìm được sẽ biểu hiện mối
quan hệ giữa ,
x y . Cụ thể là 1
2
.
x t y
x t y
=

 =

Ý tưởng: Bài toán này có thể giải bằng cách lũy thừa hai vế, đưa về phương trình bậc bốn và vấn đề ta
gặp phải sẽ là nghiệm của phương trình bậc bốn. Ta đặt dấu hỏi cho nó? Nhưng nếu đế ý 3
8 2
= nên
trong căn thức sẽ xuất hiện ngay hằng đẳng thức ( )( )
3 3 3 2
8 2 2 2 4
x x x x x
+ = + = + − + . Vậy nên trong
căn bậc hai đã xuất hiện hai tích. Do đó để xuất hiện phương trình đẳng cấp thì ta cần làm công việc sau,
đó là đồng nhất hệ số của biểu thức dạng: ( ) ( )
2 2
1
1
2 4 2 8 2 0
2
4 2 8
a
a
a x x b x x b a
b
a b
=

=


− + + + = + ⇔ − = ⇔ 
 =

 + =

.
Đến đây, ta viết lại phương trình ban đầu như sau:
( ) ( ) ( )( )
2 3 2 2
8 3 8 2 4 2 2 3 2 4 2
x x x x x x x x
+ = + ⇔ − + + + = − + + (i)
Liên hệ tài liệu word toán SĐT (zalo): 45
Website:
* Đặt
2
2 4 0
2 0
u x x
v x
 = − + >


= + ≥


nên phương trình trên trở thành:
(i) ( )( )
2 2
3 2 0 2 0
2
u v
u uv v u v u v
u v
=

⇔ − + = ⇔ − − = ⇔  =

* Hoặc nhận thấy ( )
2
2
2 4 1 3 0
x x x
− + = + + ≠ nên phương trình (i) tương đương với:
2 2
2 2
2. 3 1 0
2 4 2 4
x x
x x x x
+ +
− + =
− + − +
. Có thể đặt căn thức là ẩn phụ để Giải phương trình: bậc hai.
* Hai hướng trên đều cho ta:
TH1. Với 2
2
2 1
2 4 2
2
3 2 0
x x
x x x
x
x x
≥ − =
 
− + = + ⇔ ⇔
 
=
− + = 

.
TH2. Với 2
2
2
2 4 2 2 3 13
6 4 0
x
x x x x
x x
≥ −

− + = + ⇔ ⇔ = ±

− − =

.
Bài toán kết thúc.
BÀI TẬP TƯƠNG TỰ
1) Giải phương trình: 2 3 2
3 5 2 6 4
x x x x x
+ −
= + − + .
Đáp số:
1 13
2
x
− +
=
2) Giải phương trình: 2 4 2
3
3 1 1
3
x x x x
− + =
− + +
Đáp số: 1
x = .
Liên hệ tài liệu word toán SĐT (zalo): 1
Website:
Chuyên đề 2. HỆ PHƯƠNG TRÌNH
I. TÓM TẮT LÝ THUYẾT
A. HỆ PHƯƠNG TRÌNH ĐỐI XỨNG LOẠI I
Hệ phương trình loại I theo ẩn x và y: Là hệ phương trình mà khi ta đổi vai trò của các ẩn x và y
thì hệ phương trình vẫn không thay đổi.
Dạng hệ phương trình
( )
( )
, 0
, 0
f x y
g x y
 =


=


với
( ) ( )
( ) ( )
, y,
, y,
f x y f x
g x y g x
 =


=


.
Đặt
S x y
P xy
 = +

=

.
Hệ phương trình ở dạng thu gọn
( )
( )
, 0
, 0
f S P
g S P
 =


=


, điều kiện 2
4
S P
≥ .
(với S là tổng hai nghiệm và P là tích hai nghiệm).
 Phương pháp giải:
Bước 1: Đặt
S x y
P xy
 = +

=

và x, y chính là nghiệm của phương trình.
Điều kiện 2
4
S P
≥ .
Bước 2: Xác định S và P.
Khi đó S và P là nghiệm của phương trình bậc hai: 2
X 0
X S P
− + =.
Bước 3: Giải phương trình: bậc hai theo ẩn X.
Bước 4: Suy ra giá trị x, y.
Ví dụ 1:
Giải hệ phương trình:
2 2
3 2 2 3
5
6
x y x y
x x y xy y
 − + − =


− − + =


.
Giải:
Ta có hệ phương trình tương đương
( ) ( )
( )( )
2 2
2 2
5
6
x y x y
x y x y
 − + − =


− − =


.
Đặt
2 2
a x y
b x y
= −


= −


.
Hệ phương trình trở thành:
5
6
a b
ab
 + =

=

.
Liên hệ tài liệu word toán SĐT (zalo): 2
Website:
Suy ra a và b là hai nghiệm của phương trình: 2 2
5X 6 0
3
X
X
X
 =
− + = ⇔ 
=

.
Với
2 2
11
2
2 2 6
3
3 7
3
3
6
x
a x y
x y
b x y
x y y

  =


= + =
− =
   
⇒ ⇔ ⇔
   
= − =


  
− = = −

 

.
Với
2 2
7
3
3 3 4
2
2 1
2
2
4
x
a x y x y
b x y
x y y
   
=
   
= − = + =
   
⇒ ⇔ ⇔
   
= − =
   
− = = −

   



.
Ví dụ 2:
Giải hệ phương trình :
2 2
4
2
x xy y
x xy y
 + + =


+ + =


.
Giải:
Đặt ( )
2
4
S x y
S P
P xy
 = +
≥

=

.
Hệ phương trình trở thành:
2
4
2
S P
S P
 − =


+ =


.
( )
2 2
2
2 2 2
2
2 4 6 0 0
3
P S
P S P S S
S
S S S S P
S
   = −
= −
 
=
− =
 
⇔ ⇔ ⇔ ⇔
 =
   
− − = + −
= =
 
   = −




hoặc
3
5
S
P
 = −

=

.
Với
2 2 2
0 0 0
S x y x
P xy y

 
= +
= =

⇒ ⇔
  
= = =

 

hoặc
0
2
x
y
 =

=

.
Với
( ) 2
3
3 3 3
3 5
5 5 3 5 0
x y
S x y x y
y y
P xy y y
   =− − 
=− + =− =− −
  
⇒ ⇔ ⇔
   
− − =
= = + +
=
 

 


2
3
3 11
0
2 4
x y
y
 =− −

⇔  
+ + =
 
 

(hệ phương trình vô nghiệm).
Vậy hệ phương trình có nghiệm: ( ) ( ) ( )
; 2;0 , 0;2
x y = .
Bài tập tương tự:
1) Giải hệ phương trình 2 2
19
84
xy x y
x y xy
 + + =

+ =

.
Đáp số: ( ) ( ) ( ) ( ) ( )
; 6 42;6 42 , 6 42;6 42 , 3;4 , 4;3
x y =
+ − − + .
Liên hệ tài liệu word toán SĐT (zalo): 3
Website:
2) Giải hệ phương trình
 + =


+ + =


2 2
6
5
x y xy
xy x y
.
Đáp số: ( ) ( ) ( )
; 1;2 , 2;1
x y = .
3) Giải hệ phương trình
2 2
2
3
y x
xy
 − =


= −


.
Đáp số: ( ) ( ) ( )
; 1; 3 , 1; 3
x y =− − .
4) Giải hệ phương trình 4 4
5
97
x y
x y
 + =

+ =

.
Đáp số: ( ) ( ) ( )
; 2;3 , 3;2
x y = .
5) Giải hệ phương trình
2 2
1
3
x xy y
x y xy
 + + =


− − =


.
Đáp số: ( ) ( )
; 1; 1
x y= − .
B. HỆ PHƯƠNG TRÌNH ĐỐI XỨNG LOẠI II
Hệ phương trình đối xứng loại II theo ẩn x và y là hệ phương trình mà khi ta đổi vai trò của x
cho y thì hai phương trình của hệ sẽ hoán đổi cho nhau. Dạng phương trình
( )
( )
, 0
; 0
f x y
f y x
 =


=


.
 Phương pháp giải:
Bước 1: Cộng hoặc trừ hai vế của hai phương trình để đưa hệ phương trình về phương trình tích và lập
hệ phương trình:
Đưa về dạng
( ) ( )
( )
, , 0
, 0
f x y f y x
f x y
 − =


=


hoặc
( ) ( )
( )
, , 0
, 0
f x y f y x
f x y
 + =


=


( ) ( ) ( )
. , 0
, 0
x y
x y f x y
f x y
 =
⇔ − = ⇔ 
=


.
Bước 2: Giải hệ phương trình vừa lập được.
Bước 3: Xét nghiệm của hệ phương trình là nghiệm của từng phương trình trong hệ ở bước 1.
Ví dụ 1:
Giải hệ phương trình:
( )
( )
2
2
1 3 1
1 3 2
x y
y x
 + =


+ =


Giải:
Lấy (1) trừ vế theo vế cho (2), ta được: ( ) ( )
2 2
1 1 3 3
x y y x
+ − + = −
Liên hệ tài liệu word toán SĐT (zalo): 4
Website:
( ) ( )( ) ( )
2 2
3 3 0
x y y x x y x y x y
⇔ − = − ⇔ − + + − =
( )( )
3 0
3 0 3
x y x y
x y x y
x y x y
 
= =
⇔ − + + = ⇔ ⇔
 
+ + = =
− −
 

.
Với x y
= thế vào (1), ta được: 2
1 3
y y
+ =
2
3 5 3 5
2 2
3 1 0
3 5 3 5
2 2
y x
y y
y x
 + +
= ⇒
=


⇔ − + = ⇔
 − −
= ⇒
=


.
Với 3
x y
=− − thế vào (2), ta được ( )
2
1 3 3
y y
+ = − −
2
2 3 31
3 10 0 0
2 4
y y y
 
⇔ + + = ⇔ + + =
 
 
(vô nghiệm).
Vậy hệ phương trình có nghiệm:
( ) 3 5 3 5 3 5 3 5
, , , ,
2 2 2 2
x y
   
+ + − −
=    
   
   
.
Ví dụ 2:
Giải hệ phương trình:
( )
( )
2 2
2 2
2 2 1
2 2 2
x y x y
y x y x
 − = +


− = +


Giải:
Lấy (1) trừ vế theo vế cho (2), ta được: ( ) ( ) ( ) ( )
2 2 2 2
2 2 2 2
x y y x x y y x
− − − = + − +
( ) ( )( ) ( )
2 2
3 3
3 1
x y
x y x y x y x y x y
x y
 =
⇔ − = − ⇔ − + = − ⇔ 
+ =


.
Với x y
= thế vào (1) ta được: 2 0 0
3x
3 3
x y
x
x y
 = ⇒ =
− = ⇔ 
=
− ⇒ =
−

.
Với ( ) 1 3
3 1
3
y
x y x
−
+ = ⇔ = thế vào (2), ta được
2
2 1 3 1 3
2 2
3 3
y y
y y
 
− −
− =
+
 
 
2
2 1 5 1 19
0 0
3 9 6 36
y y y
 
⇔ − + = ⇔ − + =
 
 
(vô nghiệm).
Vậy hệ phương trình có nghiệm: ( ) ( ) ( )
; 0;0 , 3; 3
x y
= − − .
Bài tập tương tự:
1) Giải hệ phương trình
3
3
2
2
x x y
y y x
 = +


= +


Liên hệ tài liệu word toán SĐT (zalo): 5
Website:
Đáp số: ( ) ( ) ( ) ( )
; 0;0 , 3; 3 , 3; 3
x y
= − −
2) Giải hệ phương trình
2 2
2 2
2
2
x y y
xy x
 + =


+ =


.
Đáp số: ( ) ( )
; 1; 1
x y = − −
3) Giải hệ phương trình
 + + =


+ + =


2
2
1
1
x xy y
x xy y
.
Đáp số: ( ) ( ) ( )
1 1
; 1; 1 , ; , ;2
2 2
x y t t
 
= − −  
 
với t R+
∈ .
C. HỆ PHƯƠNG TRÌNH ĐẲNG CẤP BẬC HAI
Dạng hệ phương trình
( )
( )
2 2
2 2
0 1
0 2
ax by cxy d
a x b y c xy d
 + + + =


′ ′ ′ ′
+ + + =


.
 Phương pháp giải:
Bước 1: Xét ( )
0 0
x y
= = , thế vào cả hai phương trình để tìm nghiệm (nếu có).
Bước 2: Xét ( )
0 0
x y
≠ ≠ .
Đặt ( )
x ty y tx
= = .
Ta có hệ phương trình tương đương
( )
( )
2 2 2 2
2 2 2 2
3
4
at y by cty d
a t y b y c ty d
 + + =
−


′ ′ ′ ′
+ + =
−


.
Bước 3: Chia (3) cho (4), ta được:
2
2
at b ct d
d
a t b c t
+ +
=
′
′ ′ ′
+ +
.
Ta đưa về phương trình bậc nhất hai ẩn t rồi tiến hành giải.
Ví dụ 1:
Giải hệ phương trình
2 2
2 2
2 3 13
4 2 6
x xy y
x xy y
 − + =


+ − =
−


.
Giải:
Xét 0
x = , ta có hệ phương trình
2
2
3 13
2 6
y
y
 =


− =
−


(vô nghiệm).
Xét 0
x ≠ , đặt y tx
= .
Hệ phương trình trở thành:
( )
( )
2 2
2 2 2 2
2 2 2 2 2 2
2 3 13 (1)
2 3 13
4 2 6 1 4 2 6 (2)
x t t
x tx t x
x tx t x x t t
  − + =
− + =
 
⇔
 
+ − =
− + − =
−
 


.
Liên hệ tài liệu word toán SĐT (zalo): 6
Website:
Chia (1) cho (2) theo vế, ta được:
2
2
2 3 13
6
1 4 2
t t
t t
− +
=
−
+ −
2
25
4
8 46 25 0
1
2
t
t t
t

=

⇔ − − = ⇔ 
 = −


.
Với
25
4
t = thế vào (1), ta được:
2
2 2
25 25 1807
2 3. 13 13
4 4 16
x x
 
 
 
− + = ⇔ =
 
 
 
 
2
4 25
16 139 139
139 4 25
139 139
x y
x
x y

= ⇒
=


⇔ = ⇔

=
− ⇒ =
−


.
Với
1
2
t = − thế vào (1), ta được:.
2
2 2
1 1 13
2 3. 13 13
2 2 4
x x
 
   
 
− − + − = ⇔ =
   
 
   
 
2 2 1
4
2 1
x y
x
x y
 = ⇒ =
−
⇔ = ⇔ 
=
− ⇒ =

.
Vậy hệ phương trình có nghiệm:
( ) ( ) ( )
4 25 4 25
; ; , ; , 2; 1 , 2;1
139 139 139 139
x y
   
= − − − −
   
   
.
Ví dụ 2:
Giải hệ phương trình:
2 2
2 2
3 1
3 3 13
x xy y
x xy y
 − + =
−


− + =


Giải:
Xét 0
x = , ta được hệ phương trình
2
2
1
3 13
y
y
 = −


=


(vô nghiệm).
Xét 0
x ≠ , đặt y tx
= ta được hệ phương trình
( )
( )
2 2 2 2
2 2 2 2
3 1 1
3 3 13 2
x tx t x
x tx t x
 − + =
−


− + =


.
Chia (1) cho (2) theo vế, ta được
2
2
1 3 1
13
3 3
t t
t t
− +
= −
− +
.
2
2
2 5 2 0 1
2
t
t t
t
 =

⇔ − + = ⇔
 =


.
Liên hệ tài liệu word toán SĐT (zalo): 7
Website:
Với 2
t = thế vào (1), ta được: 2 1 2
1
1 2
x y
x
x y
 = ⇒ =
− =− ⇔ 
=− ⇒ =−

.
Với
1
2
t = thế vào (1), ta được: 2 2 1
1
1
4 2 1
x y
x
x y
 = ⇒ =
− =− ⇔ 
=
− ⇒ =
−

.
Vậy hệ phương trình có nghiệm: ( ) ( ) ( ) ( ) ( )
; 1;2 , 1; 2 , 2;1 , 2; 1
x y
= − − − − .
Bài tập tương tự:
1) Giải hệ phương trình
2 2
2 2
3 2 2 7
6 3 8
x xy y
x xy y
 − + =


+ − =
−


.
Đáp số: ( ) ( )
; 1;1
x y = − , ( )
1; 1
− ,
5 31
;
241 241
 
 
 
,
5 31
;
241 241
 
− −
 
 
.
2) Giải hệ phương trình:
2 2
2 2
3 5 4 3
9 11 8 6
x xy y
x xy y
 − − =
−


+ − =


.
3) Giải hệ phương trình:
2 2
2 2
3 2 11
2 3 17
x xy y
x xy y
 + + =


+ + =


.
4) Giải hệ phương trình:
2 2
2 2
3 8 4 0
5 7 6 0
x xy y
x xy y
 − + =


− − =


.
5) Giải hệ phương trình:
2
2 2
3 2 160
3 2 8
x xy
x xy y
 − =


− − =


.
II. BÀI TẬP
Bài 1:
Giải hệ phương trình:
1 1 9
2
1 3 1 1
4 2
x y
x y
x xy
y xy

+ + + =



 
 + + = +
 
  

Giải:
Hệ phương trình tương đương với:
1 1 9
2
1 3 1 1 1
2
4 2
x y
y x
x x y
y y x
    
+ + + =
    
    

    
 + + = + + −
    
     

Đặt
1
1
u x
y
v y
x

= +



 = +


Liên hệ tài liệu word toán SĐT (zalo): 8
Website:
Hệ phương trình trở thành:
9
9
2
2
9 3 9
1 3
2
4 2 2
4 2
v u
u v
u
u u
u uv
 
= −
+ =
 
 
⇔
 
 
  + = −
+ = −  
   


Suy ra: 2 2
9 3 9 9
3 0
4 2 2 4
u u
u u u
+ = − ⇔ − + = .
2
1 3
3
3 2
0 2
2 1
3 3
x
u y
u
v y
x
 
+ =
 
  =
 
⇔ − = ⇒ ⇔
 
 
   
= + =
 


2
3 1
1 1
3 2
3 3 3 2 0
2 2
2 2
1 3 2 1
y
xy y x
y y
x x y y y
y
xy x y x
 
+ = = ⇒ =
 
⇔ ⇒ = ⇒ = ⇒ + = ⇔ − + = ⇒
 
 + = = ⇒ =



Hệ phương trình có nghiệm: ( ) ( )
1
; ;1 , 1;2
2
x y
 
=  
 
.
Nhận xét: Bài toán sử dụng phương pháp đặt hai ẩn phụ, đưa về hệ phương trình bậc hai cơ bản giải
bằng phương pháp thế. Sau đó từ nghiệm ẩn phụ suy ngược lại nghiệm của hệ phương trình.
Ý tưởng: Hình thức bài toán khá phức tạp vì sự xuất hiện của phân thức, quan sát ta thấy ở hai phương
trình của hệ đều xuất hiện biểu thức
1
x
y
+ . Ta sẽ nghĩ đến chuyện thế
1 9 1
2
x y
y x
 
+ = − +
 
 
xuống
phương trình hai nhưng còn đại lượng
1
xy
xy
+ chưa biết xử lý như thế nào. Có lẽ tác giả đã gợi mở theo
con đường đặt ẩn phụ, nếu đặt
1
u x
y
= + ;
1
v y
x
= + thì bây giờ ta chỉ cần biểu diễn
1
xy
xy
+ qua u, v thì
hệ phương trình đã cho sẽ được giải quyết.
Ta có
1
1
1
1
u x uy xy
y
v y vx xy
x

= + ⇔ = +



 = + ⇔ = +


.
( )
2 1
1 2
uvxy xy uv xy
xy
⇒ = + ⇔ = + + .
Khi đó, hệ phương trình đã cho tương đương với:
9
2
1 3
2
4 2
u v
u uv

+ =



 + = −


Hệ phương trình trên là hệ phương trình cơ bản, hoàn toàn giải quyết được bằng phương pháp thế.
Liên hệ tài liệu word toán SĐT (zalo): 9
Website:
Bài toán kết thúc.
Bài tập tương tự:
1) Giải hệ phương trình
( )
2 2
2 2 2
1 2
1 3
x y
xy x y x
 + =


+ + =


.
Đáp số: ( ) ( ) ( ) 7 5 7 5
; 1;1 , 1; 1 , ; , ;
4 4
7 7
x y
   
= − − − −
   
   
   
.
2) Giải hệ phương trình
( ) ( )
( )( )
2 2
2 2
1 1 27
1 1 10
x y xy
x y xy
 + + =


+ + =


.
Đáp số: ( ) ( )
1 1
; ;2 3 , 2;2 3 , 2 3;
2 2
x y
   
= ± ± −
   
   
.
Bài 2:
Giải hệ phương trình:
2 2
2 2
1
2 4
x xy y
x xy y
 − + =


+ + =


Giải:
Giải hệ phương trình:
2 2
2 2
1
2 4
x xy y
x xy y
 − + =


+ + =


.
Thế 2 2
1 x xy y
= − + xuống vế phải phương trình thứ hai của hệ phương trình, ta có:
( )
2 2 2 2
2 4
x xy y x xy y
+ + = − +
( )( )
2 2
3 5 2 0 3 2 0 .
3 2
x y
x xy y x y x y
x y
 =
⇔ − + = ⇔ − − = ⇔ 
=

Với y x
= , thế vào phương trình thứ nhất trong hệ, ta được 2 1 1
1
1 1
x y
x
x y
 = ⇒ =
= ⇔ 
=− ⇒ =−

.
Với
3
3 2
2
x
x y y
= ⇔ = , thế vào phương trình thứ nhất trong hệ ta được:
2
2 3
7 7 7
1
4 2 3
7 7
x y
x
x y

= ⇒ =


= ⇔

=
− ⇒ =
−


.
Vậy hệ phương trình đã cho có bốn nghiệm kể trên.
Nhận xét: Bài toán sử dụng phương pháp thế và tách ghép phương trình đẳng cấp bậc hai tìm nhân tử
chung.
Nhắc lại kiến thức và phương pháp:
Liên hệ tài liệu word toán SĐT (zalo): 10
Website:
• Phương trình đẳng cấp bậc hai có dạng: 2 2
. . . 0
a x b xy c y
+ + =
.
• Nhóm nhân tử chung, đưa về dạng:
( )( )
2 2
. . . 0 . . 0
x my
a x b xy c y x m y x n y
x ny
 =
+ + =⇔ − − =⇔ 
=

.
Hai trường hợp này, thế vào một trong hai phương trình còn lại của hệ, sẽ được phương trình có dạng
2
2
x t x t
y u y u
 =⇔ =
±

 = ⇔ =
±

Chú ý: Nếu , 0
t u < thì phương trình sẽ vô nghiệm.
Ý tưởng: Nhận thấy ở vế trái của mỗi phương trình đều có dạng của phương trình đẳng cấp bậc hai
2 2
. . .
a x b xy c y
+ + , nếu thế một trong hai phương trình còn lại ta cũng sẽ thu được một phương trình bậc
đẳng cấp bậc hai bằng 0. Từ đó tìm mối liên hệ giữa x, y. Thế ngược lại phương trình thứ nhất của hệ.
Tìm nghiệm.
• Ở vế phải phương trình thứ hai có 4 4.1
= mà từ phương trình một 2 2
1 x xy y
= − + . Vậy nên phương
trình hai trở thành:
( )
2 2 2 2
2 4
x xy y x xy y
+ + = − +
( )( )
2 2
3 5 2 0 3 2 0
x xy y x y x y
⇔ − + = ⇔ − − = .
• Có thể dễ dàng giải phương trình theo cách đồng nhất hệ số ở bước làm ra nháp như sau:
( )( ) ( )
2 2 2 2
3 5 2 3 3
x xy y x my x ny x mxy nxy mny
− + = − − = − − +
Đồng nhất hệ số hai phương trình, ta có
3 3 5
3 2
m n
mn
 + =

=

Đây là phương trình hai ẩn, đã biết cách giải theo Vi-ét, tìm được
2
1;
3
m n
= = .
Bài toán kết thúc
Bài tập tương tự:
1) Giải hệ phương trình
2 2
2 2
7
3 5 5
x xy y
x xy y
 + + =


− − =


.
Đáp số: ( ) ( ) ( ) 5 4 5 4
; 2;1 , 2; 1 , ; , ;
3 3 3 3
x y
   
= − − − −
   
   
2) Giải hệ phương trình
 + + =


− + =


3 2 3
2 2 3
3
3 3
x x y y
x xy y
.
Đáp số: ( ) ( ) ( ) ( )
3
; 1;1 , 2; 1 , 3;0
x y
= − .
Liên hệ tài liệu word toán SĐT (zalo): 11
Website:
Bài 3:
Giải hệ phương trình
2 2
2 2
1
2 3 2
x xy y
x xy y x y
 − + =


+ − − − =
−


.
Giải
Cộng từng vế hai phương trình ta có: 2
2 3 1
x xy x y
+ − − =
−
( ) ( )
2 2 1 0
x x y x x y
⇔ + − − + + =
( )( )
1 2 1 0
x x y
⇔ − + − =
TH1: 2
1 0 0
x y y y
= ⇒ − = ⇒ = hoặc 1
y = (thỏa mãn)
TH2: 2 1 1 2
x y y x
+ = ⇒ = − suy ra
( ) ( )
2
2 2
0 1
1 2 1 2 1 7 5 0 5 3
7 7
x y
x x x x x x
x y
 = ⇒ =

− − + − = ⇔ − = ⇔
 = ⇒ =
−


.
Đáp số: ( ) ( ) ( ) ( ) 5 5
; 1;0 , 1;1 , 0;1 , ;
7 7
x y
 
= −
 
 
.
Nhận xét: Bài toán sử dụng phương pháp hằng số biến thiên tìm ra được một phương trình biểu diễn
mối liên hệ giữa hai biến và từ đó thế ngược lại một trong hai phương trình, tìm nghiệm của hệ.
Ý tưởng: Đây là hệ phương trình bậc hai, trước hết ta sẽ đi tìm nhân tử ở từng phương trình trong hệ,
nếu công việc này thất bại. Ta sẽ nghĩ đến việc kết hợp cả hai phương trình. Và điều tối ưu ta nghĩ tới sẽ
là xét đenta theo ẩn x hoặc y từng phương trình (bạn đọc tự làm) khi đó không tìm được nhân tử x; y.
Chính vì thế, còn hướng duy nhất đó là kết hợp hai phương trình của hệ, giả sử tồn tại k R
∈ thỏa mãn
phương trình:
( )
− + − + + − − − + =
2 2 2 2
1 2 3 2 0
k x xy y x xy y x y
( ) ( ) ( ) ( )
2 2
1 2 3 1 2 0
k x k y x k y y k i
 
⇔ + + − − + − − + − =
  .
Và ta coi (i) là phương trình bậc hai ẩn x đồng thời khi xét x
∆ nó phải là một số chính phương. Ta có:
( ) ( ) ( )
2
2
2 3 4 1 1 2
x
k y k k y y k
   
∆ = − − − + − − + −
   
( ) ( )
2 2 2
5 4 10 8 4 4 1
k k y k y k k
= − − − + − + .
Để x
∆ là số chính phương khi hệ số 2
y phải là một số chính phương, tức là ta đi giải phương trình:
nghiệm nguyên 2 2
5 4
k k m
− =. Không khó để ta thấy rằng 1 1
k m
= ⇒ = thỏa mãn. Hay nói cách khác:
( )
2
1
x
y
∆ = − . Khi đó phương trình (i):
Liên hệ tài liệu word toán SĐT (zalo): 12
Website:
( )
2
3 1
1
2
3 1 0
3 1
2
2
y y
x
x y x y
y y
x y
 − + −
= =

⇔ + − + − = ⇔ 
− − +
 = = −


Việc còn lại là thế 1
x = hoặc 2
x y
+ = vào phương trình một để tìm nghiệm của hệ phương trình
Bài toán kết thúc.
Bài tập tương tự:
1) Giải hệ phương trình
3 3 2
2 2
3 9
4
x y y
x y x y
 − = +


+ = −


.
Đáp số: ( ) 1 13 5 13
; ;
2 2
x y
 
± − ±
=  
 
 
.
2) Giải hệ phương trình
2 2
2
2 2 3 0
3 1 0
x xy y x
xy y y
 + + + =


+ + + =


.
Đáp số: ( ) ( ) 1 5
; 3 2 2;1 2 , 3 5;
2
x y
 
±
= − ± − ±
 
 
 
 .
Bài 4:
Giải hệ phương trình:
( )( )
2 2
3
2
2 2
x y
x x y xy
 + =


= + −


Giải
Từ hệ phương trình đã cho ta có ( )( )
3 2 2 3 3
2x x y x y xy x y x y
= + + − = + ⇔ = .
Do đó hệ đã cho tương đương với 2 2
1
2
x y
x y
x y
 =
⇔ = =
±

+ =

.
Vậy hệ phương trình có nghiệm ( ) ( ) ( )
; 1;1 , 1; 1
x y
= − − .
Nhận xét: Bài toán sử dụng phương pháp thế để đưa về phương trình đẳng cấp có mối liên hệ giữa các
biến sau đó thế ngược lại một trong hai phương trình của hệ ban đầu để tìm nghiệm.
Nhắc lại kiến thức và phương pháp:
• Hằng đẳng thức dạng:
( )( )
3 3 2 2
a b a b a ab b
+ = + − + và ( )( )
3 3 2 2
a b a b a ab b
− = − + + .
• Phương trình dạng: ( )( )
3 3 3 3 2 2
0 0
x y x y x y x xy y
= ⇔ − = ⇔ − + + = .
Vì
2 2
2 2 3
0
2 4
y y
x xy y x
 
+ + = + + >
 
 
.
Liên hệ tài liệu word toán SĐT (zalo): 13
Website:
Ý tưởng: Cả hai phương trình đều xuất hiện hằng số 2. Đồng thời ở phương trình thứ hai có dáng dấp là
một phương trình đẳng cấp bậc ba vì vế trái chứa 3
2x còn vế phải chứa tích của một hàm số bậc nhất và
một hàm số bậc hai vì vậy ta sẽ thực hiện phép thế 2 2
2 x y
= + vào vế phải của phương trình hai, ta sẽ
có: ( )( )
3 2 2
2x x y x y xy
= + + − (*). Với phương trình này (*) nếu bạn nào tinh ý sẽ phát hiện được hằng
đẳng thức ( )( )
+ = + − +
3 3 2 2
x y x y x xy y . Vì thế (*) ⇔ = + ⇔ =
3 3 3
2x x y x y . Không thì khai triển tích
ra ta cũng sẽ có được điều đó. Việc còn lại chỉ là thế x y
= ngược lại phương trình thứ nhất trong hệ và
tìm nghiệm.
Bài toán kết thúc.
Bài tập tương tự:
1) Giải hệ phương trình
( )( )
2
3 4
2 2
x y
x x y x
 + =


= + −


.
Đáp số: ( ) ( ) 5
; 1;1 , 1;
3
x y
 
= −
 
 
.
2) Giải hệ phương trình
( )( )
2 2
3
3 4
2 2
x y
x x y xy
 + =


= + −


.
Đáp số: ( ) ( ) ( )
; 1;1 , 1; 1
x y
= − − .
Bài 5:
Giải hệ phương trình
3 3
1
7 7
x y y x xy
xy y x
 + = + − +


+ − =


Giải
Cộng hai phương trình của hệ ta thu được
( ) ( )
3
3 3 3 3
6 8 2 3 2 0
x y xy x y xy
+ + = ⇔ + + − − − =
( )( )
2 2
2 4 2 2 0
x y x y xy y x
⇔ + − + + − + + =
Ta luôn có: ( ) ( ) ( )
2
2 2
2 2 2
x y xy y x
+ + − ≥ + − + − đẳng thức xảy ra khi và chỉ khi 2
x y
= = − .
Vậy nếu 2 2
4 2 2 0
x y xy y x
+ + − + + = ta suy ra 2
x y
= = − (loại) vì không thỏa mãn phương trình
7 7
xy y x
+ − =.
Vậy thu được hệ
2 2
7 7
x y y x
xy y x
 + = ⇒ = −

+ − =

.
Liên hệ tài liệu word toán SĐT (zalo): 14
Website:
Suy ra ( ) 2
1 1
7 2 2 2 7 7 12 5 0 5 9
7 7
x y
x x x x x
x y
 = ⇒ =

− + − = ⇔ − + = ⇔
 = ⇒ =


.
Nhận xét: Bài toán sử dụng phương pháp thế (hay cộng vế) để ra được phương trình có mối liên hệ giữa
các biến. Sau đó thế ngược lại tìm nghiệm của hệ phương trình.
Nhắc lại kiến thức và phương pháp:
• Tổng các đại lượng không âm:
( ) ( ) ( )
2 2 2 2 2 2
0
a b b c a c a b c ab bc ca
− + − + − ≥ ⇔ + + ≥ + + .
• Đẳng thức:
( )( )
3 3 3 2 2 2
3
a b c abc a b c a b c ab bc ca
+ + − = + + + + − − − .
Ý tưởng: Cả hai phương trình của hệ, đều xuất hiện nhân tử x y
− vì thế ta sẽ nghĩ đến chuyện thế x y
−
từ phương trình một vào phương trình hai (hoặc ngược lại), do đó ta có được 3 3
6 8 0
x y xy
+ + − = (i).
Đến đấy ta mong muốn sẽ biểu diễn mối quan hệ giữa x, y, quan sát phương trình (i), ta thấy rằng
( )
3
2 8
− =
− và ( )
6 3 2
xy xy
= − do đó nếu đặt 2
z = − thì (i) 3 3 3
3 0
x y z xyz
⇔ + + − =
. Một biểu thức đối
xứng rất đẹp, bằng cách nhóm nhân tử, ta có:
( ) ( )
3
3 3 3 3
3 0 3 0
x y z xyz x y z xy x y z
+ + − = ⇔ + + − + + = .
( )( ) ( )
2 2 2
2 3 0
x y z x xy y xz yz z xy x y z
⇔ + + + + − − + − + + =.
( )( )
2 2 2
0 (*)
x y z x y z xy yz xz
⇔ + + + + − − − = .
Dễ thấy ( ) ( ) ( )
2 2 2
2 2 2 1
2
x y z xy yz xz x y y z z x
 
+ + − − − = − + − + −
 
 
do đó, phương trình (*)
 + + =
⇔ 
= =

0
x y z
x y z
. Công việc còn lại là thay 2
z = − suy ra
2
2
x y
x y
 + =

= = −

. Nhưng 2
x y
= = − loại vì không
thỏa mãn phương trình hai trong hệ. Với 2
x y
+ = thay xuống phương trình hai, ta tìm được nghiệm của
hệ phương trình là ( ) ( ) 5 9
; 1;1 , ;
7 7
x y
 
=  
 
.
Bài toán kết thúc.
Bài tập tương tự:
1) Giải hệ phương trình
3 3
2 2
1 3
2 3 0
x y xy
x y x y
 + + =


+ − + − =


.
Đáp số: ( ) ( ) 1 33 5 33 1 33 5 33
; 1;1 , ; , ;
4 4 4 4
x y
   
− − + + − −
=    
   
   
.
CHINH PHỤC KÌ THI VÀO LỚP 10 THPT MÔN TOÁN NĂM 2024 CÁC CHUYÊN ĐỀ HAY VÀ KHÓ, 20 ĐỀ LUYỆN TẬP CÓ HƯỚNG DẪN CHI TIẾT.pdf
CHINH PHỤC KÌ THI VÀO LỚP 10 THPT MÔN TOÁN NĂM 2024 CÁC CHUYÊN ĐỀ HAY VÀ KHÓ, 20 ĐỀ LUYỆN TẬP CÓ HƯỚNG DẪN CHI TIẾT.pdf
CHINH PHỤC KÌ THI VÀO LỚP 10 THPT MÔN TOÁN NĂM 2024 CÁC CHUYÊN ĐỀ HAY VÀ KHÓ, 20 ĐỀ LUYỆN TẬP CÓ HƯỚNG DẪN CHI TIẾT.pdf
CHINH PHỤC KÌ THI VÀO LỚP 10 THPT MÔN TOÁN NĂM 2024 CÁC CHUYÊN ĐỀ HAY VÀ KHÓ, 20 ĐỀ LUYỆN TẬP CÓ HƯỚNG DẪN CHI TIẾT.pdf
CHINH PHỤC KÌ THI VÀO LỚP 10 THPT MÔN TOÁN NĂM 2024 CÁC CHUYÊN ĐỀ HAY VÀ KHÓ, 20 ĐỀ LUYỆN TẬP CÓ HƯỚNG DẪN CHI TIẾT.pdf
CHINH PHỤC KÌ THI VÀO LỚP 10 THPT MÔN TOÁN NĂM 2024 CÁC CHUYÊN ĐỀ HAY VÀ KHÓ, 20 ĐỀ LUYỆN TẬP CÓ HƯỚNG DẪN CHI TIẾT.pdf
CHINH PHỤC KÌ THI VÀO LỚP 10 THPT MÔN TOÁN NĂM 2024 CÁC CHUYÊN ĐỀ HAY VÀ KHÓ, 20 ĐỀ LUYỆN TẬP CÓ HƯỚNG DẪN CHI TIẾT.pdf
CHINH PHỤC KÌ THI VÀO LỚP 10 THPT MÔN TOÁN NĂM 2024 CÁC CHUYÊN ĐỀ HAY VÀ KHÓ, 20 ĐỀ LUYỆN TẬP CÓ HƯỚNG DẪN CHI TIẾT.pdf
CHINH PHỤC KÌ THI VÀO LỚP 10 THPT MÔN TOÁN NĂM 2024 CÁC CHUYÊN ĐỀ HAY VÀ KHÓ, 20 ĐỀ LUYỆN TẬP CÓ HƯỚNG DẪN CHI TIẾT.pdf
CHINH PHỤC KÌ THI VÀO LỚP 10 THPT MÔN TOÁN NĂM 2024 CÁC CHUYÊN ĐỀ HAY VÀ KHÓ, 20 ĐỀ LUYỆN TẬP CÓ HƯỚNG DẪN CHI TIẾT.pdf
CHINH PHỤC KÌ THI VÀO LỚP 10 THPT MÔN TOÁN NĂM 2024 CÁC CHUYÊN ĐỀ HAY VÀ KHÓ, 20 ĐỀ LUYỆN TẬP CÓ HƯỚNG DẪN CHI TIẾT.pdf
CHINH PHỤC KÌ THI VÀO LỚP 10 THPT MÔN TOÁN NĂM 2024 CÁC CHUYÊN ĐỀ HAY VÀ KHÓ, 20 ĐỀ LUYỆN TẬP CÓ HƯỚNG DẪN CHI TIẾT.pdf
CHINH PHỤC KÌ THI VÀO LỚP 10 THPT MÔN TOÁN NĂM 2024 CÁC CHUYÊN ĐỀ HAY VÀ KHÓ, 20 ĐỀ LUYỆN TẬP CÓ HƯỚNG DẪN CHI TIẾT.pdf
CHINH PHỤC KÌ THI VÀO LỚP 10 THPT MÔN TOÁN NĂM 2024 CÁC CHUYÊN ĐỀ HAY VÀ KHÓ, 20 ĐỀ LUYỆN TẬP CÓ HƯỚNG DẪN CHI TIẾT.pdf
CHINH PHỤC KÌ THI VÀO LỚP 10 THPT MÔN TOÁN NĂM 2024 CÁC CHUYÊN ĐỀ HAY VÀ KHÓ, 20 ĐỀ LUYỆN TẬP CÓ HƯỚNG DẪN CHI TIẾT.pdf
CHINH PHỤC KÌ THI VÀO LỚP 10 THPT MÔN TOÁN NĂM 2024 CÁC CHUYÊN ĐỀ HAY VÀ KHÓ, 20 ĐỀ LUYỆN TẬP CÓ HƯỚNG DẪN CHI TIẾT.pdf
CHINH PHỤC KÌ THI VÀO LỚP 10 THPT MÔN TOÁN NĂM 2024 CÁC CHUYÊN ĐỀ HAY VÀ KHÓ, 20 ĐỀ LUYỆN TẬP CÓ HƯỚNG DẪN CHI TIẾT.pdf
CHINH PHỤC KÌ THI VÀO LỚP 10 THPT MÔN TOÁN NĂM 2024 CÁC CHUYÊN ĐỀ HAY VÀ KHÓ, 20 ĐỀ LUYỆN TẬP CÓ HƯỚNG DẪN CHI TIẾT.pdf
CHINH PHỤC KÌ THI VÀO LỚP 10 THPT MÔN TOÁN NĂM 2024 CÁC CHUYÊN ĐỀ HAY VÀ KHÓ, 20 ĐỀ LUYỆN TẬP CÓ HƯỚNG DẪN CHI TIẾT.pdf
CHINH PHỤC KÌ THI VÀO LỚP 10 THPT MÔN TOÁN NĂM 2024 CÁC CHUYÊN ĐỀ HAY VÀ KHÓ, 20 ĐỀ LUYỆN TẬP CÓ HƯỚNG DẪN CHI TIẾT.pdf
CHINH PHỤC KÌ THI VÀO LỚP 10 THPT MÔN TOÁN NĂM 2024 CÁC CHUYÊN ĐỀ HAY VÀ KHÓ, 20 ĐỀ LUYỆN TẬP CÓ HƯỚNG DẪN CHI TIẾT.pdf
CHINH PHỤC KÌ THI VÀO LỚP 10 THPT MÔN TOÁN NĂM 2024 CÁC CHUYÊN ĐỀ HAY VÀ KHÓ, 20 ĐỀ LUYỆN TẬP CÓ HƯỚNG DẪN CHI TIẾT.pdf
CHINH PHỤC KÌ THI VÀO LỚP 10 THPT MÔN TOÁN NĂM 2024 CÁC CHUYÊN ĐỀ HAY VÀ KHÓ, 20 ĐỀ LUYỆN TẬP CÓ HƯỚNG DẪN CHI TIẾT.pdf
CHINH PHỤC KÌ THI VÀO LỚP 10 THPT MÔN TOÁN NĂM 2024 CÁC CHUYÊN ĐỀ HAY VÀ KHÓ, 20 ĐỀ LUYỆN TẬP CÓ HƯỚNG DẪN CHI TIẾT.pdf
CHINH PHỤC KÌ THI VÀO LỚP 10 THPT MÔN TOÁN NĂM 2024 CÁC CHUYÊN ĐỀ HAY VÀ KHÓ, 20 ĐỀ LUYỆN TẬP CÓ HƯỚNG DẪN CHI TIẾT.pdf
CHINH PHỤC KÌ THI VÀO LỚP 10 THPT MÔN TOÁN NĂM 2024 CÁC CHUYÊN ĐỀ HAY VÀ KHÓ, 20 ĐỀ LUYỆN TẬP CÓ HƯỚNG DẪN CHI TIẾT.pdf
CHINH PHỤC KÌ THI VÀO LỚP 10 THPT MÔN TOÁN NĂM 2024 CÁC CHUYÊN ĐỀ HAY VÀ KHÓ, 20 ĐỀ LUYỆN TẬP CÓ HƯỚNG DẪN CHI TIẾT.pdf
CHINH PHỤC KÌ THI VÀO LỚP 10 THPT MÔN TOÁN NĂM 2024 CÁC CHUYÊN ĐỀ HAY VÀ KHÓ, 20 ĐỀ LUYỆN TẬP CÓ HƯỚNG DẪN CHI TIẾT.pdf
CHINH PHỤC KÌ THI VÀO LỚP 10 THPT MÔN TOÁN NĂM 2024 CÁC CHUYÊN ĐỀ HAY VÀ KHÓ, 20 ĐỀ LUYỆN TẬP CÓ HƯỚNG DẪN CHI TIẾT.pdf
CHINH PHỤC KÌ THI VÀO LỚP 10 THPT MÔN TOÁN NĂM 2024 CÁC CHUYÊN ĐỀ HAY VÀ KHÓ, 20 ĐỀ LUYỆN TẬP CÓ HƯỚNG DẪN CHI TIẾT.pdf
CHINH PHỤC KÌ THI VÀO LỚP 10 THPT MÔN TOÁN NĂM 2024 CÁC CHUYÊN ĐỀ HAY VÀ KHÓ, 20 ĐỀ LUYỆN TẬP CÓ HƯỚNG DẪN CHI TIẾT.pdf
CHINH PHỤC KÌ THI VÀO LỚP 10 THPT MÔN TOÁN NĂM 2024 CÁC CHUYÊN ĐỀ HAY VÀ KHÓ, 20 ĐỀ LUYỆN TẬP CÓ HƯỚNG DẪN CHI TIẾT.pdf
CHINH PHỤC KÌ THI VÀO LỚP 10 THPT MÔN TOÁN NĂM 2024 CÁC CHUYÊN ĐỀ HAY VÀ KHÓ, 20 ĐỀ LUYỆN TẬP CÓ HƯỚNG DẪN CHI TIẾT.pdf
CHINH PHỤC KÌ THI VÀO LỚP 10 THPT MÔN TOÁN NĂM 2024 CÁC CHUYÊN ĐỀ HAY VÀ KHÓ, 20 ĐỀ LUYỆN TẬP CÓ HƯỚNG DẪN CHI TIẾT.pdf
CHINH PHỤC KÌ THI VÀO LỚP 10 THPT MÔN TOÁN NĂM 2024 CÁC CHUYÊN ĐỀ HAY VÀ KHÓ, 20 ĐỀ LUYỆN TẬP CÓ HƯỚNG DẪN CHI TIẾT.pdf
CHINH PHỤC KÌ THI VÀO LỚP 10 THPT MÔN TOÁN NĂM 2024 CÁC CHUYÊN ĐỀ HAY VÀ KHÓ, 20 ĐỀ LUYỆN TẬP CÓ HƯỚNG DẪN CHI TIẾT.pdf
CHINH PHỤC KÌ THI VÀO LỚP 10 THPT MÔN TOÁN NĂM 2024 CÁC CHUYÊN ĐỀ HAY VÀ KHÓ, 20 ĐỀ LUYỆN TẬP CÓ HƯỚNG DẪN CHI TIẾT.pdf
CHINH PHỤC KÌ THI VÀO LỚP 10 THPT MÔN TOÁN NĂM 2024 CÁC CHUYÊN ĐỀ HAY VÀ KHÓ, 20 ĐỀ LUYỆN TẬP CÓ HƯỚNG DẪN CHI TIẾT.pdf
CHINH PHỤC KÌ THI VÀO LỚP 10 THPT MÔN TOÁN NĂM 2024 CÁC CHUYÊN ĐỀ HAY VÀ KHÓ, 20 ĐỀ LUYỆN TẬP CÓ HƯỚNG DẪN CHI TIẾT.pdf
CHINH PHỤC KÌ THI VÀO LỚP 10 THPT MÔN TOÁN NĂM 2024 CÁC CHUYÊN ĐỀ HAY VÀ KHÓ, 20 ĐỀ LUYỆN TẬP CÓ HƯỚNG DẪN CHI TIẾT.pdf
CHINH PHỤC KÌ THI VÀO LỚP 10 THPT MÔN TOÁN NĂM 2024 CÁC CHUYÊN ĐỀ HAY VÀ KHÓ, 20 ĐỀ LUYỆN TẬP CÓ HƯỚNG DẪN CHI TIẾT.pdf
CHINH PHỤC KÌ THI VÀO LỚP 10 THPT MÔN TOÁN NĂM 2024 CÁC CHUYÊN ĐỀ HAY VÀ KHÓ, 20 ĐỀ LUYỆN TẬP CÓ HƯỚNG DẪN CHI TIẾT.pdf
CHINH PHỤC KÌ THI VÀO LỚP 10 THPT MÔN TOÁN NĂM 2024 CÁC CHUYÊN ĐỀ HAY VÀ KHÓ, 20 ĐỀ LUYỆN TẬP CÓ HƯỚNG DẪN CHI TIẾT.pdf
CHINH PHỤC KÌ THI VÀO LỚP 10 THPT MÔN TOÁN NĂM 2024 CÁC CHUYÊN ĐỀ HAY VÀ KHÓ, 20 ĐỀ LUYỆN TẬP CÓ HƯỚNG DẪN CHI TIẾT.pdf
CHINH PHỤC KÌ THI VÀO LỚP 10 THPT MÔN TOÁN NĂM 2024 CÁC CHUYÊN ĐỀ HAY VÀ KHÓ, 20 ĐỀ LUYỆN TẬP CÓ HƯỚNG DẪN CHI TIẾT.pdf
CHINH PHỤC KÌ THI VÀO LỚP 10 THPT MÔN TOÁN NĂM 2024 CÁC CHUYÊN ĐỀ HAY VÀ KHÓ, 20 ĐỀ LUYỆN TẬP CÓ HƯỚNG DẪN CHI TIẾT.pdf
CHINH PHỤC KÌ THI VÀO LỚP 10 THPT MÔN TOÁN NĂM 2024 CÁC CHUYÊN ĐỀ HAY VÀ KHÓ, 20 ĐỀ LUYỆN TẬP CÓ HƯỚNG DẪN CHI TIẾT.pdf
CHINH PHỤC KÌ THI VÀO LỚP 10 THPT MÔN TOÁN NĂM 2024 CÁC CHUYÊN ĐỀ HAY VÀ KHÓ, 20 ĐỀ LUYỆN TẬP CÓ HƯỚNG DẪN CHI TIẾT.pdf
CHINH PHỤC KÌ THI VÀO LỚP 10 THPT MÔN TOÁN NĂM 2024 CÁC CHUYÊN ĐỀ HAY VÀ KHÓ, 20 ĐỀ LUYỆN TẬP CÓ HƯỚNG DẪN CHI TIẾT.pdf
CHINH PHỤC KÌ THI VÀO LỚP 10 THPT MÔN TOÁN NĂM 2024 CÁC CHUYÊN ĐỀ HAY VÀ KHÓ, 20 ĐỀ LUYỆN TẬP CÓ HƯỚNG DẪN CHI TIẾT.pdf
CHINH PHỤC KÌ THI VÀO LỚP 10 THPT MÔN TOÁN NĂM 2024 CÁC CHUYÊN ĐỀ HAY VÀ KHÓ, 20 ĐỀ LUYỆN TẬP CÓ HƯỚNG DẪN CHI TIẾT.pdf
CHINH PHỤC KÌ THI VÀO LỚP 10 THPT MÔN TOÁN NĂM 2024 CÁC CHUYÊN ĐỀ HAY VÀ KHÓ, 20 ĐỀ LUYỆN TẬP CÓ HƯỚNG DẪN CHI TIẾT.pdf
CHINH PHỤC KÌ THI VÀO LỚP 10 THPT MÔN TOÁN NĂM 2024 CÁC CHUYÊN ĐỀ HAY VÀ KHÓ, 20 ĐỀ LUYỆN TẬP CÓ HƯỚNG DẪN CHI TIẾT.pdf
CHINH PHỤC KÌ THI VÀO LỚP 10 THPT MÔN TOÁN NĂM 2024 CÁC CHUYÊN ĐỀ HAY VÀ KHÓ, 20 ĐỀ LUYỆN TẬP CÓ HƯỚNG DẪN CHI TIẾT.pdf
CHINH PHỤC KÌ THI VÀO LỚP 10 THPT MÔN TOÁN NĂM 2024 CÁC CHUYÊN ĐỀ HAY VÀ KHÓ, 20 ĐỀ LUYỆN TẬP CÓ HƯỚNG DẪN CHI TIẾT.pdf
CHINH PHỤC KÌ THI VÀO LỚP 10 THPT MÔN TOÁN NĂM 2024 CÁC CHUYÊN ĐỀ HAY VÀ KHÓ, 20 ĐỀ LUYỆN TẬP CÓ HƯỚNG DẪN CHI TIẾT.pdf
CHINH PHỤC KÌ THI VÀO LỚP 10 THPT MÔN TOÁN NĂM 2024 CÁC CHUYÊN ĐỀ HAY VÀ KHÓ, 20 ĐỀ LUYỆN TẬP CÓ HƯỚNG DẪN CHI TIẾT.pdf
CHINH PHỤC KÌ THI VÀO LỚP 10 THPT MÔN TOÁN NĂM 2024 CÁC CHUYÊN ĐỀ HAY VÀ KHÓ, 20 ĐỀ LUYỆN TẬP CÓ HƯỚNG DẪN CHI TIẾT.pdf
CHINH PHỤC KÌ THI VÀO LỚP 10 THPT MÔN TOÁN NĂM 2024 CÁC CHUYÊN ĐỀ HAY VÀ KHÓ, 20 ĐỀ LUYỆN TẬP CÓ HƯỚNG DẪN CHI TIẾT.pdf
CHINH PHỤC KÌ THI VÀO LỚP 10 THPT MÔN TOÁN NĂM 2024 CÁC CHUYÊN ĐỀ HAY VÀ KHÓ, 20 ĐỀ LUYỆN TẬP CÓ HƯỚNG DẪN CHI TIẾT.pdf
CHINH PHỤC KÌ THI VÀO LỚP 10 THPT MÔN TOÁN NĂM 2024 CÁC CHUYÊN ĐỀ HAY VÀ KHÓ, 20 ĐỀ LUYỆN TẬP CÓ HƯỚNG DẪN CHI TIẾT.pdf
CHINH PHỤC KÌ THI VÀO LỚP 10 THPT MÔN TOÁN NĂM 2024 CÁC CHUYÊN ĐỀ HAY VÀ KHÓ, 20 ĐỀ LUYỆN TẬP CÓ HƯỚNG DẪN CHI TIẾT.pdf
CHINH PHỤC KÌ THI VÀO LỚP 10 THPT MÔN TOÁN NĂM 2024 CÁC CHUYÊN ĐỀ HAY VÀ KHÓ, 20 ĐỀ LUYỆN TẬP CÓ HƯỚNG DẪN CHI TIẾT.pdf
CHINH PHỤC KÌ THI VÀO LỚP 10 THPT MÔN TOÁN NĂM 2024 CÁC CHUYÊN ĐỀ HAY VÀ KHÓ, 20 ĐỀ LUYỆN TẬP CÓ HƯỚNG DẪN CHI TIẾT.pdf
CHINH PHỤC KÌ THI VÀO LỚP 10 THPT MÔN TOÁN NĂM 2024 CÁC CHUYÊN ĐỀ HAY VÀ KHÓ, 20 ĐỀ LUYỆN TẬP CÓ HƯỚNG DẪN CHI TIẾT.pdf
CHINH PHỤC KÌ THI VÀO LỚP 10 THPT MÔN TOÁN NĂM 2024 CÁC CHUYÊN ĐỀ HAY VÀ KHÓ, 20 ĐỀ LUYỆN TẬP CÓ HƯỚNG DẪN CHI TIẾT.pdf
CHINH PHỤC KÌ THI VÀO LỚP 10 THPT MÔN TOÁN NĂM 2024 CÁC CHUYÊN ĐỀ HAY VÀ KHÓ, 20 ĐỀ LUYỆN TẬP CÓ HƯỚNG DẪN CHI TIẾT.pdf
CHINH PHỤC KÌ THI VÀO LỚP 10 THPT MÔN TOÁN NĂM 2024 CÁC CHUYÊN ĐỀ HAY VÀ KHÓ, 20 ĐỀ LUYỆN TẬP CÓ HƯỚNG DẪN CHI TIẾT.pdf
CHINH PHỤC KÌ THI VÀO LỚP 10 THPT MÔN TOÁN NĂM 2024 CÁC CHUYÊN ĐỀ HAY VÀ KHÓ, 20 ĐỀ LUYỆN TẬP CÓ HƯỚNG DẪN CHI TIẾT.pdf
CHINH PHỤC KÌ THI VÀO LỚP 10 THPT MÔN TOÁN NĂM 2024 CÁC CHUYÊN ĐỀ HAY VÀ KHÓ, 20 ĐỀ LUYỆN TẬP CÓ HƯỚNG DẪN CHI TIẾT.pdf
CHINH PHỤC KÌ THI VÀO LỚP 10 THPT MÔN TOÁN NĂM 2024 CÁC CHUYÊN ĐỀ HAY VÀ KHÓ, 20 ĐỀ LUYỆN TẬP CÓ HƯỚNG DẪN CHI TIẾT.pdf
CHINH PHỤC KÌ THI VÀO LỚP 10 THPT MÔN TOÁN NĂM 2024 CÁC CHUYÊN ĐỀ HAY VÀ KHÓ, 20 ĐỀ LUYỆN TẬP CÓ HƯỚNG DẪN CHI TIẾT.pdf
CHINH PHỤC KÌ THI VÀO LỚP 10 THPT MÔN TOÁN NĂM 2024 CÁC CHUYÊN ĐỀ HAY VÀ KHÓ, 20 ĐỀ LUYỆN TẬP CÓ HƯỚNG DẪN CHI TIẾT.pdf
CHINH PHỤC KÌ THI VÀO LỚP 10 THPT MÔN TOÁN NĂM 2024 CÁC CHUYÊN ĐỀ HAY VÀ KHÓ, 20 ĐỀ LUYỆN TẬP CÓ HƯỚNG DẪN CHI TIẾT.pdf
CHINH PHỤC KÌ THI VÀO LỚP 10 THPT MÔN TOÁN NĂM 2024 CÁC CHUYÊN ĐỀ HAY VÀ KHÓ, 20 ĐỀ LUYỆN TẬP CÓ HƯỚNG DẪN CHI TIẾT.pdf
CHINH PHỤC KÌ THI VÀO LỚP 10 THPT MÔN TOÁN NĂM 2024 CÁC CHUYÊN ĐỀ HAY VÀ KHÓ, 20 ĐỀ LUYỆN TẬP CÓ HƯỚNG DẪN CHI TIẾT.pdf
CHINH PHỤC KÌ THI VÀO LỚP 10 THPT MÔN TOÁN NĂM 2024 CÁC CHUYÊN ĐỀ HAY VÀ KHÓ, 20 ĐỀ LUYỆN TẬP CÓ HƯỚNG DẪN CHI TIẾT.pdf
CHINH PHỤC KÌ THI VÀO LỚP 10 THPT MÔN TOÁN NĂM 2024 CÁC CHUYÊN ĐỀ HAY VÀ KHÓ, 20 ĐỀ LUYỆN TẬP CÓ HƯỚNG DẪN CHI TIẾT.pdf
CHINH PHỤC KÌ THI VÀO LỚP 10 THPT MÔN TOÁN NĂM 2024 CÁC CHUYÊN ĐỀ HAY VÀ KHÓ, 20 ĐỀ LUYỆN TẬP CÓ HƯỚNG DẪN CHI TIẾT.pdf
CHINH PHỤC KÌ THI VÀO LỚP 10 THPT MÔN TOÁN NĂM 2024 CÁC CHUYÊN ĐỀ HAY VÀ KHÓ, 20 ĐỀ LUYỆN TẬP CÓ HƯỚNG DẪN CHI TIẾT.pdf
CHINH PHỤC KÌ THI VÀO LỚP 10 THPT MÔN TOÁN NĂM 2024 CÁC CHUYÊN ĐỀ HAY VÀ KHÓ, 20 ĐỀ LUYỆN TẬP CÓ HƯỚNG DẪN CHI TIẾT.pdf
CHINH PHỤC KÌ THI VÀO LỚP 10 THPT MÔN TOÁN NĂM 2024 CÁC CHUYÊN ĐỀ HAY VÀ KHÓ, 20 ĐỀ LUYỆN TẬP CÓ HƯỚNG DẪN CHI TIẾT.pdf
CHINH PHỤC KÌ THI VÀO LỚP 10 THPT MÔN TOÁN NĂM 2024 CÁC CHUYÊN ĐỀ HAY VÀ KHÓ, 20 ĐỀ LUYỆN TẬP CÓ HƯỚNG DẪN CHI TIẾT.pdf
CHINH PHỤC KÌ THI VÀO LỚP 10 THPT MÔN TOÁN NĂM 2024 CÁC CHUYÊN ĐỀ HAY VÀ KHÓ, 20 ĐỀ LUYỆN TẬP CÓ HƯỚNG DẪN CHI TIẾT.pdf
CHINH PHỤC KÌ THI VÀO LỚP 10 THPT MÔN TOÁN NĂM 2024 CÁC CHUYÊN ĐỀ HAY VÀ KHÓ, 20 ĐỀ LUYỆN TẬP CÓ HƯỚNG DẪN CHI TIẾT.pdf
CHINH PHỤC KÌ THI VÀO LỚP 10 THPT MÔN TOÁN NĂM 2024 CÁC CHUYÊN ĐỀ HAY VÀ KHÓ, 20 ĐỀ LUYỆN TẬP CÓ HƯỚNG DẪN CHI TIẾT.pdf
CHINH PHỤC KÌ THI VÀO LỚP 10 THPT MÔN TOÁN NĂM 2024 CÁC CHUYÊN ĐỀ HAY VÀ KHÓ, 20 ĐỀ LUYỆN TẬP CÓ HƯỚNG DẪN CHI TIẾT.pdf
CHINH PHỤC KÌ THI VÀO LỚP 10 THPT MÔN TOÁN NĂM 2024 CÁC CHUYÊN ĐỀ HAY VÀ KHÓ, 20 ĐỀ LUYỆN TẬP CÓ HƯỚNG DẪN CHI TIẾT.pdf
CHINH PHỤC KÌ THI VÀO LỚP 10 THPT MÔN TOÁN NĂM 2024 CÁC CHUYÊN ĐỀ HAY VÀ KHÓ, 20 ĐỀ LUYỆN TẬP CÓ HƯỚNG DẪN CHI TIẾT.pdf
CHINH PHỤC KÌ THI VÀO LỚP 10 THPT MÔN TOÁN NĂM 2024 CÁC CHUYÊN ĐỀ HAY VÀ KHÓ, 20 ĐỀ LUYỆN TẬP CÓ HƯỚNG DẪN CHI TIẾT.pdf
CHINH PHỤC KÌ THI VÀO LỚP 10 THPT MÔN TOÁN NĂM 2024 CÁC CHUYÊN ĐỀ HAY VÀ KHÓ, 20 ĐỀ LUYỆN TẬP CÓ HƯỚNG DẪN CHI TIẾT.pdf
CHINH PHỤC KÌ THI VÀO LỚP 10 THPT MÔN TOÁN NĂM 2024 CÁC CHUYÊN ĐỀ HAY VÀ KHÓ, 20 ĐỀ LUYỆN TẬP CÓ HƯỚNG DẪN CHI TIẾT.pdf
CHINH PHỤC KÌ THI VÀO LỚP 10 THPT MÔN TOÁN NĂM 2024 CÁC CHUYÊN ĐỀ HAY VÀ KHÓ, 20 ĐỀ LUYỆN TẬP CÓ HƯỚNG DẪN CHI TIẾT.pdf
CHINH PHỤC KÌ THI VÀO LỚP 10 THPT MÔN TOÁN NĂM 2024 CÁC CHUYÊN ĐỀ HAY VÀ KHÓ, 20 ĐỀ LUYỆN TẬP CÓ HƯỚNG DẪN CHI TIẾT.pdf
CHINH PHỤC KÌ THI VÀO LỚP 10 THPT MÔN TOÁN NĂM 2024 CÁC CHUYÊN ĐỀ HAY VÀ KHÓ, 20 ĐỀ LUYỆN TẬP CÓ HƯỚNG DẪN CHI TIẾT.pdf
CHINH PHỤC KÌ THI VÀO LỚP 10 THPT MÔN TOÁN NĂM 2024 CÁC CHUYÊN ĐỀ HAY VÀ KHÓ, 20 ĐỀ LUYỆN TẬP CÓ HƯỚNG DẪN CHI TIẾT.pdf
CHINH PHỤC KÌ THI VÀO LỚP 10 THPT MÔN TOÁN NĂM 2024 CÁC CHUYÊN ĐỀ HAY VÀ KHÓ, 20 ĐỀ LUYỆN TẬP CÓ HƯỚNG DẪN CHI TIẾT.pdf
CHINH PHỤC KÌ THI VÀO LỚP 10 THPT MÔN TOÁN NĂM 2024 CÁC CHUYÊN ĐỀ HAY VÀ KHÓ, 20 ĐỀ LUYỆN TẬP CÓ HƯỚNG DẪN CHI TIẾT.pdf
CHINH PHỤC KÌ THI VÀO LỚP 10 THPT MÔN TOÁN NĂM 2024 CÁC CHUYÊN ĐỀ HAY VÀ KHÓ, 20 ĐỀ LUYỆN TẬP CÓ HƯỚNG DẪN CHI TIẾT.pdf
CHINH PHỤC KÌ THI VÀO LỚP 10 THPT MÔN TOÁN NĂM 2024 CÁC CHUYÊN ĐỀ HAY VÀ KHÓ, 20 ĐỀ LUYỆN TẬP CÓ HƯỚNG DẪN CHI TIẾT.pdf
CHINH PHỤC KÌ THI VÀO LỚP 10 THPT MÔN TOÁN NĂM 2024 CÁC CHUYÊN ĐỀ HAY VÀ KHÓ, 20 ĐỀ LUYỆN TẬP CÓ HƯỚNG DẪN CHI TIẾT.pdf
CHINH PHỤC KÌ THI VÀO LỚP 10 THPT MÔN TOÁN NĂM 2024 CÁC CHUYÊN ĐỀ HAY VÀ KHÓ, 20 ĐỀ LUYỆN TẬP CÓ HƯỚNG DẪN CHI TIẾT.pdf
CHINH PHỤC KÌ THI VÀO LỚP 10 THPT MÔN TOÁN NĂM 2024 CÁC CHUYÊN ĐỀ HAY VÀ KHÓ, 20 ĐỀ LUYỆN TẬP CÓ HƯỚNG DẪN CHI TIẾT.pdf
CHINH PHỤC KÌ THI VÀO LỚP 10 THPT MÔN TOÁN NĂM 2024 CÁC CHUYÊN ĐỀ HAY VÀ KHÓ, 20 ĐỀ LUYỆN TẬP CÓ HƯỚNG DẪN CHI TIẾT.pdf
CHINH PHỤC KÌ THI VÀO LỚP 10 THPT MÔN TOÁN NĂM 2024 CÁC CHUYÊN ĐỀ HAY VÀ KHÓ, 20 ĐỀ LUYỆN TẬP CÓ HƯỚNG DẪN CHI TIẾT.pdf
CHINH PHỤC KÌ THI VÀO LỚP 10 THPT MÔN TOÁN NĂM 2024 CÁC CHUYÊN ĐỀ HAY VÀ KHÓ, 20 ĐỀ LUYỆN TẬP CÓ HƯỚNG DẪN CHI TIẾT.pdf
CHINH PHỤC KÌ THI VÀO LỚP 10 THPT MÔN TOÁN NĂM 2024 CÁC CHUYÊN ĐỀ HAY VÀ KHÓ, 20 ĐỀ LUYỆN TẬP CÓ HƯỚNG DẪN CHI TIẾT.pdf
CHINH PHỤC KÌ THI VÀO LỚP 10 THPT MÔN TOÁN NĂM 2024 CÁC CHUYÊN ĐỀ HAY VÀ KHÓ, 20 ĐỀ LUYỆN TẬP CÓ HƯỚNG DẪN CHI TIẾT.pdf
CHINH PHỤC KÌ THI VÀO LỚP 10 THPT MÔN TOÁN NĂM 2024 CÁC CHUYÊN ĐỀ HAY VÀ KHÓ, 20 ĐỀ LUYỆN TẬP CÓ HƯỚNG DẪN CHI TIẾT.pdf
CHINH PHỤC KÌ THI VÀO LỚP 10 THPT MÔN TOÁN NĂM 2024 CÁC CHUYÊN ĐỀ HAY VÀ KHÓ, 20 ĐỀ LUYỆN TẬP CÓ HƯỚNG DẪN CHI TIẾT.pdf
CHINH PHỤC KÌ THI VÀO LỚP 10 THPT MÔN TOÁN NĂM 2024 CÁC CHUYÊN ĐỀ HAY VÀ KHÓ, 20 ĐỀ LUYỆN TẬP CÓ HƯỚNG DẪN CHI TIẾT.pdf
CHINH PHỤC KÌ THI VÀO LỚP 10 THPT MÔN TOÁN NĂM 2024 CÁC CHUYÊN ĐỀ HAY VÀ KHÓ, 20 ĐỀ LUYỆN TẬP CÓ HƯỚNG DẪN CHI TIẾT.pdf
CHINH PHỤC KÌ THI VÀO LỚP 10 THPT MÔN TOÁN NĂM 2024 CÁC CHUYÊN ĐỀ HAY VÀ KHÓ, 20 ĐỀ LUYỆN TẬP CÓ HƯỚNG DẪN CHI TIẾT.pdf
CHINH PHỤC KÌ THI VÀO LỚP 10 THPT MÔN TOÁN NĂM 2024 CÁC CHUYÊN ĐỀ HAY VÀ KHÓ, 20 ĐỀ LUYỆN TẬP CÓ HƯỚNG DẪN CHI TIẾT.pdf
CHINH PHỤC KÌ THI VÀO LỚP 10 THPT MÔN TOÁN NĂM 2024 CÁC CHUYÊN ĐỀ HAY VÀ KHÓ, 20 ĐỀ LUYỆN TẬP CÓ HƯỚNG DẪN CHI TIẾT.pdf
CHINH PHỤC KÌ THI VÀO LỚP 10 THPT MÔN TOÁN NĂM 2024 CÁC CHUYÊN ĐỀ HAY VÀ KHÓ, 20 ĐỀ LUYỆN TẬP CÓ HƯỚNG DẪN CHI TIẾT.pdf
CHINH PHỤC KÌ THI VÀO LỚP 10 THPT MÔN TOÁN NĂM 2024 CÁC CHUYÊN ĐỀ HAY VÀ KHÓ, 20 ĐỀ LUYỆN TẬP CÓ HƯỚNG DẪN CHI TIẾT.pdf
CHINH PHỤC KÌ THI VÀO LỚP 10 THPT MÔN TOÁN NĂM 2024 CÁC CHUYÊN ĐỀ HAY VÀ KHÓ, 20 ĐỀ LUYỆN TẬP CÓ HƯỚNG DẪN CHI TIẾT.pdf
CHINH PHỤC KÌ THI VÀO LỚP 10 THPT MÔN TOÁN NĂM 2024 CÁC CHUYÊN ĐỀ HAY VÀ KHÓ, 20 ĐỀ LUYỆN TẬP CÓ HƯỚNG DẪN CHI TIẾT.pdf
CHINH PHỤC KÌ THI VÀO LỚP 10 THPT MÔN TOÁN NĂM 2024 CÁC CHUYÊN ĐỀ HAY VÀ KHÓ, 20 ĐỀ LUYỆN TẬP CÓ HƯỚNG DẪN CHI TIẾT.pdf
CHINH PHỤC KÌ THI VÀO LỚP 10 THPT MÔN TOÁN NĂM 2024 CÁC CHUYÊN ĐỀ HAY VÀ KHÓ, 20 ĐỀ LUYỆN TẬP CÓ HƯỚNG DẪN CHI TIẾT.pdf
CHINH PHỤC KÌ THI VÀO LỚP 10 THPT MÔN TOÁN NĂM 2024 CÁC CHUYÊN ĐỀ HAY VÀ KHÓ, 20 ĐỀ LUYỆN TẬP CÓ HƯỚNG DẪN CHI TIẾT.pdf
CHINH PHỤC KÌ THI VÀO LỚP 10 THPT MÔN TOÁN NĂM 2024 CÁC CHUYÊN ĐỀ HAY VÀ KHÓ, 20 ĐỀ LUYỆN TẬP CÓ HƯỚNG DẪN CHI TIẾT.pdf
CHINH PHỤC KÌ THI VÀO LỚP 10 THPT MÔN TOÁN NĂM 2024 CÁC CHUYÊN ĐỀ HAY VÀ KHÓ, 20 ĐỀ LUYỆN TẬP CÓ HƯỚNG DẪN CHI TIẾT.pdf
CHINH PHỤC KÌ THI VÀO LỚP 10 THPT MÔN TOÁN NĂM 2024 CÁC CHUYÊN ĐỀ HAY VÀ KHÓ, 20 ĐỀ LUYỆN TẬP CÓ HƯỚNG DẪN CHI TIẾT.pdf
CHINH PHỤC KÌ THI VÀO LỚP 10 THPT MÔN TOÁN NĂM 2024 CÁC CHUYÊN ĐỀ HAY VÀ KHÓ, 20 ĐỀ LUYỆN TẬP CÓ HƯỚNG DẪN CHI TIẾT.pdf
CHINH PHỤC KÌ THI VÀO LỚP 10 THPT MÔN TOÁN NĂM 2024 CÁC CHUYÊN ĐỀ HAY VÀ KHÓ, 20 ĐỀ LUYỆN TẬP CÓ HƯỚNG DẪN CHI TIẾT.pdf
CHINH PHỤC KÌ THI VÀO LỚP 10 THPT MÔN TOÁN NĂM 2024 CÁC CHUYÊN ĐỀ HAY VÀ KHÓ, 20 ĐỀ LUYỆN TẬP CÓ HƯỚNG DẪN CHI TIẾT.pdf
CHINH PHỤC KÌ THI VÀO LỚP 10 THPT MÔN TOÁN NĂM 2024 CÁC CHUYÊN ĐỀ HAY VÀ KHÓ, 20 ĐỀ LUYỆN TẬP CÓ HƯỚNG DẪN CHI TIẾT.pdf
CHINH PHỤC KÌ THI VÀO LỚP 10 THPT MÔN TOÁN NĂM 2024 CÁC CHUYÊN ĐỀ HAY VÀ KHÓ, 20 ĐỀ LUYỆN TẬP CÓ HƯỚNG DẪN CHI TIẾT.pdf
CHINH PHỤC KÌ THI VÀO LỚP 10 THPT MÔN TOÁN NĂM 2024 CÁC CHUYÊN ĐỀ HAY VÀ KHÓ, 20 ĐỀ LUYỆN TẬP CÓ HƯỚNG DẪN CHI TIẾT.pdf
CHINH PHỤC KÌ THI VÀO LỚP 10 THPT MÔN TOÁN NĂM 2024 CÁC CHUYÊN ĐỀ HAY VÀ KHÓ, 20 ĐỀ LUYỆN TẬP CÓ HƯỚNG DẪN CHI TIẾT.pdf
CHINH PHỤC KÌ THI VÀO LỚP 10 THPT MÔN TOÁN NĂM 2024 CÁC CHUYÊN ĐỀ HAY VÀ KHÓ, 20 ĐỀ LUYỆN TẬP CÓ HƯỚNG DẪN CHI TIẾT.pdf
CHINH PHỤC KÌ THI VÀO LỚP 10 THPT MÔN TOÁN NĂM 2024 CÁC CHUYÊN ĐỀ HAY VÀ KHÓ, 20 ĐỀ LUYỆN TẬP CÓ HƯỚNG DẪN CHI TIẾT.pdf
CHINH PHỤC KÌ THI VÀO LỚP 10 THPT MÔN TOÁN NĂM 2024 CÁC CHUYÊN ĐỀ HAY VÀ KHÓ, 20 ĐỀ LUYỆN TẬP CÓ HƯỚNG DẪN CHI TIẾT.pdf
CHINH PHỤC KÌ THI VÀO LỚP 10 THPT MÔN TOÁN NĂM 2024 CÁC CHUYÊN ĐỀ HAY VÀ KHÓ, 20 ĐỀ LUYỆN TẬP CÓ HƯỚNG DẪN CHI TIẾT.pdf
CHINH PHỤC KÌ THI VÀO LỚP 10 THPT MÔN TOÁN NĂM 2024 CÁC CHUYÊN ĐỀ HAY VÀ KHÓ, 20 ĐỀ LUYỆN TẬP CÓ HƯỚNG DẪN CHI TIẾT.pdf
CHINH PHỤC KÌ THI VÀO LỚP 10 THPT MÔN TOÁN NĂM 2024 CÁC CHUYÊN ĐỀ HAY VÀ KHÓ, 20 ĐỀ LUYỆN TẬP CÓ HƯỚNG DẪN CHI TIẾT.pdf
CHINH PHỤC KÌ THI VÀO LỚP 10 THPT MÔN TOÁN NĂM 2024 CÁC CHUYÊN ĐỀ HAY VÀ KHÓ, 20 ĐỀ LUYỆN TẬP CÓ HƯỚNG DẪN CHI TIẾT.pdf
CHINH PHỤC KÌ THI VÀO LỚP 10 THPT MÔN TOÁN NĂM 2024 CÁC CHUYÊN ĐỀ HAY VÀ KHÓ, 20 ĐỀ LUYỆN TẬP CÓ HƯỚNG DẪN CHI TIẾT.pdf
CHINH PHỤC KÌ THI VÀO LỚP 10 THPT MÔN TOÁN NĂM 2024 CÁC CHUYÊN ĐỀ HAY VÀ KHÓ, 20 ĐỀ LUYỆN TẬP CÓ HƯỚNG DẪN CHI TIẾT.pdf
CHINH PHỤC KÌ THI VÀO LỚP 10 THPT MÔN TOÁN NĂM 2024 CÁC CHUYÊN ĐỀ HAY VÀ KHÓ, 20 ĐỀ LUYỆN TẬP CÓ HƯỚNG DẪN CHI TIẾT.pdf
CHINH PHỤC KÌ THI VÀO LỚP 10 THPT MÔN TOÁN NĂM 2024 CÁC CHUYÊN ĐỀ HAY VÀ KHÓ, 20 ĐỀ LUYỆN TẬP CÓ HƯỚNG DẪN CHI TIẾT.pdf
CHINH PHỤC KÌ THI VÀO LỚP 10 THPT MÔN TOÁN NĂM 2024 CÁC CHUYÊN ĐỀ HAY VÀ KHÓ, 20 ĐỀ LUYỆN TẬP CÓ HƯỚNG DẪN CHI TIẾT.pdf
CHINH PHỤC KÌ THI VÀO LỚP 10 THPT MÔN TOÁN NĂM 2024 CÁC CHUYÊN ĐỀ HAY VÀ KHÓ, 20 ĐỀ LUYỆN TẬP CÓ HƯỚNG DẪN CHI TIẾT.pdf
CHINH PHỤC KÌ THI VÀO LỚP 10 THPT MÔN TOÁN NĂM 2024 CÁC CHUYÊN ĐỀ HAY VÀ KHÓ, 20 ĐỀ LUYỆN TẬP CÓ HƯỚNG DẪN CHI TIẾT.pdf
CHINH PHỤC KÌ THI VÀO LỚP 10 THPT MÔN TOÁN NĂM 2024 CÁC CHUYÊN ĐỀ HAY VÀ KHÓ, 20 ĐỀ LUYỆN TẬP CÓ HƯỚNG DẪN CHI TIẾT.pdf
CHINH PHỤC KÌ THI VÀO LỚP 10 THPT MÔN TOÁN NĂM 2024 CÁC CHUYÊN ĐỀ HAY VÀ KHÓ, 20 ĐỀ LUYỆN TẬP CÓ HƯỚNG DẪN CHI TIẾT.pdf
CHINH PHỤC KÌ THI VÀO LỚP 10 THPT MÔN TOÁN NĂM 2024 CÁC CHUYÊN ĐỀ HAY VÀ KHÓ, 20 ĐỀ LUYỆN TẬP CÓ HƯỚNG DẪN CHI TIẾT.pdf
CHINH PHỤC KÌ THI VÀO LỚP 10 THPT MÔN TOÁN NĂM 2024 CÁC CHUYÊN ĐỀ HAY VÀ KHÓ, 20 ĐỀ LUYỆN TẬP CÓ HƯỚNG DẪN CHI TIẾT.pdf
CHINH PHỤC KÌ THI VÀO LỚP 10 THPT MÔN TOÁN NĂM 2024 CÁC CHUYÊN ĐỀ HAY VÀ KHÓ, 20 ĐỀ LUYỆN TẬP CÓ HƯỚNG DẪN CHI TIẾT.pdf
CHINH PHỤC KÌ THI VÀO LỚP 10 THPT MÔN TOÁN NĂM 2024 CÁC CHUYÊN ĐỀ HAY VÀ KHÓ, 20 ĐỀ LUYỆN TẬP CÓ HƯỚNG DẪN CHI TIẾT.pdf
CHINH PHỤC KÌ THI VÀO LỚP 10 THPT MÔN TOÁN NĂM 2024 CÁC CHUYÊN ĐỀ HAY VÀ KHÓ, 20 ĐỀ LUYỆN TẬP CÓ HƯỚNG DẪN CHI TIẾT.pdf
CHINH PHỤC KÌ THI VÀO LỚP 10 THPT MÔN TOÁN NĂM 2024 CÁC CHUYÊN ĐỀ HAY VÀ KHÓ, 20 ĐỀ LUYỆN TẬP CÓ HƯỚNG DẪN CHI TIẾT.pdf
CHINH PHỤC KÌ THI VÀO LỚP 10 THPT MÔN TOÁN NĂM 2024 CÁC CHUYÊN ĐỀ HAY VÀ KHÓ, 20 ĐỀ LUYỆN TẬP CÓ HƯỚNG DẪN CHI TIẾT.pdf
CHINH PHỤC KÌ THI VÀO LỚP 10 THPT MÔN TOÁN NĂM 2024 CÁC CHUYÊN ĐỀ HAY VÀ KHÓ, 20 ĐỀ LUYỆN TẬP CÓ HƯỚNG DẪN CHI TIẾT.pdf
CHINH PHỤC KÌ THI VÀO LỚP 10 THPT MÔN TOÁN NĂM 2024 CÁC CHUYÊN ĐỀ HAY VÀ KHÓ, 20 ĐỀ LUYỆN TẬP CÓ HƯỚNG DẪN CHI TIẾT.pdf
CHINH PHỤC KÌ THI VÀO LỚP 10 THPT MÔN TOÁN NĂM 2024 CÁC CHUYÊN ĐỀ HAY VÀ KHÓ, 20 ĐỀ LUYỆN TẬP CÓ HƯỚNG DẪN CHI TIẾT.pdf
CHINH PHỤC KÌ THI VÀO LỚP 10 THPT MÔN TOÁN NĂM 2024 CÁC CHUYÊN ĐỀ HAY VÀ KHÓ, 20 ĐỀ LUYỆN TẬP CÓ HƯỚNG DẪN CHI TIẾT.pdf
CHINH PHỤC KÌ THI VÀO LỚP 10 THPT MÔN TOÁN NĂM 2024 CÁC CHUYÊN ĐỀ HAY VÀ KHÓ, 20 ĐỀ LUYỆN TẬP CÓ HƯỚNG DẪN CHI TIẾT.pdf
CHINH PHỤC KÌ THI VÀO LỚP 10 THPT MÔN TOÁN NĂM 2024 CÁC CHUYÊN ĐỀ HAY VÀ KHÓ, 20 ĐỀ LUYỆN TẬP CÓ HƯỚNG DẪN CHI TIẾT.pdf
CHINH PHỤC KÌ THI VÀO LỚP 10 THPT MÔN TOÁN NĂM 2024 CÁC CHUYÊN ĐỀ HAY VÀ KHÓ, 20 ĐỀ LUYỆN TẬP CÓ HƯỚNG DẪN CHI TIẾT.pdf
CHINH PHỤC KÌ THI VÀO LỚP 10 THPT MÔN TOÁN NĂM 2024 CÁC CHUYÊN ĐỀ HAY VÀ KHÓ, 20 ĐỀ LUYỆN TẬP CÓ HƯỚNG DẪN CHI TIẾT.pdf
CHINH PHỤC KÌ THI VÀO LỚP 10 THPT MÔN TOÁN NĂM 2024 CÁC CHUYÊN ĐỀ HAY VÀ KHÓ, 20 ĐỀ LUYỆN TẬP CÓ HƯỚNG DẪN CHI TIẾT.pdf
CHINH PHỤC KÌ THI VÀO LỚP 10 THPT MÔN TOÁN NĂM 2024 CÁC CHUYÊN ĐỀ HAY VÀ KHÓ, 20 ĐỀ LUYỆN TẬP CÓ HƯỚNG DẪN CHI TIẾT.pdf
CHINH PHỤC KÌ THI VÀO LỚP 10 THPT MÔN TOÁN NĂM 2024 CÁC CHUYÊN ĐỀ HAY VÀ KHÓ, 20 ĐỀ LUYỆN TẬP CÓ HƯỚNG DẪN CHI TIẾT.pdf
CHINH PHỤC KÌ THI VÀO LỚP 10 THPT MÔN TOÁN NĂM 2024 CÁC CHUYÊN ĐỀ HAY VÀ KHÓ, 20 ĐỀ LUYỆN TẬP CÓ HƯỚNG DẪN CHI TIẾT.pdf
CHINH PHỤC KÌ THI VÀO LỚP 10 THPT MÔN TOÁN NĂM 2024 CÁC CHUYÊN ĐỀ HAY VÀ KHÓ, 20 ĐỀ LUYỆN TẬP CÓ HƯỚNG DẪN CHI TIẾT.pdf
CHINH PHỤC KÌ THI VÀO LỚP 10 THPT MÔN TOÁN NĂM 2024 CÁC CHUYÊN ĐỀ HAY VÀ KHÓ, 20 ĐỀ LUYỆN TẬP CÓ HƯỚNG DẪN CHI TIẾT.pdf
CHINH PHỤC KÌ THI VÀO LỚP 10 THPT MÔN TOÁN NĂM 2024 CÁC CHUYÊN ĐỀ HAY VÀ KHÓ, 20 ĐỀ LUYỆN TẬP CÓ HƯỚNG DẪN CHI TIẾT.pdf
CHINH PHỤC KÌ THI VÀO LỚP 10 THPT MÔN TOÁN NĂM 2024 CÁC CHUYÊN ĐỀ HAY VÀ KHÓ, 20 ĐỀ LUYỆN TẬP CÓ HƯỚNG DẪN CHI TIẾT.pdf
CHINH PHỤC KÌ THI VÀO LỚP 10 THPT MÔN TOÁN NĂM 2024 CÁC CHUYÊN ĐỀ HAY VÀ KHÓ, 20 ĐỀ LUYỆN TẬP CÓ HƯỚNG DẪN CHI TIẾT.pdf
CHINH PHỤC KÌ THI VÀO LỚP 10 THPT MÔN TOÁN NĂM 2024 CÁC CHUYÊN ĐỀ HAY VÀ KHÓ, 20 ĐỀ LUYỆN TẬP CÓ HƯỚNG DẪN CHI TIẾT.pdf
CHINH PHỤC KÌ THI VÀO LỚP 10 THPT MÔN TOÁN NĂM 2024 CÁC CHUYÊN ĐỀ HAY VÀ KHÓ, 20 ĐỀ LUYỆN TẬP CÓ HƯỚNG DẪN CHI TIẾT.pdf
CHINH PHỤC KÌ THI VÀO LỚP 10 THPT MÔN TOÁN NĂM 2024 CÁC CHUYÊN ĐỀ HAY VÀ KHÓ, 20 ĐỀ LUYỆN TẬP CÓ HƯỚNG DẪN CHI TIẾT.pdf
CHINH PHỤC KÌ THI VÀO LỚP 10 THPT MÔN TOÁN NĂM 2024 CÁC CHUYÊN ĐỀ HAY VÀ KHÓ, 20 ĐỀ LUYỆN TẬP CÓ HƯỚNG DẪN CHI TIẾT.pdf
CHINH PHỤC KÌ THI VÀO LỚP 10 THPT MÔN TOÁN NĂM 2024 CÁC CHUYÊN ĐỀ HAY VÀ KHÓ, 20 ĐỀ LUYỆN TẬP CÓ HƯỚNG DẪN CHI TIẾT.pdf
CHINH PHỤC KÌ THI VÀO LỚP 10 THPT MÔN TOÁN NĂM 2024 CÁC CHUYÊN ĐỀ HAY VÀ KHÓ, 20 ĐỀ LUYỆN TẬP CÓ HƯỚNG DẪN CHI TIẾT.pdf
CHINH PHỤC KÌ THI VÀO LỚP 10 THPT MÔN TOÁN NĂM 2024 CÁC CHUYÊN ĐỀ HAY VÀ KHÓ, 20 ĐỀ LUYỆN TẬP CÓ HƯỚNG DẪN CHI TIẾT.pdf
CHINH PHỤC KÌ THI VÀO LỚP 10 THPT MÔN TOÁN NĂM 2024 CÁC CHUYÊN ĐỀ HAY VÀ KHÓ, 20 ĐỀ LUYỆN TẬP CÓ HƯỚNG DẪN CHI TIẾT.pdf
CHINH PHỤC KÌ THI VÀO LỚP 10 THPT MÔN TOÁN NĂM 2024 CÁC CHUYÊN ĐỀ HAY VÀ KHÓ, 20 ĐỀ LUYỆN TẬP CÓ HƯỚNG DẪN CHI TIẾT.pdf
CHINH PHỤC KÌ THI VÀO LỚP 10 THPT MÔN TOÁN NĂM 2024 CÁC CHUYÊN ĐỀ HAY VÀ KHÓ, 20 ĐỀ LUYỆN TẬP CÓ HƯỚNG DẪN CHI TIẾT.pdf
CHINH PHỤC KÌ THI VÀO LỚP 10 THPT MÔN TOÁN NĂM 2024 CÁC CHUYÊN ĐỀ HAY VÀ KHÓ, 20 ĐỀ LUYỆN TẬP CÓ HƯỚNG DẪN CHI TIẾT.pdf
CHINH PHỤC KÌ THI VÀO LỚP 10 THPT MÔN TOÁN NĂM 2024 CÁC CHUYÊN ĐỀ HAY VÀ KHÓ, 20 ĐỀ LUYỆN TẬP CÓ HƯỚNG DẪN CHI TIẾT.pdf
CHINH PHỤC KÌ THI VÀO LỚP 10 THPT MÔN TOÁN NĂM 2024 CÁC CHUYÊN ĐỀ HAY VÀ KHÓ, 20 ĐỀ LUYỆN TẬP CÓ HƯỚNG DẪN CHI TIẾT.pdf
CHINH PHỤC KÌ THI VÀO LỚP 10 THPT MÔN TOÁN NĂM 2024 CÁC CHUYÊN ĐỀ HAY VÀ KHÓ, 20 ĐỀ LUYỆN TẬP CÓ HƯỚNG DẪN CHI TIẾT.pdf
CHINH PHỤC KÌ THI VÀO LỚP 10 THPT MÔN TOÁN NĂM 2024 CÁC CHUYÊN ĐỀ HAY VÀ KHÓ, 20 ĐỀ LUYỆN TẬP CÓ HƯỚNG DẪN CHI TIẾT.pdf
CHINH PHỤC KÌ THI VÀO LỚP 10 THPT MÔN TOÁN NĂM 2024 CÁC CHUYÊN ĐỀ HAY VÀ KHÓ, 20 ĐỀ LUYỆN TẬP CÓ HƯỚNG DẪN CHI TIẾT.pdf
CHINH PHỤC KÌ THI VÀO LỚP 10 THPT MÔN TOÁN NĂM 2024 CÁC CHUYÊN ĐỀ HAY VÀ KHÓ, 20 ĐỀ LUYỆN TẬP CÓ HƯỚNG DẪN CHI TIẾT.pdf
CHINH PHỤC KÌ THI VÀO LỚP 10 THPT MÔN TOÁN NĂM 2024 CÁC CHUYÊN ĐỀ HAY VÀ KHÓ, 20 ĐỀ LUYỆN TẬP CÓ HƯỚNG DẪN CHI TIẾT.pdf
CHINH PHỤC KÌ THI VÀO LỚP 10 THPT MÔN TOÁN NĂM 2024 CÁC CHUYÊN ĐỀ HAY VÀ KHÓ, 20 ĐỀ LUYỆN TẬP CÓ HƯỚNG DẪN CHI TIẾT.pdf
CHINH PHỤC KÌ THI VÀO LỚP 10 THPT MÔN TOÁN NĂM 2024 CÁC CHUYÊN ĐỀ HAY VÀ KHÓ, 20 ĐỀ LUYỆN TẬP CÓ HƯỚNG DẪN CHI TIẾT.pdf
CHINH PHỤC KÌ THI VÀO LỚP 10 THPT MÔN TOÁN NĂM 2024 CÁC CHUYÊN ĐỀ HAY VÀ KHÓ, 20 ĐỀ LUYỆN TẬP CÓ HƯỚNG DẪN CHI TIẾT.pdf
CHINH PHỤC KÌ THI VÀO LỚP 10 THPT MÔN TOÁN NĂM 2024 CÁC CHUYÊN ĐỀ HAY VÀ KHÓ, 20 ĐỀ LUYỆN TẬP CÓ HƯỚNG DẪN CHI TIẾT.pdf
CHINH PHỤC KÌ THI VÀO LỚP 10 THPT MÔN TOÁN NĂM 2024 CÁC CHUYÊN ĐỀ HAY VÀ KHÓ, 20 ĐỀ LUYỆN TẬP CÓ HƯỚNG DẪN CHI TIẾT.pdf
CHINH PHỤC KÌ THI VÀO LỚP 10 THPT MÔN TOÁN NĂM 2024 CÁC CHUYÊN ĐỀ HAY VÀ KHÓ, 20 ĐỀ LUYỆN TẬP CÓ HƯỚNG DẪN CHI TIẾT.pdf
CHINH PHỤC KÌ THI VÀO LỚP 10 THPT MÔN TOÁN NĂM 2024 CÁC CHUYÊN ĐỀ HAY VÀ KHÓ, 20 ĐỀ LUYỆN TẬP CÓ HƯỚNG DẪN CHI TIẾT.pdf
CHINH PHỤC KÌ THI VÀO LỚP 10 THPT MÔN TOÁN NĂM 2024 CÁC CHUYÊN ĐỀ HAY VÀ KHÓ, 20 ĐỀ LUYỆN TẬP CÓ HƯỚNG DẪN CHI TIẾT.pdf
CHINH PHỤC KÌ THI VÀO LỚP 10 THPT MÔN TOÁN NĂM 2024 CÁC CHUYÊN ĐỀ HAY VÀ KHÓ, 20 ĐỀ LUYỆN TẬP CÓ HƯỚNG DẪN CHI TIẾT.pdf
CHINH PHỤC KÌ THI VÀO LỚP 10 THPT MÔN TOÁN NĂM 2024 CÁC CHUYÊN ĐỀ HAY VÀ KHÓ, 20 ĐỀ LUYỆN TẬP CÓ HƯỚNG DẪN CHI TIẾT.pdf
CHINH PHỤC KÌ THI VÀO LỚP 10 THPT MÔN TOÁN NĂM 2024 CÁC CHUYÊN ĐỀ HAY VÀ KHÓ, 20 ĐỀ LUYỆN TẬP CÓ HƯỚNG DẪN CHI TIẾT.pdf
CHINH PHỤC KÌ THI VÀO LỚP 10 THPT MÔN TOÁN NĂM 2024 CÁC CHUYÊN ĐỀ HAY VÀ KHÓ, 20 ĐỀ LUYỆN TẬP CÓ HƯỚNG DẪN CHI TIẾT.pdf
CHINH PHỤC KÌ THI VÀO LỚP 10 THPT MÔN TOÁN NĂM 2024 CÁC CHUYÊN ĐỀ HAY VÀ KHÓ, 20 ĐỀ LUYỆN TẬP CÓ HƯỚNG DẪN CHI TIẾT.pdf
CHINH PHỤC KÌ THI VÀO LỚP 10 THPT MÔN TOÁN NĂM 2024 CÁC CHUYÊN ĐỀ HAY VÀ KHÓ, 20 ĐỀ LUYỆN TẬP CÓ HƯỚNG DẪN CHI TIẾT.pdf
CHINH PHỤC KÌ THI VÀO LỚP 10 THPT MÔN TOÁN NĂM 2024 CÁC CHUYÊN ĐỀ HAY VÀ KHÓ, 20 ĐỀ LUYỆN TẬP CÓ HƯỚNG DẪN CHI TIẾT.pdf
CHINH PHỤC KÌ THI VÀO LỚP 10 THPT MÔN TOÁN NĂM 2024 CÁC CHUYÊN ĐỀ HAY VÀ KHÓ, 20 ĐỀ LUYỆN TẬP CÓ HƯỚNG DẪN CHI TIẾT.pdf
CHINH PHỤC KÌ THI VÀO LỚP 10 THPT MÔN TOÁN NĂM 2024 CÁC CHUYÊN ĐỀ HAY VÀ KHÓ, 20 ĐỀ LUYỆN TẬP CÓ HƯỚNG DẪN CHI TIẾT.pdf
CHINH PHỤC KÌ THI VÀO LỚP 10 THPT MÔN TOÁN NĂM 2024 CÁC CHUYÊN ĐỀ HAY VÀ KHÓ, 20 ĐỀ LUYỆN TẬP CÓ HƯỚNG DẪN CHI TIẾT.pdf
CHINH PHỤC KÌ THI VÀO LỚP 10 THPT MÔN TOÁN NĂM 2024 CÁC CHUYÊN ĐỀ HAY VÀ KHÓ, 20 ĐỀ LUYỆN TẬP CÓ HƯỚNG DẪN CHI TIẾT.pdf
CHINH PHỤC KÌ THI VÀO LỚP 10 THPT MÔN TOÁN NĂM 2024 CÁC CHUYÊN ĐỀ HAY VÀ KHÓ, 20 ĐỀ LUYỆN TẬP CÓ HƯỚNG DẪN CHI TIẾT.pdf
CHINH PHỤC KÌ THI VÀO LỚP 10 THPT MÔN TOÁN NĂM 2024 CÁC CHUYÊN ĐỀ HAY VÀ KHÓ, 20 ĐỀ LUYỆN TẬP CÓ HƯỚNG DẪN CHI TIẾT.pdf
CHINH PHỤC KÌ THI VÀO LỚP 10 THPT MÔN TOÁN NĂM 2024 CÁC CHUYÊN ĐỀ HAY VÀ KHÓ, 20 ĐỀ LUYỆN TẬP CÓ HƯỚNG DẪN CHI TIẾT.pdf
CHINH PHỤC KÌ THI VÀO LỚP 10 THPT MÔN TOÁN NĂM 2024 CÁC CHUYÊN ĐỀ HAY VÀ KHÓ, 20 ĐỀ LUYỆN TẬP CÓ HƯỚNG DẪN CHI TIẾT.pdf
CHINH PHỤC KÌ THI VÀO LỚP 10 THPT MÔN TOÁN NĂM 2024 CÁC CHUYÊN ĐỀ HAY VÀ KHÓ, 20 ĐỀ LUYỆN TẬP CÓ HƯỚNG DẪN CHI TIẾT.pdf
CHINH PHỤC KÌ THI VÀO LỚP 10 THPT MÔN TOÁN NĂM 2024 CÁC CHUYÊN ĐỀ HAY VÀ KHÓ, 20 ĐỀ LUYỆN TẬP CÓ HƯỚNG DẪN CHI TIẾT.pdf
CHINH PHỤC KÌ THI VÀO LỚP 10 THPT MÔN TOÁN NĂM 2024 CÁC CHUYÊN ĐỀ HAY VÀ KHÓ, 20 ĐỀ LUYỆN TẬP CÓ HƯỚNG DẪN CHI TIẾT.pdf
CHINH PHỤC KÌ THI VÀO LỚP 10 THPT MÔN TOÁN NĂM 2024 CÁC CHUYÊN ĐỀ HAY VÀ KHÓ, 20 ĐỀ LUYỆN TẬP CÓ HƯỚNG DẪN CHI TIẾT.pdf
CHINH PHỤC KÌ THI VÀO LỚP 10 THPT MÔN TOÁN NĂM 2024 CÁC CHUYÊN ĐỀ HAY VÀ KHÓ, 20 ĐỀ LUYỆN TẬP CÓ HƯỚNG DẪN CHI TIẾT.pdf
CHINH PHỤC KÌ THI VÀO LỚP 10 THPT MÔN TOÁN NĂM 2024 CÁC CHUYÊN ĐỀ HAY VÀ KHÓ, 20 ĐỀ LUYỆN TẬP CÓ HƯỚNG DẪN CHI TIẾT.pdf
CHINH PHỤC KÌ THI VÀO LỚP 10 THPT MÔN TOÁN NĂM 2024 CÁC CHUYÊN ĐỀ HAY VÀ KHÓ, 20 ĐỀ LUYỆN TẬP CÓ HƯỚNG DẪN CHI TIẾT.pdf
CHINH PHỤC KÌ THI VÀO LỚP 10 THPT MÔN TOÁN NĂM 2024 CÁC CHUYÊN ĐỀ HAY VÀ KHÓ, 20 ĐỀ LUYỆN TẬP CÓ HƯỚNG DẪN CHI TIẾT.pdf
CHINH PHỤC KÌ THI VÀO LỚP 10 THPT MÔN TOÁN NĂM 2024 CÁC CHUYÊN ĐỀ HAY VÀ KHÓ, 20 ĐỀ LUYỆN TẬP CÓ HƯỚNG DẪN CHI TIẾT.pdf
CHINH PHỤC KÌ THI VÀO LỚP 10 THPT MÔN TOÁN NĂM 2024 CÁC CHUYÊN ĐỀ HAY VÀ KHÓ, 20 ĐỀ LUYỆN TẬP CÓ HƯỚNG DẪN CHI TIẾT.pdf
CHINH PHỤC KÌ THI VÀO LỚP 10 THPT MÔN TOÁN NĂM 2024 CÁC CHUYÊN ĐỀ HAY VÀ KHÓ, 20 ĐỀ LUYỆN TẬP CÓ HƯỚNG DẪN CHI TIẾT.pdf
CHINH PHỤC KÌ THI VÀO LỚP 10 THPT MÔN TOÁN NĂM 2024 CÁC CHUYÊN ĐỀ HAY VÀ KHÓ, 20 ĐỀ LUYỆN TẬP CÓ HƯỚNG DẪN CHI TIẾT.pdf
CHINH PHỤC KÌ THI VÀO LỚP 10 THPT MÔN TOÁN NĂM 2024 CÁC CHUYÊN ĐỀ HAY VÀ KHÓ, 20 ĐỀ LUYỆN TẬP CÓ HƯỚNG DẪN CHI TIẾT.pdf
CHINH PHỤC KÌ THI VÀO LỚP 10 THPT MÔN TOÁN NĂM 2024 CÁC CHUYÊN ĐỀ HAY VÀ KHÓ, 20 ĐỀ LUYỆN TẬP CÓ HƯỚNG DẪN CHI TIẾT.pdf
CHINH PHỤC KÌ THI VÀO LỚP 10 THPT MÔN TOÁN NĂM 2024 CÁC CHUYÊN ĐỀ HAY VÀ KHÓ, 20 ĐỀ LUYỆN TẬP CÓ HƯỚNG DẪN CHI TIẾT.pdf
CHINH PHỤC KÌ THI VÀO LỚP 10 THPT MÔN TOÁN NĂM 2024 CÁC CHUYÊN ĐỀ HAY VÀ KHÓ, 20 ĐỀ LUYỆN TẬP CÓ HƯỚNG DẪN CHI TIẾT.pdf
CHINH PHỤC KÌ THI VÀO LỚP 10 THPT MÔN TOÁN NĂM 2024 CÁC CHUYÊN ĐỀ HAY VÀ KHÓ, 20 ĐỀ LUYỆN TẬP CÓ HƯỚNG DẪN CHI TIẾT.pdf
CHINH PHỤC KÌ THI VÀO LỚP 10 THPT MÔN TOÁN NĂM 2024 CÁC CHUYÊN ĐỀ HAY VÀ KHÓ, 20 ĐỀ LUYỆN TẬP CÓ HƯỚNG DẪN CHI TIẾT.pdf
CHINH PHỤC KÌ THI VÀO LỚP 10 THPT MÔN TOÁN NĂM 2024 CÁC CHUYÊN ĐỀ HAY VÀ KHÓ, 20 ĐỀ LUYỆN TẬP CÓ HƯỚNG DẪN CHI TIẾT.pdf
CHINH PHỤC KÌ THI VÀO LỚP 10 THPT MÔN TOÁN NĂM 2024 CÁC CHUYÊN ĐỀ HAY VÀ KHÓ, 20 ĐỀ LUYỆN TẬP CÓ HƯỚNG DẪN CHI TIẾT.pdf
CHINH PHỤC KÌ THI VÀO LỚP 10 THPT MÔN TOÁN NĂM 2024 CÁC CHUYÊN ĐỀ HAY VÀ KHÓ, 20 ĐỀ LUYỆN TẬP CÓ HƯỚNG DẪN CHI TIẾT.pdf
CHINH PHỤC KÌ THI VÀO LỚP 10 THPT MÔN TOÁN NĂM 2024 CÁC CHUYÊN ĐỀ HAY VÀ KHÓ, 20 ĐỀ LUYỆN TẬP CÓ HƯỚNG DẪN CHI TIẾT.pdf
CHINH PHỤC KÌ THI VÀO LỚP 10 THPT MÔN TOÁN NĂM 2024 CÁC CHUYÊN ĐỀ HAY VÀ KHÓ, 20 ĐỀ LUYỆN TẬP CÓ HƯỚNG DẪN CHI TIẾT.pdf
CHINH PHỤC KÌ THI VÀO LỚP 10 THPT MÔN TOÁN NĂM 2024 CÁC CHUYÊN ĐỀ HAY VÀ KHÓ, 20 ĐỀ LUYỆN TẬP CÓ HƯỚNG DẪN CHI TIẾT.pdf
CHINH PHỤC KÌ THI VÀO LỚP 10 THPT MÔN TOÁN NĂM 2024 CÁC CHUYÊN ĐỀ HAY VÀ KHÓ, 20 ĐỀ LUYỆN TẬP CÓ HƯỚNG DẪN CHI TIẾT.pdf
CHINH PHỤC KÌ THI VÀO LỚP 10 THPT MÔN TOÁN NĂM 2024 CÁC CHUYÊN ĐỀ HAY VÀ KHÓ, 20 ĐỀ LUYỆN TẬP CÓ HƯỚNG DẪN CHI TIẾT.pdf
CHINH PHỤC KÌ THI VÀO LỚP 10 THPT MÔN TOÁN NĂM 2024 CÁC CHUYÊN ĐỀ HAY VÀ KHÓ, 20 ĐỀ LUYỆN TẬP CÓ HƯỚNG DẪN CHI TIẾT.pdf
CHINH PHỤC KÌ THI VÀO LỚP 10 THPT MÔN TOÁN NĂM 2024 CÁC CHUYÊN ĐỀ HAY VÀ KHÓ, 20 ĐỀ LUYỆN TẬP CÓ HƯỚNG DẪN CHI TIẾT.pdf
CHINH PHỤC KÌ THI VÀO LỚP 10 THPT MÔN TOÁN NĂM 2024 CÁC CHUYÊN ĐỀ HAY VÀ KHÓ, 20 ĐỀ LUYỆN TẬP CÓ HƯỚNG DẪN CHI TIẾT.pdf
CHINH PHỤC KÌ THI VÀO LỚP 10 THPT MÔN TOÁN NĂM 2024 CÁC CHUYÊN ĐỀ HAY VÀ KHÓ, 20 ĐỀ LUYỆN TẬP CÓ HƯỚNG DẪN CHI TIẾT.pdf
CHINH PHỤC KÌ THI VÀO LỚP 10 THPT MÔN TOÁN NĂM 2024 CÁC CHUYÊN ĐỀ HAY VÀ KHÓ, 20 ĐỀ LUYỆN TẬP CÓ HƯỚNG DẪN CHI TIẾT.pdf
CHINH PHỤC KÌ THI VÀO LỚP 10 THPT MÔN TOÁN NĂM 2024 CÁC CHUYÊN ĐỀ HAY VÀ KHÓ, 20 ĐỀ LUYỆN TẬP CÓ HƯỚNG DẪN CHI TIẾT.pdf
CHINH PHỤC KÌ THI VÀO LỚP 10 THPT MÔN TOÁN NĂM 2024 CÁC CHUYÊN ĐỀ HAY VÀ KHÓ, 20 ĐỀ LUYỆN TẬP CÓ HƯỚNG DẪN CHI TIẾT.pdf
CHINH PHỤC KÌ THI VÀO LỚP 10 THPT MÔN TOÁN NĂM 2024 CÁC CHUYÊN ĐỀ HAY VÀ KHÓ, 20 ĐỀ LUYỆN TẬP CÓ HƯỚNG DẪN CHI TIẾT.pdf
CHINH PHỤC KÌ THI VÀO LỚP 10 THPT MÔN TOÁN NĂM 2024 CÁC CHUYÊN ĐỀ HAY VÀ KHÓ, 20 ĐỀ LUYỆN TẬP CÓ HƯỚNG DẪN CHI TIẾT.pdf
CHINH PHỤC KÌ THI VÀO LỚP 10 THPT MÔN TOÁN NĂM 2024 CÁC CHUYÊN ĐỀ HAY VÀ KHÓ, 20 ĐỀ LUYỆN TẬP CÓ HƯỚNG DẪN CHI TIẾT.pdf
CHINH PHỤC KÌ THI VÀO LỚP 10 THPT MÔN TOÁN NĂM 2024 CÁC CHUYÊN ĐỀ HAY VÀ KHÓ, 20 ĐỀ LUYỆN TẬP CÓ HƯỚNG DẪN CHI TIẾT.pdf
CHINH PHỤC KÌ THI VÀO LỚP 10 THPT MÔN TOÁN NĂM 2024 CÁC CHUYÊN ĐỀ HAY VÀ KHÓ, 20 ĐỀ LUYỆN TẬP CÓ HƯỚNG DẪN CHI TIẾT.pdf
CHINH PHỤC KÌ THI VÀO LỚP 10 THPT MÔN TOÁN NĂM 2024 CÁC CHUYÊN ĐỀ HAY VÀ KHÓ, 20 ĐỀ LUYỆN TẬP CÓ HƯỚNG DẪN CHI TIẾT.pdf
CHINH PHỤC KÌ THI VÀO LỚP 10 THPT MÔN TOÁN NĂM 2024 CÁC CHUYÊN ĐỀ HAY VÀ KHÓ, 20 ĐỀ LUYỆN TẬP CÓ HƯỚNG DẪN CHI TIẾT.pdf
CHINH PHỤC KÌ THI VÀO LỚP 10 THPT MÔN TOÁN NĂM 2024 CÁC CHUYÊN ĐỀ HAY VÀ KHÓ, 20 ĐỀ LUYỆN TẬP CÓ HƯỚNG DẪN CHI TIẾT.pdf
CHINH PHỤC KÌ THI VÀO LỚP 10 THPT MÔN TOÁN NĂM 2024 CÁC CHUYÊN ĐỀ HAY VÀ KHÓ, 20 ĐỀ LUYỆN TẬP CÓ HƯỚNG DẪN CHI TIẾT.pdf
CHINH PHỤC KÌ THI VÀO LỚP 10 THPT MÔN TOÁN NĂM 2024 CÁC CHUYÊN ĐỀ HAY VÀ KHÓ, 20 ĐỀ LUYỆN TẬP CÓ HƯỚNG DẪN CHI TIẾT.pdf
CHINH PHỤC KÌ THI VÀO LỚP 10 THPT MÔN TOÁN NĂM 2024 CÁC CHUYÊN ĐỀ HAY VÀ KHÓ, 20 ĐỀ LUYỆN TẬP CÓ HƯỚNG DẪN CHI TIẾT.pdf
CHINH PHỤC KÌ THI VÀO LỚP 10 THPT MÔN TOÁN NĂM 2024 CÁC CHUYÊN ĐỀ HAY VÀ KHÓ, 20 ĐỀ LUYỆN TẬP CÓ HƯỚNG DẪN CHI TIẾT.pdf
CHINH PHỤC KÌ THI VÀO LỚP 10 THPT MÔN TOÁN NĂM 2024 CÁC CHUYÊN ĐỀ HAY VÀ KHÓ, 20 ĐỀ LUYỆN TẬP CÓ HƯỚNG DẪN CHI TIẾT.pdf
CHINH PHỤC KÌ THI VÀO LỚP 10 THPT MÔN TOÁN NĂM 2024 CÁC CHUYÊN ĐỀ HAY VÀ KHÓ, 20 ĐỀ LUYỆN TẬP CÓ HƯỚNG DẪN CHI TIẾT.pdf
CHINH PHỤC KÌ THI VÀO LỚP 10 THPT MÔN TOÁN NĂM 2024 CÁC CHUYÊN ĐỀ HAY VÀ KHÓ, 20 ĐỀ LUYỆN TẬP CÓ HƯỚNG DẪN CHI TIẾT.pdf
CHINH PHỤC KÌ THI VÀO LỚP 10 THPT MÔN TOÁN NĂM 2024 CÁC CHUYÊN ĐỀ HAY VÀ KHÓ, 20 ĐỀ LUYỆN TẬP CÓ HƯỚNG DẪN CHI TIẾT.pdf
CHINH PHỤC KÌ THI VÀO LỚP 10 THPT MÔN TOÁN NĂM 2024 CÁC CHUYÊN ĐỀ HAY VÀ KHÓ, 20 ĐỀ LUYỆN TẬP CÓ HƯỚNG DẪN CHI TIẾT.pdf
CHINH PHỤC KÌ THI VÀO LỚP 10 THPT MÔN TOÁN NĂM 2024 CÁC CHUYÊN ĐỀ HAY VÀ KHÓ, 20 ĐỀ LUYỆN TẬP CÓ HƯỚNG DẪN CHI TIẾT.pdf
CHINH PHỤC KÌ THI VÀO LỚP 10 THPT MÔN TOÁN NĂM 2024 CÁC CHUYÊN ĐỀ HAY VÀ KHÓ, 20 ĐỀ LUYỆN TẬP CÓ HƯỚNG DẪN CHI TIẾT.pdf
CHINH PHỤC KÌ THI VÀO LỚP 10 THPT MÔN TOÁN NĂM 2024 CÁC CHUYÊN ĐỀ HAY VÀ KHÓ, 20 ĐỀ LUYỆN TẬP CÓ HƯỚNG DẪN CHI TIẾT.pdf
CHINH PHỤC KÌ THI VÀO LỚP 10 THPT MÔN TOÁN NĂM 2024 CÁC CHUYÊN ĐỀ HAY VÀ KHÓ, 20 ĐỀ LUYỆN TẬP CÓ HƯỚNG DẪN CHI TIẾT.pdf
CHINH PHỤC KÌ THI VÀO LỚP 10 THPT MÔN TOÁN NĂM 2024 CÁC CHUYÊN ĐỀ HAY VÀ KHÓ, 20 ĐỀ LUYỆN TẬP CÓ HƯỚNG DẪN CHI TIẾT.pdf
CHINH PHỤC KÌ THI VÀO LỚP 10 THPT MÔN TOÁN NĂM 2024 CÁC CHUYÊN ĐỀ HAY VÀ KHÓ, 20 ĐỀ LUYỆN TẬP CÓ HƯỚNG DẪN CHI TIẾT.pdf
CHINH PHỤC KÌ THI VÀO LỚP 10 THPT MÔN TOÁN NĂM 2024 CÁC CHUYÊN ĐỀ HAY VÀ KHÓ, 20 ĐỀ LUYỆN TẬP CÓ HƯỚNG DẪN CHI TIẾT.pdf
CHINH PHỤC KÌ THI VÀO LỚP 10 THPT MÔN TOÁN NĂM 2024 CÁC CHUYÊN ĐỀ HAY VÀ KHÓ, 20 ĐỀ LUYỆN TẬP CÓ HƯỚNG DẪN CHI TIẾT.pdf
CHINH PHỤC KÌ THI VÀO LỚP 10 THPT MÔN TOÁN NĂM 2024 CÁC CHUYÊN ĐỀ HAY VÀ KHÓ, 20 ĐỀ LUYỆN TẬP CÓ HƯỚNG DẪN CHI TIẾT.pdf
CHINH PHỤC KÌ THI VÀO LỚP 10 THPT MÔN TOÁN NĂM 2024 CÁC CHUYÊN ĐỀ HAY VÀ KHÓ, 20 ĐỀ LUYỆN TẬP CÓ HƯỚNG DẪN CHI TIẾT.pdf
CHINH PHỤC KÌ THI VÀO LỚP 10 THPT MÔN TOÁN NĂM 2024 CÁC CHUYÊN ĐỀ HAY VÀ KHÓ, 20 ĐỀ LUYỆN TẬP CÓ HƯỚNG DẪN CHI TIẾT.pdf
CHINH PHỤC KÌ THI VÀO LỚP 10 THPT MÔN TOÁN NĂM 2024 CÁC CHUYÊN ĐỀ HAY VÀ KHÓ, 20 ĐỀ LUYỆN TẬP CÓ HƯỚNG DẪN CHI TIẾT.pdf
CHINH PHỤC KÌ THI VÀO LỚP 10 THPT MÔN TOÁN NĂM 2024 CÁC CHUYÊN ĐỀ HAY VÀ KHÓ, 20 ĐỀ LUYỆN TẬP CÓ HƯỚNG DẪN CHI TIẾT.pdf
CHINH PHỤC KÌ THI VÀO LỚP 10 THPT MÔN TOÁN NĂM 2024 CÁC CHUYÊN ĐỀ HAY VÀ KHÓ, 20 ĐỀ LUYỆN TẬP CÓ HƯỚNG DẪN CHI TIẾT.pdf
CHINH PHỤC KÌ THI VÀO LỚP 10 THPT MÔN TOÁN NĂM 2024 CÁC CHUYÊN ĐỀ HAY VÀ KHÓ, 20 ĐỀ LUYỆN TẬP CÓ HƯỚNG DẪN CHI TIẾT.pdf
CHINH PHỤC KÌ THI VÀO LỚP 10 THPT MÔN TOÁN NĂM 2024 CÁC CHUYÊN ĐỀ HAY VÀ KHÓ, 20 ĐỀ LUYỆN TẬP CÓ HƯỚNG DẪN CHI TIẾT.pdf
CHINH PHỤC KÌ THI VÀO LỚP 10 THPT MÔN TOÁN NĂM 2024 CÁC CHUYÊN ĐỀ HAY VÀ KHÓ, 20 ĐỀ LUYỆN TẬP CÓ HƯỚNG DẪN CHI TIẾT.pdf
CHINH PHỤC KÌ THI VÀO LỚP 10 THPT MÔN TOÁN NĂM 2024 CÁC CHUYÊN ĐỀ HAY VÀ KHÓ, 20 ĐỀ LUYỆN TẬP CÓ HƯỚNG DẪN CHI TIẾT.pdf
CHINH PHỤC KÌ THI VÀO LỚP 10 THPT MÔN TOÁN NĂM 2024 CÁC CHUYÊN ĐỀ HAY VÀ KHÓ, 20 ĐỀ LUYỆN TẬP CÓ HƯỚNG DẪN CHI TIẾT.pdf
CHINH PHỤC KÌ THI VÀO LỚP 10 THPT MÔN TOÁN NĂM 2024 CÁC CHUYÊN ĐỀ HAY VÀ KHÓ, 20 ĐỀ LUYỆN TẬP CÓ HƯỚNG DẪN CHI TIẾT.pdf
CHINH PHỤC KÌ THI VÀO LỚP 10 THPT MÔN TOÁN NĂM 2024 CÁC CHUYÊN ĐỀ HAY VÀ KHÓ, 20 ĐỀ LUYỆN TẬP CÓ HƯỚNG DẪN CHI TIẾT.pdf
CHINH PHỤC KÌ THI VÀO LỚP 10 THPT MÔN TOÁN NĂM 2024 CÁC CHUYÊN ĐỀ HAY VÀ KHÓ, 20 ĐỀ LUYỆN TẬP CÓ HƯỚNG DẪN CHI TIẾT.pdf
CHINH PHỤC KÌ THI VÀO LỚP 10 THPT MÔN TOÁN NĂM 2024 CÁC CHUYÊN ĐỀ HAY VÀ KHÓ, 20 ĐỀ LUYỆN TẬP CÓ HƯỚNG DẪN CHI TIẾT.pdf
CHINH PHỤC KÌ THI VÀO LỚP 10 THPT MÔN TOÁN NĂM 2024 CÁC CHUYÊN ĐỀ HAY VÀ KHÓ, 20 ĐỀ LUYỆN TẬP CÓ HƯỚNG DẪN CHI TIẾT.pdf
CHINH PHỤC KÌ THI VÀO LỚP 10 THPT MÔN TOÁN NĂM 2024 CÁC CHUYÊN ĐỀ HAY VÀ KHÓ, 20 ĐỀ LUYỆN TẬP CÓ HƯỚNG DẪN CHI TIẾT.pdf
CHINH PHỤC KÌ THI VÀO LỚP 10 THPT MÔN TOÁN NĂM 2024 CÁC CHUYÊN ĐỀ HAY VÀ KHÓ, 20 ĐỀ LUYỆN TẬP CÓ HƯỚNG DẪN CHI TIẾT.pdf
CHINH PHỤC KÌ THI VÀO LỚP 10 THPT MÔN TOÁN NĂM 2024 CÁC CHUYÊN ĐỀ HAY VÀ KHÓ, 20 ĐỀ LUYỆN TẬP CÓ HƯỚNG DẪN CHI TIẾT.pdf
CHINH PHỤC KÌ THI VÀO LỚP 10 THPT MÔN TOÁN NĂM 2024 CÁC CHUYÊN ĐỀ HAY VÀ KHÓ, 20 ĐỀ LUYỆN TẬP CÓ HƯỚNG DẪN CHI TIẾT.pdf
CHINH PHỤC KÌ THI VÀO LỚP 10 THPT MÔN TOÁN NĂM 2024 CÁC CHUYÊN ĐỀ HAY VÀ KHÓ, 20 ĐỀ LUYỆN TẬP CÓ HƯỚNG DẪN CHI TIẾT.pdf
CHINH PHỤC KÌ THI VÀO LỚP 10 THPT MÔN TOÁN NĂM 2024 CÁC CHUYÊN ĐỀ HAY VÀ KHÓ, 20 ĐỀ LUYỆN TẬP CÓ HƯỚNG DẪN CHI TIẾT.pdf
CHINH PHỤC KÌ THI VÀO LỚP 10 THPT MÔN TOÁN NĂM 2024 CÁC CHUYÊN ĐỀ HAY VÀ KHÓ, 20 ĐỀ LUYỆN TẬP CÓ HƯỚNG DẪN CHI TIẾT.pdf
CHINH PHỤC KÌ THI VÀO LỚP 10 THPT MÔN TOÁN NĂM 2024 CÁC CHUYÊN ĐỀ HAY VÀ KHÓ, 20 ĐỀ LUYỆN TẬP CÓ HƯỚNG DẪN CHI TIẾT.pdf
CHINH PHỤC KÌ THI VÀO LỚP 10 THPT MÔN TOÁN NĂM 2024 CÁC CHUYÊN ĐỀ HAY VÀ KHÓ, 20 ĐỀ LUYỆN TẬP CÓ HƯỚNG DẪN CHI TIẾT.pdf
CHINH PHỤC KÌ THI VÀO LỚP 10 THPT MÔN TOÁN NĂM 2024 CÁC CHUYÊN ĐỀ HAY VÀ KHÓ, 20 ĐỀ LUYỆN TẬP CÓ HƯỚNG DẪN CHI TIẾT.pdf
CHINH PHỤC KÌ THI VÀO LỚP 10 THPT MÔN TOÁN NĂM 2024 CÁC CHUYÊN ĐỀ HAY VÀ KHÓ, 20 ĐỀ LUYỆN TẬP CÓ HƯỚNG DẪN CHI TIẾT.pdf
CHINH PHỤC KÌ THI VÀO LỚP 10 THPT MÔN TOÁN NĂM 2024 CÁC CHUYÊN ĐỀ HAY VÀ KHÓ, 20 ĐỀ LUYỆN TẬP CÓ HƯỚNG DẪN CHI TIẾT.pdf
CHINH PHỤC KÌ THI VÀO LỚP 10 THPT MÔN TOÁN NĂM 2024 CÁC CHUYÊN ĐỀ HAY VÀ KHÓ, 20 ĐỀ LUYỆN TẬP CÓ HƯỚNG DẪN CHI TIẾT.pdf
CHINH PHỤC KÌ THI VÀO LỚP 10 THPT MÔN TOÁN NĂM 2024 CÁC CHUYÊN ĐỀ HAY VÀ KHÓ, 20 ĐỀ LUYỆN TẬP CÓ HƯỚNG DẪN CHI TIẾT.pdf
CHINH PHỤC KÌ THI VÀO LỚP 10 THPT MÔN TOÁN NĂM 2024 CÁC CHUYÊN ĐỀ HAY VÀ KHÓ, 20 ĐỀ LUYỆN TẬP CÓ HƯỚNG DẪN CHI TIẾT.pdf
CHINH PHỤC KÌ THI VÀO LỚP 10 THPT MÔN TOÁN NĂM 2024 CÁC CHUYÊN ĐỀ HAY VÀ KHÓ, 20 ĐỀ LUYỆN TẬP CÓ HƯỚNG DẪN CHI TIẾT.pdf
CHINH PHỤC KÌ THI VÀO LỚP 10 THPT MÔN TOÁN NĂM 2024 CÁC CHUYÊN ĐỀ HAY VÀ KHÓ, 20 ĐỀ LUYỆN TẬP CÓ HƯỚNG DẪN CHI TIẾT.pdf
CHINH PHỤC KÌ THI VÀO LỚP 10 THPT MÔN TOÁN NĂM 2024 CÁC CHUYÊN ĐỀ HAY VÀ KHÓ, 20 ĐỀ LUYỆN TẬP CÓ HƯỚNG DẪN CHI TIẾT.pdf
CHINH PHỤC KÌ THI VÀO LỚP 10 THPT MÔN TOÁN NĂM 2024 CÁC CHUYÊN ĐỀ HAY VÀ KHÓ, 20 ĐỀ LUYỆN TẬP CÓ HƯỚNG DẪN CHI TIẾT.pdf
CHINH PHỤC KÌ THI VÀO LỚP 10 THPT MÔN TOÁN NĂM 2024 CÁC CHUYÊN ĐỀ HAY VÀ KHÓ, 20 ĐỀ LUYỆN TẬP CÓ HƯỚNG DẪN CHI TIẾT.pdf
CHINH PHỤC KÌ THI VÀO LỚP 10 THPT MÔN TOÁN NĂM 2024 CÁC CHUYÊN ĐỀ HAY VÀ KHÓ, 20 ĐỀ LUYỆN TẬP CÓ HƯỚNG DẪN CHI TIẾT.pdf
CHINH PHỤC KÌ THI VÀO LỚP 10 THPT MÔN TOÁN NĂM 2024 CÁC CHUYÊN ĐỀ HAY VÀ KHÓ, 20 ĐỀ LUYỆN TẬP CÓ HƯỚNG DẪN CHI TIẾT.pdf
CHINH PHỤC KÌ THI VÀO LỚP 10 THPT MÔN TOÁN NĂM 2024 CÁC CHUYÊN ĐỀ HAY VÀ KHÓ, 20 ĐỀ LUYỆN TẬP CÓ HƯỚNG DẪN CHI TIẾT.pdf
CHINH PHỤC KÌ THI VÀO LỚP 10 THPT MÔN TOÁN NĂM 2024 CÁC CHUYÊN ĐỀ HAY VÀ KHÓ, 20 ĐỀ LUYỆN TẬP CÓ HƯỚNG DẪN CHI TIẾT.pdf
CHINH PHỤC KÌ THI VÀO LỚP 10 THPT MÔN TOÁN NĂM 2024 CÁC CHUYÊN ĐỀ HAY VÀ KHÓ, 20 ĐỀ LUYỆN TẬP CÓ HƯỚNG DẪN CHI TIẾT.pdf
CHINH PHỤC KÌ THI VÀO LỚP 10 THPT MÔN TOÁN NĂM 2024 CÁC CHUYÊN ĐỀ HAY VÀ KHÓ, 20 ĐỀ LUYỆN TẬP CÓ HƯỚNG DẪN CHI TIẾT.pdf
CHINH PHỤC KÌ THI VÀO LỚP 10 THPT MÔN TOÁN NĂM 2024 CÁC CHUYÊN ĐỀ HAY VÀ KHÓ, 20 ĐỀ LUYỆN TẬP CÓ HƯỚNG DẪN CHI TIẾT.pdf
CHINH PHỤC KÌ THI VÀO LỚP 10 THPT MÔN TOÁN NĂM 2024 CÁC CHUYÊN ĐỀ HAY VÀ KHÓ, 20 ĐỀ LUYỆN TẬP CÓ HƯỚNG DẪN CHI TIẾT.pdf
CHINH PHỤC KÌ THI VÀO LỚP 10 THPT MÔN TOÁN NĂM 2024 CÁC CHUYÊN ĐỀ HAY VÀ KHÓ, 20 ĐỀ LUYỆN TẬP CÓ HƯỚNG DẪN CHI TIẾT.pdf
CHINH PHỤC KÌ THI VÀO LỚP 10 THPT MÔN TOÁN NĂM 2024 CÁC CHUYÊN ĐỀ HAY VÀ KHÓ, 20 ĐỀ LUYỆN TẬP CÓ HƯỚNG DẪN CHI TIẾT.pdf
CHINH PHỤC KÌ THI VÀO LỚP 10 THPT MÔN TOÁN NĂM 2024 CÁC CHUYÊN ĐỀ HAY VÀ KHÓ, 20 ĐỀ LUYỆN TẬP CÓ HƯỚNG DẪN CHI TIẾT.pdf
CHINH PHỤC KÌ THI VÀO LỚP 10 THPT MÔN TOÁN NĂM 2024 CÁC CHUYÊN ĐỀ HAY VÀ KHÓ, 20 ĐỀ LUYỆN TẬP CÓ HƯỚNG DẪN CHI TIẾT.pdf
CHINH PHỤC KÌ THI VÀO LỚP 10 THPT MÔN TOÁN NĂM 2024 CÁC CHUYÊN ĐỀ HAY VÀ KHÓ, 20 ĐỀ LUYỆN TẬP CÓ HƯỚNG DẪN CHI TIẾT.pdf
CHINH PHỤC KÌ THI VÀO LỚP 10 THPT MÔN TOÁN NĂM 2024 CÁC CHUYÊN ĐỀ HAY VÀ KHÓ, 20 ĐỀ LUYỆN TẬP CÓ HƯỚNG DẪN CHI TIẾT.pdf
CHINH PHỤC KÌ THI VÀO LỚP 10 THPT MÔN TOÁN NĂM 2024 CÁC CHUYÊN ĐỀ HAY VÀ KHÓ, 20 ĐỀ LUYỆN TẬP CÓ HƯỚNG DẪN CHI TIẾT.pdf
CHINH PHỤC KÌ THI VÀO LỚP 10 THPT MÔN TOÁN NĂM 2024 CÁC CHUYÊN ĐỀ HAY VÀ KHÓ, 20 ĐỀ LUYỆN TẬP CÓ HƯỚNG DẪN CHI TIẾT.pdf
CHINH PHỤC KÌ THI VÀO LỚP 10 THPT MÔN TOÁN NĂM 2024 CÁC CHUYÊN ĐỀ HAY VÀ KHÓ, 20 ĐỀ LUYỆN TẬP CÓ HƯỚNG DẪN CHI TIẾT.pdf
CHINH PHỤC KÌ THI VÀO LỚP 10 THPT MÔN TOÁN NĂM 2024 CÁC CHUYÊN ĐỀ HAY VÀ KHÓ, 20 ĐỀ LUYỆN TẬP CÓ HƯỚNG DẪN CHI TIẾT.pdf
CHINH PHỤC KÌ THI VÀO LỚP 10 THPT MÔN TOÁN NĂM 2024 CÁC CHUYÊN ĐỀ HAY VÀ KHÓ, 20 ĐỀ LUYỆN TẬP CÓ HƯỚNG DẪN CHI TIẾT.pdf
CHINH PHỤC KÌ THI VÀO LỚP 10 THPT MÔN TOÁN NĂM 2024 CÁC CHUYÊN ĐỀ HAY VÀ KHÓ, 20 ĐỀ LUYỆN TẬP CÓ HƯỚNG DẪN CHI TIẾT.pdf
CHINH PHỤC KÌ THI VÀO LỚP 10 THPT MÔN TOÁN NĂM 2024 CÁC CHUYÊN ĐỀ HAY VÀ KHÓ, 20 ĐỀ LUYỆN TẬP CÓ HƯỚNG DẪN CHI TIẾT.pdf
CHINH PHỤC KÌ THI VÀO LỚP 10 THPT MÔN TOÁN NĂM 2024 CÁC CHUYÊN ĐỀ HAY VÀ KHÓ, 20 ĐỀ LUYỆN TẬP CÓ HƯỚNG DẪN CHI TIẾT.pdf
CHINH PHỤC KÌ THI VÀO LỚP 10 THPT MÔN TOÁN NĂM 2024 CÁC CHUYÊN ĐỀ HAY VÀ KHÓ, 20 ĐỀ LUYỆN TẬP CÓ HƯỚNG DẪN CHI TIẾT.pdf
CHINH PHỤC KÌ THI VÀO LỚP 10 THPT MÔN TOÁN NĂM 2024 CÁC CHUYÊN ĐỀ HAY VÀ KHÓ, 20 ĐỀ LUYỆN TẬP CÓ HƯỚNG DẪN CHI TIẾT.pdf
CHINH PHỤC KÌ THI VÀO LỚP 10 THPT MÔN TOÁN NĂM 2024 CÁC CHUYÊN ĐỀ HAY VÀ KHÓ, 20 ĐỀ LUYỆN TẬP CÓ HƯỚNG DẪN CHI TIẾT.pdf
CHINH PHỤC KÌ THI VÀO LỚP 10 THPT MÔN TOÁN NĂM 2024 CÁC CHUYÊN ĐỀ HAY VÀ KHÓ, 20 ĐỀ LUYỆN TẬP CÓ HƯỚNG DẪN CHI TIẾT.pdf
CHINH PHỤC KÌ THI VÀO LỚP 10 THPT MÔN TOÁN NĂM 2024 CÁC CHUYÊN ĐỀ HAY VÀ KHÓ, 20 ĐỀ LUYỆN TẬP CÓ HƯỚNG DẪN CHI TIẾT.pdf
CHINH PHỤC KÌ THI VÀO LỚP 10 THPT MÔN TOÁN NĂM 2024 CÁC CHUYÊN ĐỀ HAY VÀ KHÓ, 20 ĐỀ LUYỆN TẬP CÓ HƯỚNG DẪN CHI TIẾT.pdf
CHINH PHỤC KÌ THI VÀO LỚP 10 THPT MÔN TOÁN NĂM 2024 CÁC CHUYÊN ĐỀ HAY VÀ KHÓ, 20 ĐỀ LUYỆN TẬP CÓ HƯỚNG DẪN CHI TIẾT.pdf
CHINH PHỤC KÌ THI VÀO LỚP 10 THPT MÔN TOÁN NĂM 2024 CÁC CHUYÊN ĐỀ HAY VÀ KHÓ, 20 ĐỀ LUYỆN TẬP CÓ HƯỚNG DẪN CHI TIẾT.pdf
CHINH PHỤC KÌ THI VÀO LỚP 10 THPT MÔN TOÁN NĂM 2024 CÁC CHUYÊN ĐỀ HAY VÀ KHÓ, 20 ĐỀ LUYỆN TẬP CÓ HƯỚNG DẪN CHI TIẾT.pdf
CHINH PHỤC KÌ THI VÀO LỚP 10 THPT MÔN TOÁN NĂM 2024 CÁC CHUYÊN ĐỀ HAY VÀ KHÓ, 20 ĐỀ LUYỆN TẬP CÓ HƯỚNG DẪN CHI TIẾT.pdf
CHINH PHỤC KÌ THI VÀO LỚP 10 THPT MÔN TOÁN NĂM 2024 CÁC CHUYÊN ĐỀ HAY VÀ KHÓ, 20 ĐỀ LUYỆN TẬP CÓ HƯỚNG DẪN CHI TIẾT.pdf
CHINH PHỤC KÌ THI VÀO LỚP 10 THPT MÔN TOÁN NĂM 2024 CÁC CHUYÊN ĐỀ HAY VÀ KHÓ, 20 ĐỀ LUYỆN TẬP CÓ HƯỚNG DẪN CHI TIẾT.pdf
CHINH PHỤC KÌ THI VÀO LỚP 10 THPT MÔN TOÁN NĂM 2024 CÁC CHUYÊN ĐỀ HAY VÀ KHÓ, 20 ĐỀ LUYỆN TẬP CÓ HƯỚNG DẪN CHI TIẾT.pdf
CHINH PHỤC KÌ THI VÀO LỚP 10 THPT MÔN TOÁN NĂM 2024 CÁC CHUYÊN ĐỀ HAY VÀ KHÓ, 20 ĐỀ LUYỆN TẬP CÓ HƯỚNG DẪN CHI TIẾT.pdf
CHINH PHỤC KÌ THI VÀO LỚP 10 THPT MÔN TOÁN NĂM 2024 CÁC CHUYÊN ĐỀ HAY VÀ KHÓ, 20 ĐỀ LUYỆN TẬP CÓ HƯỚNG DẪN CHI TIẾT.pdf
CHINH PHỤC KÌ THI VÀO LỚP 10 THPT MÔN TOÁN NĂM 2024 CÁC CHUYÊN ĐỀ HAY VÀ KHÓ, 20 ĐỀ LUYỆN TẬP CÓ HƯỚNG DẪN CHI TIẾT.pdf
CHINH PHỤC KÌ THI VÀO LỚP 10 THPT MÔN TOÁN NĂM 2024 CÁC CHUYÊN ĐỀ HAY VÀ KHÓ, 20 ĐỀ LUYỆN TẬP CÓ HƯỚNG DẪN CHI TIẾT.pdf
CHINH PHỤC KÌ THI VÀO LỚP 10 THPT MÔN TOÁN NĂM 2024 CÁC CHUYÊN ĐỀ HAY VÀ KHÓ, 20 ĐỀ LUYỆN TẬP CÓ HƯỚNG DẪN CHI TIẾT.pdf
CHINH PHỤC KÌ THI VÀO LỚP 10 THPT MÔN TOÁN NĂM 2024 CÁC CHUYÊN ĐỀ HAY VÀ KHÓ, 20 ĐỀ LUYỆN TẬP CÓ HƯỚNG DẪN CHI TIẾT.pdf
CHINH PHỤC KÌ THI VÀO LỚP 10 THPT MÔN TOÁN NĂM 2024 CÁC CHUYÊN ĐỀ HAY VÀ KHÓ, 20 ĐỀ LUYỆN TẬP CÓ HƯỚNG DẪN CHI TIẾT.pdf
CHINH PHỤC KÌ THI VÀO LỚP 10 THPT MÔN TOÁN NĂM 2024 CÁC CHUYÊN ĐỀ HAY VÀ KHÓ, 20 ĐỀ LUYỆN TẬP CÓ HƯỚNG DẪN CHI TIẾT.pdf
CHINH PHỤC KÌ THI VÀO LỚP 10 THPT MÔN TOÁN NĂM 2024 CÁC CHUYÊN ĐỀ HAY VÀ KHÓ, 20 ĐỀ LUYỆN TẬP CÓ HƯỚNG DẪN CHI TIẾT.pdf
CHINH PHỤC KÌ THI VÀO LỚP 10 THPT MÔN TOÁN NĂM 2024 CÁC CHUYÊN ĐỀ HAY VÀ KHÓ, 20 ĐỀ LUYỆN TẬP CÓ HƯỚNG DẪN CHI TIẾT.pdf
CHINH PHỤC KÌ THI VÀO LỚP 10 THPT MÔN TOÁN NĂM 2024 CÁC CHUYÊN ĐỀ HAY VÀ KHÓ, 20 ĐỀ LUYỆN TẬP CÓ HƯỚNG DẪN CHI TIẾT.pdf
CHINH PHỤC KÌ THI VÀO LỚP 10 THPT MÔN TOÁN NĂM 2024 CÁC CHUYÊN ĐỀ HAY VÀ KHÓ, 20 ĐỀ LUYỆN TẬP CÓ HƯỚNG DẪN CHI TIẾT.pdf
CHINH PHỤC KÌ THI VÀO LỚP 10 THPT MÔN TOÁN NĂM 2024 CÁC CHUYÊN ĐỀ HAY VÀ KHÓ, 20 ĐỀ LUYỆN TẬP CÓ HƯỚNG DẪN CHI TIẾT.pdf
CHINH PHỤC KÌ THI VÀO LỚP 10 THPT MÔN TOÁN NĂM 2024 CÁC CHUYÊN ĐỀ HAY VÀ KHÓ, 20 ĐỀ LUYỆN TẬP CÓ HƯỚNG DẪN CHI TIẾT.pdf
CHINH PHỤC KÌ THI VÀO LỚP 10 THPT MÔN TOÁN NĂM 2024 CÁC CHUYÊN ĐỀ HAY VÀ KHÓ, 20 ĐỀ LUYỆN TẬP CÓ HƯỚNG DẪN CHI TIẾT.pdf
CHINH PHỤC KÌ THI VÀO LỚP 10 THPT MÔN TOÁN NĂM 2024 CÁC CHUYÊN ĐỀ HAY VÀ KHÓ, 20 ĐỀ LUYỆN TẬP CÓ HƯỚNG DẪN CHI TIẾT.pdf
CHINH PHỤC KÌ THI VÀO LỚP 10 THPT MÔN TOÁN NĂM 2024 CÁC CHUYÊN ĐỀ HAY VÀ KHÓ, 20 ĐỀ LUYỆN TẬP CÓ HƯỚNG DẪN CHI TIẾT.pdf
CHINH PHỤC KÌ THI VÀO LỚP 10 THPT MÔN TOÁN NĂM 2024 CÁC CHUYÊN ĐỀ HAY VÀ KHÓ, 20 ĐỀ LUYỆN TẬP CÓ HƯỚNG DẪN CHI TIẾT.pdf
CHINH PHỤC KÌ THI VÀO LỚP 10 THPT MÔN TOÁN NĂM 2024 CÁC CHUYÊN ĐỀ HAY VÀ KHÓ, 20 ĐỀ LUYỆN TẬP CÓ HƯỚNG DẪN CHI TIẾT.pdf
CHINH PHỤC KÌ THI VÀO LỚP 10 THPT MÔN TOÁN NĂM 2024 CÁC CHUYÊN ĐỀ HAY VÀ KHÓ, 20 ĐỀ LUYỆN TẬP CÓ HƯỚNG DẪN CHI TIẾT.pdf
CHINH PHỤC KÌ THI VÀO LỚP 10 THPT MÔN TOÁN NĂM 2024 CÁC CHUYÊN ĐỀ HAY VÀ KHÓ, 20 ĐỀ LUYỆN TẬP CÓ HƯỚNG DẪN CHI TIẾT.pdf
CHINH PHỤC KÌ THI VÀO LỚP 10 THPT MÔN TOÁN NĂM 2024 CÁC CHUYÊN ĐỀ HAY VÀ KHÓ, 20 ĐỀ LUYỆN TẬP CÓ HƯỚNG DẪN CHI TIẾT.pdf
CHINH PHỤC KÌ THI VÀO LỚP 10 THPT MÔN TOÁN NĂM 2024 CÁC CHUYÊN ĐỀ HAY VÀ KHÓ, 20 ĐỀ LUYỆN TẬP CÓ HƯỚNG DẪN CHI TIẾT.pdf
CHINH PHỤC KÌ THI VÀO LỚP 10 THPT MÔN TOÁN NĂM 2024 CÁC CHUYÊN ĐỀ HAY VÀ KHÓ, 20 ĐỀ LUYỆN TẬP CÓ HƯỚNG DẪN CHI TIẾT.pdf
CHINH PHỤC KÌ THI VÀO LỚP 10 THPT MÔN TOÁN NĂM 2024 CÁC CHUYÊN ĐỀ HAY VÀ KHÓ, 20 ĐỀ LUYỆN TẬP CÓ HƯỚNG DẪN CHI TIẾT.pdf
CHINH PHỤC KÌ THI VÀO LỚP 10 THPT MÔN TOÁN NĂM 2024 CÁC CHUYÊN ĐỀ HAY VÀ KHÓ, 20 ĐỀ LUYỆN TẬP CÓ HƯỚNG DẪN CHI TIẾT.pdf

More Related Content

Similar to CHINH PHỤC KÌ THI VÀO LỚP 10 THPT MÔN TOÁN NĂM 2024 CÁC CHUYÊN ĐỀ HAY VÀ KHÓ, 20 ĐỀ LUYỆN TẬP CÓ HƯỚNG DẪN CHI TIẾT.pdf

04 phuong trinh mu p3
04 phuong trinh mu p304 phuong trinh mu p3
04 phuong trinh mu p3Huynh ICT
 
Bdt duythao
Bdt duythaoBdt duythao
Bdt duythao
phuonganhtran1303
 
04 phuong trinh mu p1
04 phuong trinh mu p104 phuong trinh mu p1
04 phuong trinh mu p1Huynh ICT
 
Sach pt tang hs online
Sach pt tang hs onlineSach pt tang hs online
Sach pt tang hs online
Quý Hoàng
 
Chukienthuc.com cach-tinh-tich-phan-vhquoc
Chukienthuc.com cach-tinh-tich-phan-vhquocChukienthuc.com cach-tinh-tich-phan-vhquoc
Chukienthuc.com cach-tinh-tich-phan-vhquocMarco Reus Le
 
Dap an4 thanhtung
Dap an4 thanhtungDap an4 thanhtung
Dap an4 thanhtungHuynh ICT
 
Phương pháp giải phương trình, bất phương trình mũ
Phương pháp giải phương trình, bất phương trình mũPhương pháp giải phương trình, bất phương trình mũ
Phương pháp giải phương trình, bất phương trình mũ
Linh Nguyễn
 
10 kithuatgiaiphuongtrinhvoti thanhtung
10 kithuatgiaiphuongtrinhvoti thanhtung10 kithuatgiaiphuongtrinhvoti thanhtung
10 kithuatgiaiphuongtrinhvoti thanhtungHuynh ICT
 
Chuyen de pt vo ti
Chuyen de pt vo tiChuyen de pt vo ti
Chuyen de pt vo ti
Vui Lên Bạn Nhé
 
Chuyen%20de%20phuong%20trinh%20nghiem%20nguyen
Chuyen%20de%20phuong%20trinh%20nghiem%20nguyenChuyen%20de%20phuong%20trinh%20nghiem%20nguyen
Chuyen%20de%20phuong%20trinh%20nghiem%20nguyenTam Vu Minh
 
Tập 2 chuyên đề Toán học: Phương trình vô tỷ - Megabook.vn
Tập 2 chuyên đề Toán học: Phương trình vô tỷ - Megabook.vnTập 2 chuyên đề Toán học: Phương trình vô tỷ - Megabook.vn
Tập 2 chuyên đề Toán học: Phương trình vô tỷ - Megabook.vn
Megabook
 
Phuong trinh he_phuong_trinh_vo_ti_551
Phuong trinh he_phuong_trinh_vo_ti_551Phuong trinh he_phuong_trinh_vo_ti_551
Phuong trinh he_phuong_trinh_vo_ti_551Cuong Archuleta
 
TUYỂN TẬP 23 CHUYÊN ĐỀ ĐẠI SỐ BỒI DƯỠNG HỌC SINH GIỎI TOÁN LỚP 9 - CÓ LỜI GIẢ...
TUYỂN TẬP 23 CHUYÊN ĐỀ ĐẠI SỐ BỒI DƯỠNG HỌC SINH GIỎI TOÁN LỚP 9 - CÓ LỜI GIẢ...TUYỂN TẬP 23 CHUYÊN ĐỀ ĐẠI SỐ BỒI DƯỠNG HỌC SINH GIỎI TOÁN LỚP 9 - CÓ LỜI GIẢ...
TUYỂN TẬP 23 CHUYÊN ĐỀ ĐẠI SỐ BỒI DƯỠNG HỌC SINH GIỎI TOÁN LỚP 9 - CÓ LỜI GIẢ...
Nguyen Thanh Tu Collection
 
Pp giai pt va hpt khong mau muc
Pp giai pt va hpt khong mau mucPp giai pt va hpt khong mau muc
Pp giai pt va hpt khong mau muc
keolac410
 
02 phuong phap dat an phu giai pt p2
02 phuong phap dat an phu giai pt p202 phuong phap dat an phu giai pt p2
02 phuong phap dat an phu giai pt p2Huynh ICT
 
Dethi hsg-l10-2013-ha tinh-toan
Dethi hsg-l10-2013-ha tinh-toanDethi hsg-l10-2013-ha tinh-toan
Dethi hsg-l10-2013-ha tinh-toan
Chàng Trai Khó Tính
 
06 ki thuat dong nhat tim nguyen ham
06 ki thuat dong nhat tim nguyen ham06 ki thuat dong nhat tim nguyen ham
06 ki thuat dong nhat tim nguyen hamHuynh ICT
 
52 bài hệ phương trình
52 bài hệ phương trình52 bài hệ phương trình
52 bài hệ phương trình
tuituhoc
 
Mot so chu y khi giai pt
Mot so chu y khi giai ptMot so chu y khi giai pt
Mot so chu y khi giai ptndphuc910
 

Similar to CHINH PHỤC KÌ THI VÀO LỚP 10 THPT MÔN TOÁN NĂM 2024 CÁC CHUYÊN ĐỀ HAY VÀ KHÓ, 20 ĐỀ LUYỆN TẬP CÓ HƯỚNG DẪN CHI TIẾT.pdf (20)

04 phuong trinh mu p3
04 phuong trinh mu p304 phuong trinh mu p3
04 phuong trinh mu p3
 
Bdt duythao
Bdt duythaoBdt duythao
Bdt duythao
 
04 phuong trinh mu p1
04 phuong trinh mu p104 phuong trinh mu p1
04 phuong trinh mu p1
 
Sach pt tang hs online
Sach pt tang hs onlineSach pt tang hs online
Sach pt tang hs online
 
18q5t5 o2
18q5t5 o218q5t5 o2
18q5t5 o2
 
Chukienthuc.com cach-tinh-tich-phan-vhquoc
Chukienthuc.com cach-tinh-tich-phan-vhquocChukienthuc.com cach-tinh-tich-phan-vhquoc
Chukienthuc.com cach-tinh-tich-phan-vhquoc
 
Dap an4 thanhtung
Dap an4 thanhtungDap an4 thanhtung
Dap an4 thanhtung
 
Phương pháp giải phương trình, bất phương trình mũ
Phương pháp giải phương trình, bất phương trình mũPhương pháp giải phương trình, bất phương trình mũ
Phương pháp giải phương trình, bất phương trình mũ
 
10 kithuatgiaiphuongtrinhvoti thanhtung
10 kithuatgiaiphuongtrinhvoti thanhtung10 kithuatgiaiphuongtrinhvoti thanhtung
10 kithuatgiaiphuongtrinhvoti thanhtung
 
Chuyen de pt vo ti
Chuyen de pt vo tiChuyen de pt vo ti
Chuyen de pt vo ti
 
Chuyen%20de%20phuong%20trinh%20nghiem%20nguyen
Chuyen%20de%20phuong%20trinh%20nghiem%20nguyenChuyen%20de%20phuong%20trinh%20nghiem%20nguyen
Chuyen%20de%20phuong%20trinh%20nghiem%20nguyen
 
Tập 2 chuyên đề Toán học: Phương trình vô tỷ - Megabook.vn
Tập 2 chuyên đề Toán học: Phương trình vô tỷ - Megabook.vnTập 2 chuyên đề Toán học: Phương trình vô tỷ - Megabook.vn
Tập 2 chuyên đề Toán học: Phương trình vô tỷ - Megabook.vn
 
Phuong trinh he_phuong_trinh_vo_ti_551
Phuong trinh he_phuong_trinh_vo_ti_551Phuong trinh he_phuong_trinh_vo_ti_551
Phuong trinh he_phuong_trinh_vo_ti_551
 
TUYỂN TẬP 23 CHUYÊN ĐỀ ĐẠI SỐ BỒI DƯỠNG HỌC SINH GIỎI TOÁN LỚP 9 - CÓ LỜI GIẢ...
TUYỂN TẬP 23 CHUYÊN ĐỀ ĐẠI SỐ BỒI DƯỠNG HỌC SINH GIỎI TOÁN LỚP 9 - CÓ LỜI GIẢ...TUYỂN TẬP 23 CHUYÊN ĐỀ ĐẠI SỐ BỒI DƯỠNG HỌC SINH GIỎI TOÁN LỚP 9 - CÓ LỜI GIẢ...
TUYỂN TẬP 23 CHUYÊN ĐỀ ĐẠI SỐ BỒI DƯỠNG HỌC SINH GIỎI TOÁN LỚP 9 - CÓ LỜI GIẢ...
 
Pp giai pt va hpt khong mau muc
Pp giai pt va hpt khong mau mucPp giai pt va hpt khong mau muc
Pp giai pt va hpt khong mau muc
 
02 phuong phap dat an phu giai pt p2
02 phuong phap dat an phu giai pt p202 phuong phap dat an phu giai pt p2
02 phuong phap dat an phu giai pt p2
 
Dethi hsg-l10-2013-ha tinh-toan
Dethi hsg-l10-2013-ha tinh-toanDethi hsg-l10-2013-ha tinh-toan
Dethi hsg-l10-2013-ha tinh-toan
 
06 ki thuat dong nhat tim nguyen ham
06 ki thuat dong nhat tim nguyen ham06 ki thuat dong nhat tim nguyen ham
06 ki thuat dong nhat tim nguyen ham
 
52 bài hệ phương trình
52 bài hệ phương trình52 bài hệ phương trình
52 bài hệ phương trình
 
Mot so chu y khi giai pt
Mot so chu y khi giai ptMot so chu y khi giai pt
Mot so chu y khi giai pt
 

More from Nguyen Thanh Tu Collection

20 ĐỀ DỰ ĐOÁN - PHÁT TRIỂN ĐỀ MINH HỌA BGD KỲ THI TỐT NGHIỆP THPT NĂM 2024 MÔ...
20 ĐỀ DỰ ĐOÁN - PHÁT TRIỂN ĐỀ MINH HỌA BGD KỲ THI TỐT NGHIỆP THPT NĂM 2024 MÔ...20 ĐỀ DỰ ĐOÁN - PHÁT TRIỂN ĐỀ MINH HỌA BGD KỲ THI TỐT NGHIỆP THPT NĂM 2024 MÔ...
20 ĐỀ DỰ ĐOÁN - PHÁT TRIỂN ĐỀ MINH HỌA BGD KỲ THI TỐT NGHIỆP THPT NĂM 2024 MÔ...
Nguyen Thanh Tu Collection
 
CHUYÊN ĐỀ DẠY THÊM VẬT LÝ LỚP 12 THEO FORM THI MỚI BGD 2025 DÙNG CHUNG 3 SÁCH...
CHUYÊN ĐỀ DẠY THÊM VẬT LÝ LỚP 12 THEO FORM THI MỚI BGD 2025 DÙNG CHUNG 3 SÁCH...CHUYÊN ĐỀ DẠY THÊM VẬT LÝ LỚP 12 THEO FORM THI MỚI BGD 2025 DÙNG CHUNG 3 SÁCH...
CHUYÊN ĐỀ DẠY THÊM VẬT LÝ LỚP 12 THEO FORM THI MỚI BGD 2025 DÙNG CHUNG 3 SÁCH...
Nguyen Thanh Tu Collection
 
CHUYÊN ĐỀ DẠY THÊM TOÁN LỚP 12 THEO FORM THI MỚI BGD 2025 - KẾT NỐI TRI THỨC ...
CHUYÊN ĐỀ DẠY THÊM TOÁN LỚP 12 THEO FORM THI MỚI BGD 2025 - KẾT NỐI TRI THỨC ...CHUYÊN ĐỀ DẠY THÊM TOÁN LỚP 12 THEO FORM THI MỚI BGD 2025 - KẾT NỐI TRI THỨC ...
CHUYÊN ĐỀ DẠY THÊM TOÁN LỚP 12 THEO FORM THI MỚI BGD 2025 - KẾT NỐI TRI THỨC ...
Nguyen Thanh Tu Collection
 
CHUYÊN ĐỀ DẠY THÊM TOÁN LỚP 12 THEO FORM THI MỚI BGD 2025 - CHÂN TRỜI SÁNG TẠ...
CHUYÊN ĐỀ DẠY THÊM TOÁN LỚP 12 THEO FORM THI MỚI BGD 2025 - CHÂN TRỜI SÁNG TẠ...CHUYÊN ĐỀ DẠY THÊM TOÁN LỚP 12 THEO FORM THI MỚI BGD 2025 - CHÂN TRỜI SÁNG TẠ...
CHUYÊN ĐỀ DẠY THÊM TOÁN LỚP 12 THEO FORM THI MỚI BGD 2025 - CHÂN TRỜI SÁNG TẠ...
Nguyen Thanh Tu Collection
 
TÀI LIỆU DẠY THÊM HÓA HỌC 12 - SÁCH MỚI (BẢN HS+GV) (FORM BÀI TẬP 2025 CHUNG ...
TÀI LIỆU DẠY THÊM HÓA HỌC 12 - SÁCH MỚI (BẢN HS+GV) (FORM BÀI TẬP 2025 CHUNG ...TÀI LIỆU DẠY THÊM HÓA HỌC 12 - SÁCH MỚI (BẢN HS+GV) (FORM BÀI TẬP 2025 CHUNG ...
TÀI LIỆU DẠY THÊM HÓA HỌC 12 - SÁCH MỚI (BẢN HS+GV) (FORM BÀI TẬP 2025 CHUNG ...
Nguyen Thanh Tu Collection
 
CHUYÊN ĐỀ ÔN TẬP VÀ PHÁT TRIỂN CÂU HỎI TRONG ĐỀ MINH HỌA THI TỐT NGHIỆP THPT ...
CHUYÊN ĐỀ ÔN TẬP VÀ PHÁT TRIỂN CÂU HỎI TRONG ĐỀ MINH HỌA THI TỐT NGHIỆP THPT ...CHUYÊN ĐỀ ÔN TẬP VÀ PHÁT TRIỂN CÂU HỎI TRONG ĐỀ MINH HỌA THI TỐT NGHIỆP THPT ...
CHUYÊN ĐỀ ÔN TẬP VÀ PHÁT TRIỂN CÂU HỎI TRONG ĐỀ MINH HỌA THI TỐT NGHIỆP THPT ...
Nguyen Thanh Tu Collection
 
BÀI TẬP BỔ TRỢ TIẾNG ANH LỚP 8 - CẢ NĂM - FRIENDS PLUS - NĂM HỌC 2023-2024 (B...
BÀI TẬP BỔ TRỢ TIẾNG ANH LỚP 8 - CẢ NĂM - FRIENDS PLUS - NĂM HỌC 2023-2024 (B...BÀI TẬP BỔ TRỢ TIẾNG ANH LỚP 8 - CẢ NĂM - FRIENDS PLUS - NĂM HỌC 2023-2024 (B...
BÀI TẬP BỔ TRỢ TIẾNG ANH LỚP 8 - CẢ NĂM - FRIENDS PLUS - NĂM HỌC 2023-2024 (B...
Nguyen Thanh Tu Collection
 
TỔNG HỢP 135 CÂU HỎI DI TRUYỀN PHÂN TỬ LUYỆN THI HỌC SINH GIỎI THPT MÔN SINH ...
TỔNG HỢP 135 CÂU HỎI DI TRUYỀN PHÂN TỬ LUYỆN THI HỌC SINH GIỎI THPT MÔN SINH ...TỔNG HỢP 135 CÂU HỎI DI TRUYỀN PHÂN TỬ LUYỆN THI HỌC SINH GIỎI THPT MÔN SINH ...
TỔNG HỢP 135 CÂU HỎI DI TRUYỀN PHÂN TỬ LUYỆN THI HỌC SINH GIỎI THPT MÔN SINH ...
Nguyen Thanh Tu Collection
 
BÀI TẬP DẠY THÊM HÓA HỌC LỚP 12 - CẢ NĂM - THEO FORM THI MỚI BGD 2025 (DÙNG C...
BÀI TẬP DẠY THÊM HÓA HỌC LỚP 12 - CẢ NĂM - THEO FORM THI MỚI BGD 2025 (DÙNG C...BÀI TẬP DẠY THÊM HÓA HỌC LỚP 12 - CẢ NĂM - THEO FORM THI MỚI BGD 2025 (DÙNG C...
BÀI TẬP DẠY THÊM HÓA HỌC LỚP 12 - CẢ NĂM - THEO FORM THI MỚI BGD 2025 (DÙNG C...
Nguyen Thanh Tu Collection
 
BÀI TẬP BỔ TRỢ TIẾNG ANH LỚP 9 CẢ NĂM - GLOBAL SUCCESS - NĂM HỌC 2024-2025 - ...
BÀI TẬP BỔ TRỢ TIẾNG ANH LỚP 9 CẢ NĂM - GLOBAL SUCCESS - NĂM HỌC 2024-2025 - ...BÀI TẬP BỔ TRỢ TIẾNG ANH LỚP 9 CẢ NĂM - GLOBAL SUCCESS - NĂM HỌC 2024-2025 - ...
BÀI TẬP BỔ TRỢ TIẾNG ANH LỚP 9 CẢ NĂM - GLOBAL SUCCESS - NĂM HỌC 2024-2025 - ...
Nguyen Thanh Tu Collection
 
BÀI TẬP DẠY THÊM TIẾNG ANH LỚP 7 CẢ NĂM FRIENDS PLUS SÁCH CHÂN TRỜI SÁNG TẠO ...
BÀI TẬP DẠY THÊM TIẾNG ANH LỚP 7 CẢ NĂM FRIENDS PLUS SÁCH CHÂN TRỜI SÁNG TẠO ...BÀI TẬP DẠY THÊM TIẾNG ANH LỚP 7 CẢ NĂM FRIENDS PLUS SÁCH CHÂN TRỜI SÁNG TẠO ...
BÀI TẬP DẠY THÊM TIẾNG ANH LỚP 7 CẢ NĂM FRIENDS PLUS SÁCH CHÂN TRỜI SÁNG TẠO ...
Nguyen Thanh Tu Collection
 
CHUYÊN ĐỀ DẠY THÊM HÓA HỌC LỚP 10 - SÁCH MỚI - FORM BÀI TẬP 2025 (DÙNG CHUNG ...
CHUYÊN ĐỀ DẠY THÊM HÓA HỌC LỚP 10 - SÁCH MỚI - FORM BÀI TẬP 2025 (DÙNG CHUNG ...CHUYÊN ĐỀ DẠY THÊM HÓA HỌC LỚP 10 - SÁCH MỚI - FORM BÀI TẬP 2025 (DÙNG CHUNG ...
CHUYÊN ĐỀ DẠY THÊM HÓA HỌC LỚP 10 - SÁCH MỚI - FORM BÀI TẬP 2025 (DÙNG CHUNG ...
Nguyen Thanh Tu Collection
 
BÀI TẬP BỔ TRỢ TIẾNG ANH 8 CẢ NĂM - GLOBAL SUCCESS - NĂM HỌC 2023-2024 (CÓ FI...
BÀI TẬP BỔ TRỢ TIẾNG ANH 8 CẢ NĂM - GLOBAL SUCCESS - NĂM HỌC 2023-2024 (CÓ FI...BÀI TẬP BỔ TRỢ TIẾNG ANH 8 CẢ NĂM - GLOBAL SUCCESS - NĂM HỌC 2023-2024 (CÓ FI...
BÀI TẬP BỔ TRỢ TIẾNG ANH 8 CẢ NĂM - GLOBAL SUCCESS - NĂM HỌC 2023-2024 (CÓ FI...
Nguyen Thanh Tu Collection
 
BÀI TẬP BỔ TRỢ TIẾNG ANH I-LEARN SMART WORLD 9 CẢ NĂM CÓ TEST THEO UNIT NĂM H...
BÀI TẬP BỔ TRỢ TIẾNG ANH I-LEARN SMART WORLD 9 CẢ NĂM CÓ TEST THEO UNIT NĂM H...BÀI TẬP BỔ TRỢ TIẾNG ANH I-LEARN SMART WORLD 9 CẢ NĂM CÓ TEST THEO UNIT NĂM H...
BÀI TẬP BỔ TRỢ TIẾNG ANH I-LEARN SMART WORLD 9 CẢ NĂM CÓ TEST THEO UNIT NĂM H...
Nguyen Thanh Tu Collection
 
CHUYÊN ĐỀ BỒI DƯỠNG HỌC SINH GIỎI KHOA HỌC TỰ NHIÊN 9 CHƯƠNG TRÌNH MỚI - PHẦN...
CHUYÊN ĐỀ BỒI DƯỠNG HỌC SINH GIỎI KHOA HỌC TỰ NHIÊN 9 CHƯƠNG TRÌNH MỚI - PHẦN...CHUYÊN ĐỀ BỒI DƯỠNG HỌC SINH GIỎI KHOA HỌC TỰ NHIÊN 9 CHƯƠNG TRÌNH MỚI - PHẦN...
CHUYÊN ĐỀ BỒI DƯỠNG HỌC SINH GIỎI KHOA HỌC TỰ NHIÊN 9 CHƯƠNG TRÌNH MỚI - PHẦN...
Nguyen Thanh Tu Collection
 
BÀI TẬP BỔ TRỢ TIẾNG ANH GLOBAL SUCCESS LỚP 3 - CẢ NĂM (CÓ FILE NGHE VÀ ĐÁP Á...
BÀI TẬP BỔ TRỢ TIẾNG ANH GLOBAL SUCCESS LỚP 3 - CẢ NĂM (CÓ FILE NGHE VÀ ĐÁP Á...BÀI TẬP BỔ TRỢ TIẾNG ANH GLOBAL SUCCESS LỚP 3 - CẢ NĂM (CÓ FILE NGHE VÀ ĐÁP Á...
BÀI TẬP BỔ TRỢ TIẾNG ANH GLOBAL SUCCESS LỚP 3 - CẢ NĂM (CÓ FILE NGHE VÀ ĐÁP Á...
Nguyen Thanh Tu Collection
 
98 BÀI LUYỆN NGHE TUYỂN SINH VÀO LỚP 10 TIẾNG ANH DẠNG TRẮC NGHIỆM 4 CÂU TRẢ ...
98 BÀI LUYỆN NGHE TUYỂN SINH VÀO LỚP 10 TIẾNG ANH DẠNG TRẮC NGHIỆM 4 CÂU TRẢ ...98 BÀI LUYỆN NGHE TUYỂN SINH VÀO LỚP 10 TIẾNG ANH DẠNG TRẮC NGHIỆM 4 CÂU TRẢ ...
98 BÀI LUYỆN NGHE TUYỂN SINH VÀO LỚP 10 TIẾNG ANH DẠNG TRẮC NGHIỆM 4 CÂU TRẢ ...
Nguyen Thanh Tu Collection
 
GIÁO ÁN DẠY THÊM (KẾ HOẠCH BÀI BUỔI 2) - TIẾNG ANH 8 GLOBAL SUCCESS (2 CỘT) N...
GIÁO ÁN DẠY THÊM (KẾ HOẠCH BÀI BUỔI 2) - TIẾNG ANH 8 GLOBAL SUCCESS (2 CỘT) N...GIÁO ÁN DẠY THÊM (KẾ HOẠCH BÀI BUỔI 2) - TIẾNG ANH 8 GLOBAL SUCCESS (2 CỘT) N...
GIÁO ÁN DẠY THÊM (KẾ HOẠCH BÀI BUỔI 2) - TIẾNG ANH 8 GLOBAL SUCCESS (2 CỘT) N...
Nguyen Thanh Tu Collection
 
Nghiên cứu cơ chế và động học phản ứng giữa hợp chất Aniline (C6H5NH2) với gố...
Nghiên cứu cơ chế và động học phản ứng giữa hợp chất Aniline (C6H5NH2) với gố...Nghiên cứu cơ chế và động học phản ứng giữa hợp chất Aniline (C6H5NH2) với gố...
Nghiên cứu cơ chế và động học phản ứng giữa hợp chất Aniline (C6H5NH2) với gố...
Nguyen Thanh Tu Collection
 
50 ĐỀ LUYỆN THI IOE LỚP 9 - NĂM HỌC 2022-2023 (CÓ LINK HÌNH, FILE AUDIO VÀ ĐÁ...
50 ĐỀ LUYỆN THI IOE LỚP 9 - NĂM HỌC 2022-2023 (CÓ LINK HÌNH, FILE AUDIO VÀ ĐÁ...50 ĐỀ LUYỆN THI IOE LỚP 9 - NĂM HỌC 2022-2023 (CÓ LINK HÌNH, FILE AUDIO VÀ ĐÁ...
50 ĐỀ LUYỆN THI IOE LỚP 9 - NĂM HỌC 2022-2023 (CÓ LINK HÌNH, FILE AUDIO VÀ ĐÁ...
Nguyen Thanh Tu Collection
 

More from Nguyen Thanh Tu Collection (20)

20 ĐỀ DỰ ĐOÁN - PHÁT TRIỂN ĐỀ MINH HỌA BGD KỲ THI TỐT NGHIỆP THPT NĂM 2024 MÔ...
20 ĐỀ DỰ ĐOÁN - PHÁT TRIỂN ĐỀ MINH HỌA BGD KỲ THI TỐT NGHIỆP THPT NĂM 2024 MÔ...20 ĐỀ DỰ ĐOÁN - PHÁT TRIỂN ĐỀ MINH HỌA BGD KỲ THI TỐT NGHIỆP THPT NĂM 2024 MÔ...
20 ĐỀ DỰ ĐOÁN - PHÁT TRIỂN ĐỀ MINH HỌA BGD KỲ THI TỐT NGHIỆP THPT NĂM 2024 MÔ...
 
CHUYÊN ĐỀ DẠY THÊM VẬT LÝ LỚP 12 THEO FORM THI MỚI BGD 2025 DÙNG CHUNG 3 SÁCH...
CHUYÊN ĐỀ DẠY THÊM VẬT LÝ LỚP 12 THEO FORM THI MỚI BGD 2025 DÙNG CHUNG 3 SÁCH...CHUYÊN ĐỀ DẠY THÊM VẬT LÝ LỚP 12 THEO FORM THI MỚI BGD 2025 DÙNG CHUNG 3 SÁCH...
CHUYÊN ĐỀ DẠY THÊM VẬT LÝ LỚP 12 THEO FORM THI MỚI BGD 2025 DÙNG CHUNG 3 SÁCH...
 
CHUYÊN ĐỀ DẠY THÊM TOÁN LỚP 12 THEO FORM THI MỚI BGD 2025 - KẾT NỐI TRI THỨC ...
CHUYÊN ĐỀ DẠY THÊM TOÁN LỚP 12 THEO FORM THI MỚI BGD 2025 - KẾT NỐI TRI THỨC ...CHUYÊN ĐỀ DẠY THÊM TOÁN LỚP 12 THEO FORM THI MỚI BGD 2025 - KẾT NỐI TRI THỨC ...
CHUYÊN ĐỀ DẠY THÊM TOÁN LỚP 12 THEO FORM THI MỚI BGD 2025 - KẾT NỐI TRI THỨC ...
 
CHUYÊN ĐỀ DẠY THÊM TOÁN LỚP 12 THEO FORM THI MỚI BGD 2025 - CHÂN TRỜI SÁNG TẠ...
CHUYÊN ĐỀ DẠY THÊM TOÁN LỚP 12 THEO FORM THI MỚI BGD 2025 - CHÂN TRỜI SÁNG TẠ...CHUYÊN ĐỀ DẠY THÊM TOÁN LỚP 12 THEO FORM THI MỚI BGD 2025 - CHÂN TRỜI SÁNG TẠ...
CHUYÊN ĐỀ DẠY THÊM TOÁN LỚP 12 THEO FORM THI MỚI BGD 2025 - CHÂN TRỜI SÁNG TẠ...
 
TÀI LIỆU DẠY THÊM HÓA HỌC 12 - SÁCH MỚI (BẢN HS+GV) (FORM BÀI TẬP 2025 CHUNG ...
TÀI LIỆU DẠY THÊM HÓA HỌC 12 - SÁCH MỚI (BẢN HS+GV) (FORM BÀI TẬP 2025 CHUNG ...TÀI LIỆU DẠY THÊM HÓA HỌC 12 - SÁCH MỚI (BẢN HS+GV) (FORM BÀI TẬP 2025 CHUNG ...
TÀI LIỆU DẠY THÊM HÓA HỌC 12 - SÁCH MỚI (BẢN HS+GV) (FORM BÀI TẬP 2025 CHUNG ...
 
CHUYÊN ĐỀ ÔN TẬP VÀ PHÁT TRIỂN CÂU HỎI TRONG ĐỀ MINH HỌA THI TỐT NGHIỆP THPT ...
CHUYÊN ĐỀ ÔN TẬP VÀ PHÁT TRIỂN CÂU HỎI TRONG ĐỀ MINH HỌA THI TỐT NGHIỆP THPT ...CHUYÊN ĐỀ ÔN TẬP VÀ PHÁT TRIỂN CÂU HỎI TRONG ĐỀ MINH HỌA THI TỐT NGHIỆP THPT ...
CHUYÊN ĐỀ ÔN TẬP VÀ PHÁT TRIỂN CÂU HỎI TRONG ĐỀ MINH HỌA THI TỐT NGHIỆP THPT ...
 
BÀI TẬP BỔ TRỢ TIẾNG ANH LỚP 8 - CẢ NĂM - FRIENDS PLUS - NĂM HỌC 2023-2024 (B...
BÀI TẬP BỔ TRỢ TIẾNG ANH LỚP 8 - CẢ NĂM - FRIENDS PLUS - NĂM HỌC 2023-2024 (B...BÀI TẬP BỔ TRỢ TIẾNG ANH LỚP 8 - CẢ NĂM - FRIENDS PLUS - NĂM HỌC 2023-2024 (B...
BÀI TẬP BỔ TRỢ TIẾNG ANH LỚP 8 - CẢ NĂM - FRIENDS PLUS - NĂM HỌC 2023-2024 (B...
 
TỔNG HỢP 135 CÂU HỎI DI TRUYỀN PHÂN TỬ LUYỆN THI HỌC SINH GIỎI THPT MÔN SINH ...
TỔNG HỢP 135 CÂU HỎI DI TRUYỀN PHÂN TỬ LUYỆN THI HỌC SINH GIỎI THPT MÔN SINH ...TỔNG HỢP 135 CÂU HỎI DI TRUYỀN PHÂN TỬ LUYỆN THI HỌC SINH GIỎI THPT MÔN SINH ...
TỔNG HỢP 135 CÂU HỎI DI TRUYỀN PHÂN TỬ LUYỆN THI HỌC SINH GIỎI THPT MÔN SINH ...
 
BÀI TẬP DẠY THÊM HÓA HỌC LỚP 12 - CẢ NĂM - THEO FORM THI MỚI BGD 2025 (DÙNG C...
BÀI TẬP DẠY THÊM HÓA HỌC LỚP 12 - CẢ NĂM - THEO FORM THI MỚI BGD 2025 (DÙNG C...BÀI TẬP DẠY THÊM HÓA HỌC LỚP 12 - CẢ NĂM - THEO FORM THI MỚI BGD 2025 (DÙNG C...
BÀI TẬP DẠY THÊM HÓA HỌC LỚP 12 - CẢ NĂM - THEO FORM THI MỚI BGD 2025 (DÙNG C...
 
BÀI TẬP BỔ TRỢ TIẾNG ANH LỚP 9 CẢ NĂM - GLOBAL SUCCESS - NĂM HỌC 2024-2025 - ...
BÀI TẬP BỔ TRỢ TIẾNG ANH LỚP 9 CẢ NĂM - GLOBAL SUCCESS - NĂM HỌC 2024-2025 - ...BÀI TẬP BỔ TRỢ TIẾNG ANH LỚP 9 CẢ NĂM - GLOBAL SUCCESS - NĂM HỌC 2024-2025 - ...
BÀI TẬP BỔ TRỢ TIẾNG ANH LỚP 9 CẢ NĂM - GLOBAL SUCCESS - NĂM HỌC 2024-2025 - ...
 
BÀI TẬP DẠY THÊM TIẾNG ANH LỚP 7 CẢ NĂM FRIENDS PLUS SÁCH CHÂN TRỜI SÁNG TẠO ...
BÀI TẬP DẠY THÊM TIẾNG ANH LỚP 7 CẢ NĂM FRIENDS PLUS SÁCH CHÂN TRỜI SÁNG TẠO ...BÀI TẬP DẠY THÊM TIẾNG ANH LỚP 7 CẢ NĂM FRIENDS PLUS SÁCH CHÂN TRỜI SÁNG TẠO ...
BÀI TẬP DẠY THÊM TIẾNG ANH LỚP 7 CẢ NĂM FRIENDS PLUS SÁCH CHÂN TRỜI SÁNG TẠO ...
 
CHUYÊN ĐỀ DẠY THÊM HÓA HỌC LỚP 10 - SÁCH MỚI - FORM BÀI TẬP 2025 (DÙNG CHUNG ...
CHUYÊN ĐỀ DẠY THÊM HÓA HỌC LỚP 10 - SÁCH MỚI - FORM BÀI TẬP 2025 (DÙNG CHUNG ...CHUYÊN ĐỀ DẠY THÊM HÓA HỌC LỚP 10 - SÁCH MỚI - FORM BÀI TẬP 2025 (DÙNG CHUNG ...
CHUYÊN ĐỀ DẠY THÊM HÓA HỌC LỚP 10 - SÁCH MỚI - FORM BÀI TẬP 2025 (DÙNG CHUNG ...
 
BÀI TẬP BỔ TRỢ TIẾNG ANH 8 CẢ NĂM - GLOBAL SUCCESS - NĂM HỌC 2023-2024 (CÓ FI...
BÀI TẬP BỔ TRỢ TIẾNG ANH 8 CẢ NĂM - GLOBAL SUCCESS - NĂM HỌC 2023-2024 (CÓ FI...BÀI TẬP BỔ TRỢ TIẾNG ANH 8 CẢ NĂM - GLOBAL SUCCESS - NĂM HỌC 2023-2024 (CÓ FI...
BÀI TẬP BỔ TRỢ TIẾNG ANH 8 CẢ NĂM - GLOBAL SUCCESS - NĂM HỌC 2023-2024 (CÓ FI...
 
BÀI TẬP BỔ TRỢ TIẾNG ANH I-LEARN SMART WORLD 9 CẢ NĂM CÓ TEST THEO UNIT NĂM H...
BÀI TẬP BỔ TRỢ TIẾNG ANH I-LEARN SMART WORLD 9 CẢ NĂM CÓ TEST THEO UNIT NĂM H...BÀI TẬP BỔ TRỢ TIẾNG ANH I-LEARN SMART WORLD 9 CẢ NĂM CÓ TEST THEO UNIT NĂM H...
BÀI TẬP BỔ TRỢ TIẾNG ANH I-LEARN SMART WORLD 9 CẢ NĂM CÓ TEST THEO UNIT NĂM H...
 
CHUYÊN ĐỀ BỒI DƯỠNG HỌC SINH GIỎI KHOA HỌC TỰ NHIÊN 9 CHƯƠNG TRÌNH MỚI - PHẦN...
CHUYÊN ĐỀ BỒI DƯỠNG HỌC SINH GIỎI KHOA HỌC TỰ NHIÊN 9 CHƯƠNG TRÌNH MỚI - PHẦN...CHUYÊN ĐỀ BỒI DƯỠNG HỌC SINH GIỎI KHOA HỌC TỰ NHIÊN 9 CHƯƠNG TRÌNH MỚI - PHẦN...
CHUYÊN ĐỀ BỒI DƯỠNG HỌC SINH GIỎI KHOA HỌC TỰ NHIÊN 9 CHƯƠNG TRÌNH MỚI - PHẦN...
 
BÀI TẬP BỔ TRỢ TIẾNG ANH GLOBAL SUCCESS LỚP 3 - CẢ NĂM (CÓ FILE NGHE VÀ ĐÁP Á...
BÀI TẬP BỔ TRỢ TIẾNG ANH GLOBAL SUCCESS LỚP 3 - CẢ NĂM (CÓ FILE NGHE VÀ ĐÁP Á...BÀI TẬP BỔ TRỢ TIẾNG ANH GLOBAL SUCCESS LỚP 3 - CẢ NĂM (CÓ FILE NGHE VÀ ĐÁP Á...
BÀI TẬP BỔ TRỢ TIẾNG ANH GLOBAL SUCCESS LỚP 3 - CẢ NĂM (CÓ FILE NGHE VÀ ĐÁP Á...
 
98 BÀI LUYỆN NGHE TUYỂN SINH VÀO LỚP 10 TIẾNG ANH DẠNG TRẮC NGHIỆM 4 CÂU TRẢ ...
98 BÀI LUYỆN NGHE TUYỂN SINH VÀO LỚP 10 TIẾNG ANH DẠNG TRẮC NGHIỆM 4 CÂU TRẢ ...98 BÀI LUYỆN NGHE TUYỂN SINH VÀO LỚP 10 TIẾNG ANH DẠNG TRẮC NGHIỆM 4 CÂU TRẢ ...
98 BÀI LUYỆN NGHE TUYỂN SINH VÀO LỚP 10 TIẾNG ANH DẠNG TRẮC NGHIỆM 4 CÂU TRẢ ...
 
GIÁO ÁN DẠY THÊM (KẾ HOẠCH BÀI BUỔI 2) - TIẾNG ANH 8 GLOBAL SUCCESS (2 CỘT) N...
GIÁO ÁN DẠY THÊM (KẾ HOẠCH BÀI BUỔI 2) - TIẾNG ANH 8 GLOBAL SUCCESS (2 CỘT) N...GIÁO ÁN DẠY THÊM (KẾ HOẠCH BÀI BUỔI 2) - TIẾNG ANH 8 GLOBAL SUCCESS (2 CỘT) N...
GIÁO ÁN DẠY THÊM (KẾ HOẠCH BÀI BUỔI 2) - TIẾNG ANH 8 GLOBAL SUCCESS (2 CỘT) N...
 
Nghiên cứu cơ chế và động học phản ứng giữa hợp chất Aniline (C6H5NH2) với gố...
Nghiên cứu cơ chế và động học phản ứng giữa hợp chất Aniline (C6H5NH2) với gố...Nghiên cứu cơ chế và động học phản ứng giữa hợp chất Aniline (C6H5NH2) với gố...
Nghiên cứu cơ chế và động học phản ứng giữa hợp chất Aniline (C6H5NH2) với gố...
 
50 ĐỀ LUYỆN THI IOE LỚP 9 - NĂM HỌC 2022-2023 (CÓ LINK HÌNH, FILE AUDIO VÀ ĐÁ...
50 ĐỀ LUYỆN THI IOE LỚP 9 - NĂM HỌC 2022-2023 (CÓ LINK HÌNH, FILE AUDIO VÀ ĐÁ...50 ĐỀ LUYỆN THI IOE LỚP 9 - NĂM HỌC 2022-2023 (CÓ LINK HÌNH, FILE AUDIO VÀ ĐÁ...
50 ĐỀ LUYỆN THI IOE LỚP 9 - NĂM HỌC 2022-2023 (CÓ LINK HÌNH, FILE AUDIO VÀ ĐÁ...
 

Recently uploaded

Ngon_ngu_hoc_doi_chieu Các phạm trù cơ bản.pptx
Ngon_ngu_hoc_doi_chieu Các phạm trù cơ bản.pptxNgon_ngu_hoc_doi_chieu Các phạm trù cơ bản.pptx
Ngon_ngu_hoc_doi_chieu Các phạm trù cơ bản.pptx
linhlevietdav
 
Atomic Habits - Thay Đổi Tí Hon, Hiệu Quả Bất Ngờ - James Clear & L...
Atomic Habits - Thay Đổi Tí Hon, Hiệu Quả Bất Ngờ - James Clear & L...Atomic Habits - Thay Đổi Tí Hon, Hiệu Quả Bất Ngờ - James Clear & L...
Atomic Habits - Thay Đổi Tí Hon, Hiệu Quả Bất Ngờ - James Clear & L...
williamminerva131
 
Diễn giải Tâm lý - Chiêm tinh Thầy Minh Tuệ
Diễn giải Tâm lý - Chiêm tinh Thầy Minh TuệDiễn giải Tâm lý - Chiêm tinh Thầy Minh Tuệ
Diễn giải Tâm lý - Chiêm tinh Thầy Minh Tuệ
Little Daisy
 
kltn_Nâng Cao Chất Lượng Đội Ngũ Công Chức Cấp Phường Trên Địa Bàn Quận Hà Đô...
kltn_Nâng Cao Chất Lượng Đội Ngũ Công Chức Cấp Phường Trên Địa Bàn Quận Hà Đô...kltn_Nâng Cao Chất Lượng Đội Ngũ Công Chức Cấp Phường Trên Địa Bàn Quận Hà Đô...
kltn_Nâng Cao Chất Lượng Đội Ngũ Công Chức Cấp Phường Trên Địa Bàn Quận Hà Đô...
Luận Văn Uy Tín
 
Tiểu luận Chiến dịch Chính nữ - Vì bạn xứng đáng
Tiểu luận Chiến dịch Chính nữ - Vì bạn xứng đángTiểu luận Chiến dịch Chính nữ - Vì bạn xứng đáng
Tiểu luận Chiến dịch Chính nữ - Vì bạn xứng đáng
lamluanvan.net Viết thuê luận văn
 
Những khó khăn của sinh viên năm nhất Học viện Hành chính Quốc gia trong quá ...
Những khó khăn của sinh viên năm nhất Học viện Hành chính Quốc gia trong quá ...Những khó khăn của sinh viên năm nhất Học viện Hành chính Quốc gia trong quá ...
Những khó khăn của sinh viên năm nhất Học viện Hành chính Quốc gia trong quá ...
lamluanvan.net Viết thuê luận văn
 
thuvienhoclieu.com-De-thi-thu-TN-THPT-2024-Tieng-Anh-phat-trien-tu-de-minh-ho...
thuvienhoclieu.com-De-thi-thu-TN-THPT-2024-Tieng-Anh-phat-trien-tu-de-minh-ho...thuvienhoclieu.com-De-thi-thu-TN-THPT-2024-Tieng-Anh-phat-trien-tu-de-minh-ho...
thuvienhoclieu.com-De-thi-thu-TN-THPT-2024-Tieng-Anh-phat-trien-tu-de-minh-ho...
HngNguyn2390
 
Ngon ngu hoc doi chieu Nguyen van Huy.ppt
Ngon ngu hoc doi chieu Nguyen van Huy.pptNgon ngu hoc doi chieu Nguyen van Huy.ppt
Ngon ngu hoc doi chieu Nguyen van Huy.ppt
linhlevietdav
 
GƯƠNG ĐIỂN HÌNH NGƯỜI TỐT - VIỆC TỐT DƯƠNG THU NGA - NỮ KỸ THUẬT VIÊN PHỤC HỒ...
GƯƠNG ĐIỂN HÌNH NGƯỜI TỐT - VIỆC TỐT DƯƠNG THU NGA - NỮ KỸ THUẬT VIÊN PHỤC HỒ...GƯƠNG ĐIỂN HÌNH NGƯỜI TỐT - VIỆC TỐT DƯƠNG THU NGA - NỮ KỸ THUẬT VIÊN PHỤC HỒ...
GƯƠNG ĐIỂN HÌNH NGƯỜI TỐT - VIỆC TỐT DƯƠNG THU NGA - NỮ KỸ THUẬT VIÊN PHỤC HỒ...
lamluanvan.net Viết thuê luận văn
 
Bài tập chương 5. Năng lượng phản ứng.docx
Bài tập chương 5. Năng lượng phản ứng.docxBài tập chương 5. Năng lượng phản ứng.docx
Bài tập chương 5. Năng lượng phản ứng.docx
gorse871
 
Bài 4. Khảo sát mạch dao động điện từ.doc
Bài 4. Khảo sát mạch dao động điện từ.docBài 4. Khảo sát mạch dao động điện từ.doc
Bài 4. Khảo sát mạch dao động điện từ.doc
phamvanchinhlqd
 
vật lý 1 k23 cuối kì c10-11- Nam Lê.pptx
vật lý 1 k23 cuối kì c10-11- Nam Lê.pptxvật lý 1 k23 cuối kì c10-11- Nam Lê.pptx
vật lý 1 k23 cuối kì c10-11- Nam Lê.pptx
LinhTrn115148
 
DANH SÁCH XÉT TUYỂN SỚM_NĂM 2023_học ba DPY.pdf
DANH SÁCH XÉT TUYỂN SỚM_NĂM 2023_học ba DPY.pdfDANH SÁCH XÉT TUYỂN SỚM_NĂM 2023_học ba DPY.pdf
DANH SÁCH XÉT TUYỂN SỚM_NĂM 2023_học ba DPY.pdf
thanhluan21
 
Giải phẫu sinh lý Hệ sinh sản DH Y duoc TPHCM
Giải phẫu sinh lý Hệ sinh sản DH Y duoc TPHCMGiải phẫu sinh lý Hệ sinh sản DH Y duoc TPHCM
Giải phẫu sinh lý Hệ sinh sản DH Y duoc TPHCM
LinhChu679649
 
CD6_DAI_CUONG_KIMLOAI_12CB218LTTTHU5.pdf
CD6_DAI_CUONG_KIMLOAI_12CB218LTTTHU5.pdfCD6_DAI_CUONG_KIMLOAI_12CB218LTTTHU5.pdf
CD6_DAI_CUONG_KIMLOAI_12CB218LTTTHU5.pdf
Nguyntrnhnganh
 
NHO GIÁO VÀ ẢNH HƯỞNG ĐẾN ĐỜI SỐNG TINH THẦN
NHO GIÁO VÀ ẢNH HƯỞNG ĐẾN ĐỜI SỐNG TINH THẦNNHO GIÁO VÀ ẢNH HƯỞNG ĐẾN ĐỜI SỐNG TINH THẦN
NHO GIÁO VÀ ẢNH HƯỞNG ĐẾN ĐỜI SỐNG TINH THẦN
lamluanvan.net Viết thuê luận văn
 
kl_HOÀN THIỆN CÔNG TÁC ĐÁNH GIÁ THỰC HIỆN CÔNG VIỆC TẠI CÔNG TY CỔ PHẦN ĐẦU T...
kl_HOÀN THIỆN CÔNG TÁC ĐÁNH GIÁ THỰC HIỆN CÔNG VIỆC TẠI CÔNG TY CỔ PHẦN ĐẦU T...kl_HOÀN THIỆN CÔNG TÁC ĐÁNH GIÁ THỰC HIỆN CÔNG VIỆC TẠI CÔNG TY CỔ PHẦN ĐẦU T...
kl_HOÀN THIỆN CÔNG TÁC ĐÁNH GIÁ THỰC HIỆN CÔNG VIỆC TẠI CÔNG TY CỔ PHẦN ĐẦU T...
Luận Văn Uy Tín
 
CHỮ “TRÍ” THEO TƯ TƯỞNG NHO GIÁO VÀ Ý NGHĨA TRONG ĐỔI MỚI GIAÓ DỤC Ở VIỆT NAM...
CHỮ “TRÍ” THEO TƯ TƯỞNG NHO GIÁO VÀ Ý NGHĨA TRONG ĐỔI MỚI GIAÓ DỤC Ở VIỆT NAM...CHỮ “TRÍ” THEO TƯ TƯỞNG NHO GIÁO VÀ Ý NGHĨA TRONG ĐỔI MỚI GIAÓ DỤC Ở VIỆT NAM...
CHỮ “TRÍ” THEO TƯ TƯỞNG NHO GIÁO VÀ Ý NGHĨA TRONG ĐỔI MỚI GIAÓ DỤC Ở VIỆT NAM...
lamluanvan.net Viết thuê luận văn
 
CHIẾN LƯỢC DẠY TIẾNG ANH THEO CHƯƠNG TRÌNH GD 2018 CHO HỌC SINH TRUNG BÌNH YẾ...
CHIẾN LƯỢC DẠY TIẾNG ANH THEO CHƯƠNG TRÌNH GD 2018 CHO HỌC SINH TRUNG BÌNH YẾ...CHIẾN LƯỢC DẠY TIẾNG ANH THEO CHƯƠNG TRÌNH GD 2018 CHO HỌC SINH TRUNG BÌNH YẾ...
CHIẾN LƯỢC DẠY TIẾNG ANH THEO CHƯƠNG TRÌNH GD 2018 CHO HỌC SINH TRUNG BÌNH YẾ...
lamluanvan.net Viết thuê luận văn
 
Các bình diện Ngôn ngữ học đối chiếu.pdf
Các bình diện Ngôn ngữ học đối chiếu.pdfCác bình diện Ngôn ngữ học đối chiếu.pdf
Các bình diện Ngôn ngữ học đối chiếu.pdf
linhlevietdav
 

Recently uploaded (20)

Ngon_ngu_hoc_doi_chieu Các phạm trù cơ bản.pptx
Ngon_ngu_hoc_doi_chieu Các phạm trù cơ bản.pptxNgon_ngu_hoc_doi_chieu Các phạm trù cơ bản.pptx
Ngon_ngu_hoc_doi_chieu Các phạm trù cơ bản.pptx
 
Atomic Habits - Thay Đổi Tí Hon, Hiệu Quả Bất Ngờ - James Clear & L...
Atomic Habits - Thay Đổi Tí Hon, Hiệu Quả Bất Ngờ - James Clear & L...Atomic Habits - Thay Đổi Tí Hon, Hiệu Quả Bất Ngờ - James Clear & L...
Atomic Habits - Thay Đổi Tí Hon, Hiệu Quả Bất Ngờ - James Clear & L...
 
Diễn giải Tâm lý - Chiêm tinh Thầy Minh Tuệ
Diễn giải Tâm lý - Chiêm tinh Thầy Minh TuệDiễn giải Tâm lý - Chiêm tinh Thầy Minh Tuệ
Diễn giải Tâm lý - Chiêm tinh Thầy Minh Tuệ
 
kltn_Nâng Cao Chất Lượng Đội Ngũ Công Chức Cấp Phường Trên Địa Bàn Quận Hà Đô...
kltn_Nâng Cao Chất Lượng Đội Ngũ Công Chức Cấp Phường Trên Địa Bàn Quận Hà Đô...kltn_Nâng Cao Chất Lượng Đội Ngũ Công Chức Cấp Phường Trên Địa Bàn Quận Hà Đô...
kltn_Nâng Cao Chất Lượng Đội Ngũ Công Chức Cấp Phường Trên Địa Bàn Quận Hà Đô...
 
Tiểu luận Chiến dịch Chính nữ - Vì bạn xứng đáng
Tiểu luận Chiến dịch Chính nữ - Vì bạn xứng đángTiểu luận Chiến dịch Chính nữ - Vì bạn xứng đáng
Tiểu luận Chiến dịch Chính nữ - Vì bạn xứng đáng
 
Những khó khăn của sinh viên năm nhất Học viện Hành chính Quốc gia trong quá ...
Những khó khăn của sinh viên năm nhất Học viện Hành chính Quốc gia trong quá ...Những khó khăn của sinh viên năm nhất Học viện Hành chính Quốc gia trong quá ...
Những khó khăn của sinh viên năm nhất Học viện Hành chính Quốc gia trong quá ...
 
thuvienhoclieu.com-De-thi-thu-TN-THPT-2024-Tieng-Anh-phat-trien-tu-de-minh-ho...
thuvienhoclieu.com-De-thi-thu-TN-THPT-2024-Tieng-Anh-phat-trien-tu-de-minh-ho...thuvienhoclieu.com-De-thi-thu-TN-THPT-2024-Tieng-Anh-phat-trien-tu-de-minh-ho...
thuvienhoclieu.com-De-thi-thu-TN-THPT-2024-Tieng-Anh-phat-trien-tu-de-minh-ho...
 
Ngon ngu hoc doi chieu Nguyen van Huy.ppt
Ngon ngu hoc doi chieu Nguyen van Huy.pptNgon ngu hoc doi chieu Nguyen van Huy.ppt
Ngon ngu hoc doi chieu Nguyen van Huy.ppt
 
GƯƠNG ĐIỂN HÌNH NGƯỜI TỐT - VIỆC TỐT DƯƠNG THU NGA - NỮ KỸ THUẬT VIÊN PHỤC HỒ...
GƯƠNG ĐIỂN HÌNH NGƯỜI TỐT - VIỆC TỐT DƯƠNG THU NGA - NỮ KỸ THUẬT VIÊN PHỤC HỒ...GƯƠNG ĐIỂN HÌNH NGƯỜI TỐT - VIỆC TỐT DƯƠNG THU NGA - NỮ KỸ THUẬT VIÊN PHỤC HỒ...
GƯƠNG ĐIỂN HÌNH NGƯỜI TỐT - VIỆC TỐT DƯƠNG THU NGA - NỮ KỸ THUẬT VIÊN PHỤC HỒ...
 
Bài tập chương 5. Năng lượng phản ứng.docx
Bài tập chương 5. Năng lượng phản ứng.docxBài tập chương 5. Năng lượng phản ứng.docx
Bài tập chương 5. Năng lượng phản ứng.docx
 
Bài 4. Khảo sát mạch dao động điện từ.doc
Bài 4. Khảo sát mạch dao động điện từ.docBài 4. Khảo sát mạch dao động điện từ.doc
Bài 4. Khảo sát mạch dao động điện từ.doc
 
vật lý 1 k23 cuối kì c10-11- Nam Lê.pptx
vật lý 1 k23 cuối kì c10-11- Nam Lê.pptxvật lý 1 k23 cuối kì c10-11- Nam Lê.pptx
vật lý 1 k23 cuối kì c10-11- Nam Lê.pptx
 
DANH SÁCH XÉT TUYỂN SỚM_NĂM 2023_học ba DPY.pdf
DANH SÁCH XÉT TUYỂN SỚM_NĂM 2023_học ba DPY.pdfDANH SÁCH XÉT TUYỂN SỚM_NĂM 2023_học ba DPY.pdf
DANH SÁCH XÉT TUYỂN SỚM_NĂM 2023_học ba DPY.pdf
 
Giải phẫu sinh lý Hệ sinh sản DH Y duoc TPHCM
Giải phẫu sinh lý Hệ sinh sản DH Y duoc TPHCMGiải phẫu sinh lý Hệ sinh sản DH Y duoc TPHCM
Giải phẫu sinh lý Hệ sinh sản DH Y duoc TPHCM
 
CD6_DAI_CUONG_KIMLOAI_12CB218LTTTHU5.pdf
CD6_DAI_CUONG_KIMLOAI_12CB218LTTTHU5.pdfCD6_DAI_CUONG_KIMLOAI_12CB218LTTTHU5.pdf
CD6_DAI_CUONG_KIMLOAI_12CB218LTTTHU5.pdf
 
NHO GIÁO VÀ ẢNH HƯỞNG ĐẾN ĐỜI SỐNG TINH THẦN
NHO GIÁO VÀ ẢNH HƯỞNG ĐẾN ĐỜI SỐNG TINH THẦNNHO GIÁO VÀ ẢNH HƯỞNG ĐẾN ĐỜI SỐNG TINH THẦN
NHO GIÁO VÀ ẢNH HƯỞNG ĐẾN ĐỜI SỐNG TINH THẦN
 
kl_HOÀN THIỆN CÔNG TÁC ĐÁNH GIÁ THỰC HIỆN CÔNG VIỆC TẠI CÔNG TY CỔ PHẦN ĐẦU T...
kl_HOÀN THIỆN CÔNG TÁC ĐÁNH GIÁ THỰC HIỆN CÔNG VIỆC TẠI CÔNG TY CỔ PHẦN ĐẦU T...kl_HOÀN THIỆN CÔNG TÁC ĐÁNH GIÁ THỰC HIỆN CÔNG VIỆC TẠI CÔNG TY CỔ PHẦN ĐẦU T...
kl_HOÀN THIỆN CÔNG TÁC ĐÁNH GIÁ THỰC HIỆN CÔNG VIỆC TẠI CÔNG TY CỔ PHẦN ĐẦU T...
 
CHỮ “TRÍ” THEO TƯ TƯỞNG NHO GIÁO VÀ Ý NGHĨA TRONG ĐỔI MỚI GIAÓ DỤC Ở VIỆT NAM...
CHỮ “TRÍ” THEO TƯ TƯỞNG NHO GIÁO VÀ Ý NGHĨA TRONG ĐỔI MỚI GIAÓ DỤC Ở VIỆT NAM...CHỮ “TRÍ” THEO TƯ TƯỞNG NHO GIÁO VÀ Ý NGHĨA TRONG ĐỔI MỚI GIAÓ DỤC Ở VIỆT NAM...
CHỮ “TRÍ” THEO TƯ TƯỞNG NHO GIÁO VÀ Ý NGHĨA TRONG ĐỔI MỚI GIAÓ DỤC Ở VIỆT NAM...
 
CHIẾN LƯỢC DẠY TIẾNG ANH THEO CHƯƠNG TRÌNH GD 2018 CHO HỌC SINH TRUNG BÌNH YẾ...
CHIẾN LƯỢC DẠY TIẾNG ANH THEO CHƯƠNG TRÌNH GD 2018 CHO HỌC SINH TRUNG BÌNH YẾ...CHIẾN LƯỢC DẠY TIẾNG ANH THEO CHƯƠNG TRÌNH GD 2018 CHO HỌC SINH TRUNG BÌNH YẾ...
CHIẾN LƯỢC DẠY TIẾNG ANH THEO CHƯƠNG TRÌNH GD 2018 CHO HỌC SINH TRUNG BÌNH YẾ...
 
Các bình diện Ngôn ngữ học đối chiếu.pdf
Các bình diện Ngôn ngữ học đối chiếu.pdfCác bình diện Ngôn ngữ học đối chiếu.pdf
Các bình diện Ngôn ngữ học đối chiếu.pdf
 

CHINH PHỤC KÌ THI VÀO LỚP 10 THPT MÔN TOÁN NĂM 2024 CÁC CHUYÊN ĐỀ HAY VÀ KHÓ, 20 ĐỀ LUYỆN TẬP CÓ HƯỚNG DẪN CHI TIẾT.pdf

  • 1. Hỗ trợ trực tuyến Fb www.facebook.com/DayKemQuyNhon Mobi/Zalo 0905779594 Tài liệu chuẩn tham khảo Phát triển kênh bởi Ths Nguyễn Thanh Tú Đơn vị tài trợ / phát hành / chia sẻ học thuật : Nguyen Thanh Tu Group C H I N H P H Ụ C K Ì T H I V À O L Ớ P 1 0 M Ô N T O Á N Ths Nguyễn Thanh Tú eBook Collection CHINH PHỤC KÌ THI VÀO LỚP 10 THPT MÔN TOÁN NĂM 2024 CÁC CHUYÊN ĐỀ HAY VÀ KHÓ, 20 ĐỀ LUYỆN TẬP CÓ HƯỚNG DẪN CHI TIẾT WORD VERSION | 2023 EDITION ORDER NOW / CHUYỂN GIAO QUA EMAIL TAILIEUCHUANTHAMKHAO@GMAIL.COM vectorstock.com/28062405
  • 2. Liên hệ tài liệu word toán SĐT (zalo): 521 Website: Môc lôc Trang PHẦN 1 Các chuyên đề hay và khó Chương 1 Phương trình Chương 2 Hệ phương trình Chương 3 Bất đẳng thức PHẦN 2 20 đề luyện tập có hướng dẫn chi tiết PHẦN 3 10 đề tự luyện
  • 3. Liên hệ tài liệu word toán SĐT (zalo): 1 Website: Chuyên đề 1. PHƯƠNG TRÌNH I. TÓM TẮT LÝ THUYẾT Đối với phương trình vô tỷ (tức là phương trình có chứa ẩn trong dấu căn), điều cần lưu ý nhất là tính không thuận nghịch của các phép toán. Chẳng hạn nếu trong một phương trình nào đó, bạn thay . A B (với A và B là các biểu thức nào đó của x ) bởi . A B thì tập xác định của phương trình rất có thể bị mở rộng, bởi vì . A B chỉ xác dịnh khi 0 A ≥ và 0 B ≥ trong khi . A B xác định ngay cả khi 0 A < và 0 B < . Vậy bạn chỉ thu được một phương trình hệ quả. Ngược lại, nếu thay thế . A B bởi . A B thì tập xác định có thể bị thu hẹp lại, do đó bạn rất dễ bị bỏ sót nghiệm. Điều đó cảnh báo rằng khi thực hiện một phép tính về căn thức, để biến đổi một phương trình thì nói chung bạn không được phương trình tương đương. Để tránh các sai sót kiểu như thế, người ta dùng một trong các cách sau: Cách 1: Nếu chắc chắn phép biến đổi chỉ cho phương trình hệ quả thì ở bước cuối cùng, ta dùng phép thử trực tiếp vào phương trình để loại bỏ nghiệm ngoại lai. Ví dụ: Giải phương trình 2 1 3 3 x x − + + = Giải: Phương trình đã cho, suy ra: ( ) ( )( ) 2 2 1 3 9 2 2 1 3 7 3 x x x x x − + + = ⇒ − + = − ( ) ( ) 2 2 2 1 4 2 5 3 7 3 62 61 0 61 x x x x x x x =  ⇒ + − = − ⇒ − + = ⇒  =  Thử trực tiếp vào phương trình, ta thấy 1 x = thỏa mãn, còn 61 x = không thỏa mãn. Vậy phương trình có nghiệm là: 1 x = . Cách 2: Ghi nhớ tập xác định của phương trình và các điều kiện cần thiết khác trước khi biến đổi phương trình. Nếu phép biến đổi dẫn đến phương trình hệ quả thì nghiệm ngoại lai chính là các giá trị của ẩn không nằm trong tập xác định hoặc không thỏa mãn các điều kiện đã nêu. Đôi khi, chính tập xác định và các điều kiện ấy sẽ đem lại những gợi ý hữu ích cho bạn trong quá trình giải phương trình. Ví dụ: Giải phương trình ( ) ( ) 2 3 2 1 x x x x x + −= +
  • 4. Liên hệ tài liệu word toán SĐT (zalo): 2 Website: Giải: Điều kiện ( ) ( ) 0 3 0 1 2 2 1 0 3 x x x x x x x =   − ≥    ⇔ ≤ −   + ≥    ≥  + Xét 0 x = , thỏa mãn phương trình + Xét 1 2 x ≤ − phương trình đã cho tương đương với . . 3 . 2 1 3 2 1 x x x x x x x x x − − + − − + = − − − ⇔ − + − + = − − ( ) ( ) 2 3 2 1 2 3 4 x x x x x ⇔ − + − + = − − ⇔ − − + = − (vô nghiệm vì giá trị của căn thức không thể bằng một số âm) + Xét 3 x ≥ , phương trình đã cho tương đương với . . 3 . 2 1 3 2 1 x x x x x x x x x + − = + ⇔ + − = + ( ) ( ) 2 2 4 3 2 1 2 3 4 3 4 0 1 x x x x x x x x x =  ⇔ + − = + ⇔ − = ⇔ − − = ⇔  = −  Nhận thấy 1 x = − không thỏa mãn 3 x ≥ nên bị loại. Vậy phương trình đã cho có nghiệm: 0; 4 x x = = Cách 3 Chú ý đến các điều kiện xác định của phương trình, các điều kiện để thực hiện các phép biến đổi đồng nhất hay biến đổi tương đương phương trình và đặt các điều kiện đó cùng với phương trình trong một hệ hỗn hợp (cả phương trình và bất phương trình). Hệ này sẽ tương đương với phương trình đã cho. Nhưng dù theo cách nào thì bạn cũng phải chú ý đến các điều kiện nảy sinh trong quá trình biến đổi phương trình, đặc biệt là sự thay đổi tập xác định của phương trình. Điều đó sẽ giúp bạn có những quyết định đúng đắn khi giải phương trình. Dưới đây là một số đồng nhất thức có điều kiện thường gặp: Đồng nhất thức Điều kiện ( ) 2 A A = 0 A ≥ . . A B A B = 0 A ≥ và 0 B ≥ A A B B = 0 A ≥ và 0 B > 2 . A B A B = 0 A ≥ và 0 B ≥ 2 . A B A B = − 0 A ≤ và 0 B ≥
  • 5. Liên hệ tài liệu word toán SĐT (zalo): 3 Website: * DẠNG 1: A B = Phương pháp: Sử dụng công thức của định nghĩa căn bậc hai số học 2 0 B A B A B ≥  = ⇔  =  Chú ý: Sau khi tìm nghiệm của bài toán xong, chúng ta nên thử lại nghiệm để tránh sai sót trong tính toán. Ví dụ 1: Giải phương trình: 2 4 3 2 5 x x x − + − = − Giải: Phương trình tương đương với ( ) 2 2 2 5 0 4 3 2 5 x x x x − ≥    − + − = −   2 5 5 2 14 2 2 5 5 24 28 0 14 5 x x x x x x x  ≥   ≥   ⇔ ⇔ ⇔ = =       − + =    =   Vậy phương trình có nghiệm là: 14 5 x = Ví dụ 2: Giải phương trình: 2 2 3 5 2 2 x x x + − = − Giải: Phương trình tương đương với ( ) 2 2 2 2 0 2 3 5 2 2 x x x x − ≥    + − = −   ( )( ) 1 1 1 1 9 1 2 9 0 9 2 2 x x x x x x x x ≥  =   ≥    =   ⇔ ⇔ ⇔    − − = =    =    . A B A B B = 0 A ≤ và 0 B > . A B A B B = − 0 A ≤ và 0 B <
  • 6. Liên hệ tài liệu word toán SĐT (zalo): 4 Website: Vậy phương trình có nghiệm là: 9 1; 2 x x = =
  • 7. Liên hệ tài liệu word toán SĐT (zalo): 5 Website: BÀI TẬP TƯƠNG TỰ 1) Giải phương trình: 2 3 4 3 1 x x x + + − = Đáp số: 3 105 16 x − + = 2) Giải phương trình: 2 2 6 2 x x x + − = − Đáp số: 5 3 x = 3) Giải phương trình: 2 2 3 x x x + + + = Đáp số: 1 x = 4) Giải phương trình: 2 2 3 1 0 x x x + + + + = Đáp số: 3 x = − * DẠNG 2: A B = Phương pháp: Phương trình tương đương với ( ) 0 0 A B A B ≥ ≥    =   Ví dụ : Giải phương trình: 2 3 x x x − = − Giải: Phương trình tương đương với 2 3 0 3 x x x x − ≥   − = −  2 3 3 3 3 3 x x x x x ≤ ≤    ⇔ ⇔ ⇔ = ±   = = ±    BÀI TẬP TƯƠNG TỰ: 1) Giải phương trình: 2 5 1 x x + = − Đáp số: 4 3 x = 2) Giải phương trình: 2 2 3 4 3 x x − = − Đáp số: 2 x = 3) Giải phương trình: 2 6 3 x x x − − = − Đáp số: 3 x =
  • 8. Liên hệ tài liệu word toán SĐT (zalo): 6 Website: * DẠNG 3: A B C + = Phương pháp: Bình phương 2 vế của phương trình ta được ( ) 2 . 2 C A B A B C A B − − + = ⇔ = (quay về dạng 1) Chú ý: Chỉ bình phương khi 2 vế của phương trình đều không âm. Ví dụ 1: Giải phương trình: 3 1 2 3 x x + + − = Giải: Điều kiện: 1 2 3 x − ≤ ≤ Phương trình tương đương với: ( )( ) 2 3 2 3 1 2 9 x x x + + + − = 2 2 2 3 0 3 5 2 3 3 5 2 6 9 x x x x x x x x − >  ⇔ − + + = − ⇔  − + + = − +  2 1 4 11 7 0 7 4 x x x x =   ⇔ − + = ⇔  =  Đối chiếu với điều kiện ta thu được nghiệm: 7 1; 4 x x = = Nhận xét: - Phương trình dạng: ( ) ( ) ( ) ( ) ( ) 2 2 0 f x g x m f x g x m + = > ⇔ + = ( ) ( ) ( ) ( ) ( ) ( ) ( ) ( ) 2 2 2 2 f x g x f x g x m f x g x m f x g x ⇔ + + = ⇔ = − − ( ) ( ) ( ) ( ) ( ) ( ) 2 1 2 2 2 4 m f x g x x x x x f x g x m f x g x  ≥ + =   ⇔ ⇒   =   = − −      - Phương trình trên có cách giải khác như sau: ( ) ( ) ( ) ( ) f x g x m f x m g x + = ⇔ = − ( ) ( ) ( ) ( ) ( ) ( ) ( ) ( ) 2 2 2 2 m f x m f x f x m m g x g x m g x g x m f x ≥ ≥     ⇔ ⇔   = − + = + −    
  • 9. Liên hệ tài liệu word toán SĐT (zalo): 7 Website: ( ) ( ) ( ) ( ) ( ) ( ) ( ) 2 1 2 2 2 2 ; 4 m f x g x m f x x x x x m g x g x m f x  ≥ + ≥ =   ⇔ ⇒   = = + −    Ý tưởng: Đây là một bài phương trình cơ bản, dạng toán một vế chưa hai căn thức vế còn lại là một hằng số thì phương pháp nâng lũy thừa hai vế là một phương pháp tối ưu nhất. Ví dụ 2: Giải phương trình: 5 1 2 3 14 7 x x x + + + = + Giải: Điều kiện: 5 1 0 1 2 3 0 5 14 7 0 x x x x + ≥   + ≥ ⇔ ≥ −   + ≥  Phương trình tương đương với: ( ) 2 5 1 2 3 14 7 x x x + + − = + 7 3 7 4 2 5 1. 2 3 14 7 5 1. 2 3 2 x x x x x x x + ⇔ + + + + = + ⇔ + + = ( )( ) 2 2 2 7 3 49 42 9 5 1 2 3 10 17 3 2 4 x x x x x x x + + +   ⇔ + + = ⇔ + + =     2 3 9 26 3 0 1 9 x x x x =   ⇔ − − = ⇔  = −  Vậy phương trình có nghiệm: 1 ; 3 9 x x = − = Nhận xét: Ở đây, khi Giải phương trình 7 3 5 1. 2 3 2 x x x + + + = chúng ta không cần đặt điều kiện 2 0 B A B ≥   =  (tức 7 3 0 2 x + ≥ ) vì khi 1 5 x ≥ − thì 7 3 0 2 x + > . Nhưng nếu chúng ta không nhận xét được vế phải thì chúng ta vẫn phải đặt điều kiện bình thường như ở dạng 1 Ví dụ 3: Giải phương trình: 3 3 5 2 4 x x x − − − = − Giải:
  • 10. Liên hệ tài liệu word toán SĐT (zalo): 8 Website: Điều kiện 3 3 0 5 0 2 5 2 4 0 x x x x − ≥   − ≥ ⇔ ≤ ≤   − ≥  Phương trình tương đương với: 3 3 2 4 5 x x x − = − + − ( ) 2 3 3 2 4 5 3 3 1 2 2 4. 5 x x x x x x x ⇔ − = − + − ⇔ − = + + − − ( ) ( )( ) 2 2 2 4. 5 2 2 4 5 x x x x x x ⇔ − = − − ⇔ − = − − 2 2 3 18 24 0 4 x x x x =  ⇔ − + = ⇔  =  Vậy phương trình có nghiệm: 2; 4 x x = = Nhận xét: Ở đây, khi giải phương trình: 2 2 4. 5 x x x − = − − chúng ta không cần đặt điều kiện 2 0 B A B ≥   =  (tức 2 0 x − ≥ ) vì khi 2 5 x ≤ ≤ Nhưng nếu chúng ta không nhận xét được vế trái thì chúng ta vẫn phải đặt điều kiện bình thường như ở dạng 1. BÀI TẬP TƯƠNG TỰ 1) Giải phương trình: 11 3 1 4 2 5 x x x + − + = − Đáp số: 3 x = 2) Giải phương trình: 5 1 3 2 1 x x x − − − = − Đáp số: 2 x = 3) Giải phương trình: 2 3 1 1 2 2 1 x x x + − − = − Đáp số: 5 x = 4) Giải phương trình: 3 1 1 8 x x + + + = Đáp số: 8 x = 5) Giải phương trình: 7 4 1 3 x x + − + = Đáp số: 3 x = * DẠNG 4: ( ) 2 2 2 2 0 x a x b a b x a x b a b cx d a + − + − + − − + − = + > Phương pháp: Đặt ( ) 0 t x b t = − ≥ , suy ra 2 x t b = + . Phương trình trở thành: ( ) 2 2 2 2 2 2 2 t at a t at a c t b d + + + − + = + +
  • 11. Liên hệ tài liệu word toán SĐT (zalo): 9 Website: ( ) ( ) 2 2 t a t a c t b d t a t a c t b d ⇔ + + − = + + ⇔ + + − = + + Sau đó, sử dụng định nghĩa trị tuyệt đối: ( ) ( ) 0 0 A A A A A ≥  =  − <   hoặc sử dụng phương pháp chia khoảng để giải. Ví dụ 1: Giải phương trình: 1 2 2 1 2 2 1 x x x x − + − − − − − = Giải: Điều kiện 2 2 0 2 2 6 9 0 1 2 2 0 x x x x x x x − ≥ ≥    ⇔ ⇔ ≥   − + ≥ − − − ≥    Đặt: ( ) 2 0 t x t = − ≥ , suy ra 2 1 1 x t − = + . Phương trình trở thành: 2 2 1 2 1 2 1 t t t t + + − + − = ( ) ( ) 2 2 1 1 1 1 1 1 1 1 1 1 2 t t t t t t t t t t t − =  + − − = ⇔ + − − = ⇔ − = ⇔ ⇔ =  − = −  1 9 2 2 4 x x ⇔ − = ⇔ = Vậy phương trình có nghiệm: 9 4 x = Ví dụ 2: Giải phương trình: 2 1 2 1 2 x x x x + − − − − = Giải: Điều kiện 2 1 0 1 1 4 4 0 2 1 0 x x x x x x x − ≥ ≥    ⇔ ⇔ ≥   − + ≥ − − ≥    Đặt ( ) 1 0 t x t = − ≥ , suy ra 2 1 x t = + Phương trình trở thành: 2 2 1 2 1 2 2 t t t t + + − + − = ( ) ( ) 2 2 1 1 2 1 1 2 1 1 1 0 t t t t t t t + − − = ⇔ + − − = ⇔ − = − ⇔ − ≥ 1 1 2 x x ⇔ − ≥ ⇔ ≥ Vậy phương trình có nghiệm với mọi 2 x ≥
  • 12. Liên hệ tài liệu word toán SĐT (zalo): 10 Website: BÀI TẬP TƯƠNG TỰ 1) Giải phương trình: 14 49 14 49 14 x x x x + − + − − = Đáp số: 7 ; 7 2 x   ∈     . 2) Giải phương trình: 3 2 1 2 1 2 x x x x x + + − + − − = Đáp số: 1; 5 x x = = 3) Giải phương trình: 4 4 4 4 4 x x x x + − + − − = Đáp số: [ ] 4; 8 x ∈ * DẠNG 5: ( ) ( ) . . a f x b c f x d e + + + = Phương pháp: Đặt ( ) t f x = , Phương trình trở thành: . . a t b c t d e + + + = Sau đó bình phương hai vế đưa về dạng A B = Ví dụ: Giải phương trình: 2 2 3 6 2 6 5 9 x x x x + + + + + = Giải: Điều kiện x ∈  . Phương trình tương đương với: ( ) ( ) 2 2 3 6 2 3 5 9 x x x x + + + + + = (1) Đặt ( ) 2 5 3 * 2 t x x t   = + ≥ −     Phương trình (1) trở thành: 6 2 5 9 t t + + + = ( ) 2 6 2 5 81 3 11 2 6. 2 5 81 t t t t t + + + = ⇔ + + + + = ( )( ) ( ) 2 70 3 0 2 6. 2 5 70 3 4 6 2 5 70 3 t t t t t t t − ≥   ⇔ + + = − ⇔  + + = −   ( )( ) ( ) 2 2 2 70 70 3 3 8 68 120 4900 420 9 4 6 2 5 70 3 t t t t t t t t t   ≤ ≤   ⇔ ⇔     + + = − + ⇔ + + = −  
  • 13. Liên hệ tài liệu word toán SĐT (zalo): 11 Website: 2 70 70 3 10 3 478 488 4780 0 10 t t t t t t t  ≤   ≤   ⇔ ⇔ ⇔ =   =   ⇔ − + =    =   (thỏa mãn điều kiện (*)) Với 2 2 10 3 10 5 x t x x x =  = ⇒ + = ⇔  = −  Vậy phương trình có nghiệm: 5; 2 x x = − = . * DẠNG 6: ( ) ( ) . . a f x b c f x d + = + Phương pháp: Đặt ( ) ( ) . 0 t c f x d t = + ≥ , suy ra ( ) 2 t d f x c − = Phương trình trở thành: 2 2 0 t d a b t at ct ad bc c   − + = ⇔ − − + =     . Giải phương trình: này và sau đó thế lại tìm ẩn x Ví dụ: Giải phương trình: 2 2 2 2 4 8 20 x x x x + = + + + Giải: Điều kiện x ∈  . Đặt ( ) 2 2 4 8 0 t x x t = + + ≥ , suy ra 2 2 8 2 2 t x x − + = Phương trình trở thành: 2 8 20 2 t t − = + 2 6 2 48 0 8 t t t t = −  ⇔ − − = ⇔  =  + Với 6 t = − , không thỏa mãn điều kiện. + Với 8 t = , thỏa mãn điều kiện nên ta có 2 2 4 8 8 x x + + = 2 2 2 4 8 64 2 4 56 0 1 29 x x x x x ⇔ + + = ⇔ + − =⇔ = − ± Vậy phương trình có nghiệm: 1 29 x =− ± * DẠNG 7: ( )( ) a x b x c a x b x d + + − + + − =
  • 14. Liên hệ tài liệu word toán SĐT (zalo): 12 Website: Phương pháp: Đặt ( ) 0 t a x b x t = + + − ≥ , suy ra 2 . 2 t a b a x b x − − = + − . Phương trình trở thành: 2 . 2 t a b t c d − − + = 2 2 2 0 ct t ca cb d ⇔ + − − − = Giải phương trình: này và sau đó thế lại tìm ẩn x Ví dụ: Giải phương trình: ( )( ) 1 3 1 3 2 x x x x + + − − + − = Giải: Điều kiện 1 0 1 3 3 0 x x x + ≥  ⇔ − ≤ ≤  − ≥  Đặt ( ) 1 3 0 t x x t = + + − ≥ , suy ra 2 4 1. 3 2 t x x − + − = . Phương trình trở thành: 2 4 2 2 t t − − = 2 0 2 0 2 t t t t =  ⇔ − + = ⇔  =  . + Với 1 0 0 1 3 0 3 0 x t x x x + =  = ⇔ + + − = ⇔  − =  (vô nghiệm). + Với 1 2 1. 3 0 3 x t x x x = −  = ⇔ + − = ⇔  =  (thỏa mãn điều kiện) Vậy phương trình có nghiệm: 1; 3 x x = − = * DẠNG PHỨC TẠP Ví dụ 1: Giải phương trình: ( )( ) 2 1 1 2 2 1 8 x x x + + − + − = Giải: Điều kiện 1 1 x − ≤ ≤ Đặt: 1 1 t x x = + + −
  • 15. Liên hệ tài liệu word toán SĐT (zalo): 13 Website: ( )( ) 2 2 1 2 1 1 1 2 2 1 t x x x x x ⇒ = + + + − + − = + − . Khi đó phương trình đã cho trở thành 2 . 8 t t = 3 2 2 8 2 2 2 1 4 1 1 0 t t x x x ⇔ = ⇔ = ⇔ + − = ⇔ − = ⇔ = Vậy phương trình đã cho có nghiệm duy nhất là 0 x = . Nhận xét: Này toán sử dụng phương pháp đặt ẩn phụ Giải phương trình * NHẮC LẠI KIẾN THỨC VÀ PHƯƠNG PHÁP - Hằng đẳng thức quen thuộc: ( ) 2 2 2 2 u v u uv v + = + + . - Với ( ) [ ; ] f x a b ∈ − , đặt ( ) ( ) t a f x b f x = + + − , khi đó ( ) ( ) ( ) ( ) ( ) ( ) ( ) ( ) 2 2 2 2 t a b t a b a f x b f x a f x b f x − − = + + + − ⇔ + − = Ý tưởng: Nhận thấy ở hai căn thức, ta có tổng bình phương của chúng là một hằng số, mặt khác tích của chúng có liên quan đến biểu thức còn lại trong phương trình. - Ta có: ( ) ( ) ( )( ) 2 2 2 1 1 1 1 2; 2 1 2 1 1 x x x x x x x + + − = + + − = − = + − - Do đó: ( ) ( ) ( )( ) 2 2 2 2 2 1 1 1 2 1 1 x x x x x + − = + + − + + − ( ) 2 1 1 . x x = + + − - Đặt 1 1 t x x = + + − , phương trình đã cho trở thành: 3 8 t = 2 2 2 2 2 1 4 1 1 0 t x x x ⇔ = ⇔ + − = ⇔ − = ⇔ = Bài toán kết thúc. BÀI TẬP TƯƠNG TỰ 1) Giải phương trình: ( ) 2 6 2 4 3 2 2 x x x + − = + + − Đáp số: 2 x = ± . 2) Giải phương trình: ( )( ) 7 2 2 4 3 2 2 4 4 3 x x x x + − + = − + + Đáp số: 5 4 3 4 x ± = − Ví dụ 2: Giải phương trình: ( ) 2 2 2 2 4 2 3 x x x x + + − + − = −
  • 16. Liên hệ tài liệu word toán SĐT (zalo): 14 Website: Giải: Điều kiện 2 x ≥ . Đặt: 2 2 0 t x x = + + − > 2 2 2 2 2 2 4 2 2 4 t x x x x x ⇒ = + + − + − =+ − Phương trình đã cho tương đương: 2 2 2 2 2 4 6 x x x x + + − + + − = 2 6 2 t t t ⇔ + − ⇔ = hoặc 3 t = − (loại). Với 2 t = ta có 2 2 2 x x + + − = Do điều kiện 2 x ≥ , ta có: 2 2 4 0 2 x x + + − ≥ + = Suy ra phương trình có nghiệm duy nhất 2 x = . Nhận xét: Bài toán sử dụng phương pháp đặt ẩn phụ, đưa phương trình ban đầu về phương trình bậc hai tìm ẩn, sau đó dùng phương pháp nâng lũy thừa tìm nghiệm của phương trình ban đầu. * NHẮC LẠI KIẾN THỨC VÀ PHƯƠNG PHÁP - Cách giải phương trình bậc hai tổng quát: 2 . . 0 a t b t c + + = - Hằng đẳng thức: ( )( ) 2 2 a b a b a b − = − + và ( ) 2 2 2 2 a b a ab b + = + + - Phương trình có dạng: ( ) ( ) f x g x m + = , với m là số thực dương thì có hai cách nâng lũy thừa như sau: Cách 1. Bình phương hai vế của phương trình, ta có: ( ) ( ) ( ) ( ) ( ) ( ) ( ) ( ) 2 0; 0 2 . f x g x f x g x m f x g x f x g x m ≥ ≥   + = ⇔  + + =   ( ) ( ) ( ) ( ) ( ) ( ) 1 2 2 2 0; 0 . 4 . f x g x x x x x f x g x m f x g x ≥ ≥  =   ⇔ ⇒   =   = − −      Cách 2. Chuyển ( ) g x sang VP rồi bình phương, ta có: ( ) ( ) ( ) ( ) ( ) ( ) 2 2 m g x f x m g x f x m m g x g x  ≥  = − ⇔  = − +   ( ) ( ) ( ) ( ) ( ) ( ) ( ) ( ) 2 2 2 2 . 2 4 m g x m g x m g x m f x g x m g x m f x g x   ≥ ≥   ⇔ ⇔     = − + = − +       Chú ý: - Nếu ( ) ( ) f x g x k − = ; k là hằng số thì ta có thể sử dụng cách liên hợp như sau:
  • 17. Liên hệ tài liệu word toán SĐT (zalo): 15 Website: ( ) ( ) f x g x m + = (i) ( ) ( ) ( ) ( ) ( ) ( ) ( ) ( ) ( ) f x g x f x g x m f x g x ⇔ + − = − ( ) ( ) ( ) ( ) ( ) ( ) ( ) k f x g x m f x g x f x g x m ⇔ − = − ⇔ − = (ii) Lấy (i) + (ii), ta được ( ) ( ) 2 2 4 k k f x m f x m m m   = + ⇔ = +     . - Nếu c a b ≥ ≥ và x c ≥ suy ra x a x b c a c b + + + ≥ + + + . Dấu “=” xảy ra khi và chỉ khi x c = . Ý tưởng: Bài toán xuất hiện ba căn thức bậc hai ở VT là: 2, 2 x x + − và 2 4 x − , áp dụng hằng đẳng thức ( )( ) 2 2 a b a b a b − = − + dễ thấy được rằng 2 4 2. 2 x x x − = − + , hay nói cách khác căn thức cuối cùng chính là tích của hai căn thức còn lại. Đồng thời nếu chuyển ( ) 2 3 x − từ VP sang VT thì sẽ xuất hiện 2x , mà ( ) ( ) 2 2 2 2 2 2 2 x x x x x = + + − = + + − do đó VT của phương trình ban đầu có: ( ) ( ) 2 2 2 2 2. 2. 2 2 2 6 0 x x x x x x + + − + − + + + + − − = ( ) 2 2 2 2 2 6 0 x x x x ⇔ + + − + + + − − = (*) - Đặt 2 2 0 t x x = + + − > thì phương trình (*) được viết lại thành: ( )( ) 2 0 0 2 2 3 0 6 0 t t t t t t t > >    ⇔ ⇔ =   − − = + − =    - Với 2 t = suy ra 2 2 2 2 2 2 2 4 4 x x x x x ≥   + + − = ⇔  + − =   2 2 2 4 2 x x x x ≥   ⇔ ⇔ =  − = −   Đến đây có thể đánh giá như lời giải là: 2 2 2 4 0 2 2 x x x x ≥ ⇒ + + − ≥ + = ⇒ = . - Vì ( ) 2 2 4 x x + − − = nên giải phương trình 2 2 2 x x + + − = theo chú ý như sau: ( ) 2 2 2 4 2 2 x x x x + + − = ⇔ = + + − 2 2 2 2 2 4 2 x x x x ⇔ + − = ⇒ + = ⇔ = Bài toán kết thức. BÀI TẬP TƯƠNG TỰ
  • 18. Liên hệ tài liệu word toán SĐT (zalo): 16 Website: 1) Giải phương trình: 2 2 1 2 1 2 1 3 2 x x x x + + − + − = − Đáp số: phương trình vô nghiệp thực. 2) Giải phương trình: 2 2 2 5 2 2 5 1 5 2 x x x x x + + + − = − + + Đáp số: 17 16 x = Ví dụ 3: Giải phương trình: 2 3 1 3 1 1 x x x x + + − = + + − Giải: Điều kiện 1 1 x − ≤ ≤ . Phương trình tương đương với: ( ) 2 2 2 1 1 1 1 2 1 x x x x x + + + − = + + − + + ( ) ( ) 1 1 2 1 2 1 1 1 x x x x x x ⇔ + + − + + = + + + + − ( )( ) 1 1 2 1 1 0 x x x ⇔ + + − − + − = + Giải 2 2 1 1 2 2 2 1 4 1 1 0 x x x x x + + − = ⇔ + − = ⇔ − = ⇔ = + Giải 1 1 0 x x + = ⇔ = Đáp số 0 x = Nhận xét: Bài toán sử dụng phương pháp nhóm nhân tử chung, sau đó nâng lũy thừa bậc hai để tìm nghiệm của phương trình. * NHẮC LẠI KIẾN THỨC VÀ PHƯƠNG PHÁP - Giải phương trình: ( ) ( ) ( ) ( ) 0 . 0 0 f x f x g x g x =  = ⇔  =   . - Giải phương trình: ( ) ( ) a f x a f x b − + + = ( ) ( ) ( ) ( ) ( ) 2 2 2 2 2 2 2 . 2 2 4 2 a f x a a f x a b a a f x a f x b a ≥ ≥ −  ≥ ≥ −    ⇔ = ⇔     + − − =−       Ý tưởng: Bài toán xuất hiện ba căn thức, nhưng có điều đặc biệt ở đây là căn thức còn lại là tích của hai căn thức kia. Mặt khác 2 1 , 1 x x − − có sự đồng nhất hệ số, do đó ta sẽ nhóm hai căn này lại nên ta được nhân tử chung như sau: ( ) 2 1 1 1 1 1 x x x x − − − = − + − . Và ta mong muốn biểu thức
  • 19. Liên hệ tài liệu word toán SĐT (zalo): 17 Website: 3 3 1 x x + − + sẽ phân tích được biểu thức có chứa 1 1 x + − . Thật vậy nếu coi ( ) 3 3 1 h x x x = + − + là một phương trình bậc hai ẩn 1 x + ta sẽ thấy: ( ) ( )( ) 1 3 1 2 1 1 1 2 h x x x x x = + − + + = + − + − . Chính vì thế bài toán của ta được giải quyết như sau: 2 3 1 3 1 1 x x x x + + − = + + − ( )( ) ( ) 1 1 1 2 1 1 1 0 x x x x ⇔ + − + − + − + − = ( )( ) 1 1 1 1 1 1 2 0 1 1 2 x x x x x x  + = ⇔ + − + + − − = ⇔  + + − =   . Phần còn lại chỉ là việc bình phương các phương trình và tìm nghiệm như ở trên đã nêu. Ta được nghiệm của phương trình là 0 x = Bài toán kết thúc. BÀI TẬP TƯƠNG TỰ 1) Giải phương trình: 2 4 4 3 2 2 x x x x + + − = + + − Đáp số: 1 x = 2) Giải phương trình: 2 2 3 1 4 3 2 1 1 2 x x x x + + − = + + − Đáp số: 0 x = Ví dụ: Giải phương trình: 3 3 1 1 x x x + + + = − Giải: Phương trình tương đương với: 2 3 2 3 1 2 2 x x x + + + = − . Đặt 2 2 3 2 2 3 1 u x v u x v x  = +  ⇒ − = −  = +   với ; 0 u v > , ta được ( ) ( )( ) 2 2 2 2 0 u v v u v u v u + = − ⇔ + − − = 2 3 1 3 2 3 1 7 4 3 v u x x x x x ⇔ = + ⇔ + = + + ⇔ + = + + + ( ) 2 3 2 6 4 3 2 3 3 4 3 6 9 x x x x x x x x ≥   ⇔ − = + ⇔ + = − ⇔  + = − +   2 3 5 28 5 28 10 3 0 3 x x x x x x ≥   = ±  ⇔ ⇔ ⇔ = +   − − = ≥    Nhận xét: Bài toán sử dụng phương pháp đặt ẩn phụ để đưa phương trình từ phức tạp về đơn giản hóa, sau đó dùng phương pháp nâng lũy thừa để tìm nghiệm của phương trình.
  • 20. Liên hệ tài liệu word toán SĐT (zalo): 18 Website: * NHẮC LẠI KIẾN THỨC VÀ PHƯƠNG PHÁP: - Hằng đẳng thức cơ bản: ( )( ) 2 2 a b a b a b − = − + . - Cách giải phương trình vô tỷ dạng: ( ) ( ) f x g x m = + ( ) ( ) ( ) ( ) ( ) ( ) ( ) ( ) ( ) ( ) 2 2 ; 0 ; 0 2 f x g x f x g x f x g x m m g x f x g x m  ≥ ≥    ⇔ ⇔   − − = = +     ( ) ( ) ( ) ( ) ( ) 1 2 2 2 2 ; 0 . 4 f x g x x x x x f x g x m m g x ≥  =   ⇔ ⇔   =   − − =     Ý tưởng: Một bài toán đẹp, quan sát VT của phương trình có xuất hiện hai căn bậc hai riêng biệt đồng thời trong căn chứa các biểu thức bậc nhất, cũng như VP của phương trình cũng là một biểu thức bậc nhất, nên vậy ta có thể nâng lũy thừa để đưa phương trình ban đầu về phương trình bậc bốn. Nhưng nếu tinh ý một chút, ta có ( ) ( ) ( ) 3 1 3 2 2 2 1 x x x x + − + = − = − , Do vậy đặt 3 3 1 u x v x  = +   = +   với ; 0 u v ≥ , suy ra: ( ) 2 2 2 2 2 1 v u x x − = − = − . Khi đó, phương trình đã cho tương đương với: ( ) ( )( ) 2 2 2 2 0 v u v u v u v u − = + ⇔ + − − = (i) - Vì ; 0 u v ≥ nên phương trình (i) 0 2 u v v u = =  ⇔  = +  , với 0 u v = = phương trình vô nghiệm nên ta chỉ cần giải phương trình: 2 v u = + . - Với 2 v u = + , ta có: 3 1 3 2 3 1 7 4 3 x x x x x + = + + ⇔ + = + + + ( ) 2 3 2 6 4 3 2 3 3 4 3 6 9 x x x x x x x x ≥   ⇔ − = + ⇔ + = − ⇔  + = − +   2 3 5 28 5 28 10 3 0 3 x x x x x x ≥   = ±  ⇔ ⇔ ⇔ = ±   − − = ≥    Bài toán kết thúc. BẢI TẬP TƯƠNG TỰ 1) Giải phương trình: 4 1 2 2 2 1 x x x + + + = − Đáp số: 2 5 x= + 2) Giải phương trình: 3 5 2 1 4 x x x + + + = −
  • 21. Liên hệ tài liệu word toán SĐT (zalo): 19 Website: Đáp số: 5 87 8 x − = Ví dụ 5: Giải phương trình: ( ) 2 3 7 2 1 x x x x + + = + Giải: Điều kiện 0 x > Phương trình tương đương: ( ) 2 3 2 1 7 x x x x + + = + Chia hai vế cho 0 x ≠ , ta được: 1 3 7 3 1 3 4 3 3 2 2 1 2 1 0 2 0 x x x x x x x x x x x x x x x x          + + = + ⇔ + − + + + = ⇔ + − + − =                      + Giải: 2 1 3 3 2 4 4 3 0 3 x x x x x x x x =  + = ⇔ + = ⇔ − + = ⇔  =  + Giải: 2 2 3 2 3 4 3 4 0 x x x x x x x x + = ⇔ + = ⇔ + − = ( )( ) 2 1 4 0 1 x x x x ⇔ − + + = ⇔ = Đáp số 1; 3 x x = = . Nhận xét: Bài toán sử dụng phương pháp ẩn phụ không hoàn toàn, sau đó nâng lũy thừa tìm nghiệm của phương trình ban đầu. * NHẮC LẠI KIẾN THỨC VÀ PHƯƠNG PHÁP - Phương pháp đặt ẩn phụ không hoàn toàn: xét một phương trình bậc hai có dạng ( ) ( ) 2 . . 0 mf x t ng x t k + + =(*), trong đó t là ẩn phụ được biểu diễn dưới dạng ( ) t h x = . Khi đó, ta có: ( ) ( ) 2 4 t ng x kmf x = −     Δ , với t Δ bắt buộc là một số chính phương. Do đó, tìm được nghiệm của (*), đó là ( ) ( ) ( ) ( ) ( ) ( ) ; t t ng x ng x t h x t h x mf x mf x − + − − = = = = Δ Δ - Cách giải phương trình: ( ) ( ) ( ) ( ) ( ) ( ) 2 ; 0 . f x g x f x g x f x g x ≥   = ⇔  =   .
  • 22. Liên hệ tài liệu word toán SĐT (zalo): 20 Website: Ý tưởng: Trước hết, ta cần quy đồng mẫu số bài toán, như vậy ta sẽ được phương trình có dạng ( ) ( ) ( ) . f x g x h x = và nếu nâng lũy thừa hai vế, ta sẽ thu được một phương trình bậc 5. Và phương trình bậc 5 nếu không có nghiệm nguyên thì sẽ rất khó để giải quyết. Vậy nên ta cần nghĩ đến hướng tư duy khác, đó là bài toán xuất hiện căn thức 3 x x + nên ta mong muốn sẽ tạo được lượng 2 3 k x x   +       để có thể đưa về phương trình bậc hai, sau đó đặt 3 t x x = + để sử dụng phương pháp ẩn phụ không an toàn. - Ta có: ( ) ( ) 2 2 3 7 3 2 1 7 2 1 x x x x x x x x + + = ⇔ + + = + + Chia cả hai vế của phương trình cho x, ta có: 1 3 7 3 1 3 4 2 1 2 1 0 x x x x x x x x x x x     + + = + ⇔ + − + + + =         (*) - Đặt 3 0 t x x = + > , khi đó ta có ( ) 2 1 4 * 2 1 0 t t x x   ⇔ − + + =     . Có: 2 2 1 4 1 1 1 t x x x     = + − = −         Δ nên suy ra được ( ) ( ) 3 1 1 2 2 1 1 2 1 1 3 2 1 1 x i t t x x x t t x x ii x x x x   + = = = + + −      ⇔ ⇔    =  =+ + − +  + =     - Giải (i), ta có (i) 3 1 3 4 4 3 0 3 x x x x x x =  ⇔ + = ⇔ − + = ⇔  =  - Giải (ii), ta có (ii) 3 2 3 4 3 4 0 1 x x x x x x ⇔ + = ⇔ + − = ⇔ = Bài toán kết thức. BÀI TẬP TƯƠNG TỰ: 1) Giải phương trình: ( ) ( )( ) 3 4 12 28 x x x x + − + = − Đáp số: ( ) 4 2 1 ; 31 3 x x = − = − 2) Giải phương trình: 3 2 2 2 x x x x − = − − Đáp số: 1 5 1 65 ; . 2 8 x x + − = =
  • 23. Liên hệ tài liệu word toán SĐT (zalo): 21 Website: Ví dụ 6: Giải phương trình: ( )( ) 9 2012 6 2012 9 6 . x x x x + + += + + + Giải: Điều kiện 6 x ≥ − Phương trình đã cho tương đương với: ( )( ) 9 2012 6 1 0 x x + − + − = + Giải ( ) 2 9 2012 0 2012 9 4048135 x x + − = ⇔ = − = + Giải 6 1 0 5 x x + − = ⇔ = − Vậy phương trình có hai nghiệm: 4048135; 5 x x = = − Nhận xét: Bài toán sử dụng phương pháp nhóm nhân tử chung và nâng lũy thừa tìm nghiệm của phương trình. * NHẮC LẠI KIẾN THỨC VÀ PHƯƠNG PHÁP: - Các giải phương trình dạng: ( ) ( ) ( ) ( ) ( ) ( ) ( ) ( ) 2 2 . 0 . f x m f x m f x m g x n g x n g x n  =  =  − − = ⇔ ⇔   =  =   Ý tưởng: Bài toán cho hết sức đơn giản, với sự xuất hiện của hai căn thức ( )( ) 9; 6; 9 6 x x x x + + + + nên không khó để nhóm được nhân tử chung như sau: ( )( ) 9 2012 6 2012 9 6 x x x x + + += + + + ( )( ) 9 9 6 2012 6 2012 0 x x x x ⇔ + − + + + + − = ( ) ( ) 9 1 6 2012 1 6 0 x x x ⇔ + − + − − + = ( )( ) 1 6 9 2012 0 x x ⇔ − + + − = Bài toán kết thúc. BÀI TẬP TƯƠNG TỰ 1) Giải phương trình: 2 2 6 7 2 13 42 x x x x + + + = + + + Đáp số: 3; 5 x x = − = − 2) Giải phương trình: 2 4 2 3 2 7 12 x x x x + + + = + + + Đáp số: 0; 2 x x = = −
  • 24. Liên hệ tài liệu word toán SĐT (zalo): 22 Website: Ví dụ 7: Giải phương trình: 8 2 2 1 x x + − − = Giải: Điều kiện 8 2 x − ≤ ≤ + Nếu 1 2 8 9 3 x x < ≤ ⇒ + > = 2 2 1 1 8 2 2 3 2 1 x x x − < − = ⇒ + − − > − = ⇒ loại. + Nếu 8 1 x − ≤ < , tương tự ta có: 8 2 2 9 2 1 1 x x + − − < − ==> loại Với 1 x = , thỏa mãn phương trình. Đáp số 1 x = . Nhận xét: Bài toán sử dụng phương pháp nhẩm nghiệm và đánh giá theo miền nghiệm để chứng minh nó có nghiệm duy nhất. Ý tưởng: Đây một bài toán đơn giản nhưng đòi hỏi đi theo phương pháp đánh giá thì cần đoán trước nghiệm của phương trình. Đầu tiên ta sẽ ưu tiên nghiệm nguyên trước, với nghiệm nguyên thì các biểu thức chứa căn phải là một số chính phương, tức là 2 2 8 2 x k x h  + = ∈   − = ∈     . Với điều kiện chặn của [ ] 8; 2 x ∈ − ] thì ta sẽ thử một vài giá trị nguyên của x và thấy rằng tại 1 x = thỏa mãn phương trình. Công việc còn lại là ta sẽ đi chứng minh nó là nghiệm duy nhất. Nó là nghiệm duy nhất nếu với [ 8; 1) (1; 2] x ∈ − ∪ thì phương trình bài cho vô nghiệm. Đi xét từng trường hợp ta có: - Với [ 8; 1) x ∈ − suy ra 8 2 2 9 2 1 1 x x + − − < − =. - Với (1; 2] x ∈ suy ra 8 2 2 9 2 1 1 x x + − − > − =. Cả hai trường hợp trên đều chứng minh 1 x = là nghiệm duy nhất của phương trình. Bài toán kết thúc.
  • 25. Liên hệ tài liệu word toán SĐT (zalo): 23 Website: BÀI TẬP TƯƠNG TỰ 1) Giải phương trình: 7 8 3 4 x x x + − = + 2) Giải phương trình: 9 2 1 1 x x + − − = Ví dụ 8: Giải phương trình: ( )( ) 3 1 1 1 x x x + − − + = Giải: Điều kiện 0 1 x ≤ ≤ Phương trình tương đương với: ( ) ( ) 3 1 1 1 3 1 1 3 3 x x x x x x − + = ⇔ − + = + + + + Nếu ( ) 0 1 3 1 1 3 x x ≤ ≤ ⇒ − + > , đồng thời 3 1 4 3 x x + + < + =, suy ra VT > VP (loại). Thử lại ta thấy 1 x = là nghiệm. Nhận xét: Bài toán kết hợp giữa phương pháp nhân liên hợp và phương pháp đáng giá để tìm nghiệm của phương trình. * NHẮC LẠI KIẾN THỨC VÀ PHƯƠNG PHÁP - Biểu thức liên hợp: ( )( ) x m x x m x x m x + − = + − + + m x m x x m x + − = + + với 0; 0 x x m ≥ + ≥ . - Đánh giá: ( ) ( ) ( ) m f x n g x h x − + = + với ( ) 0 2 m f x n m ≥ ≥    =   . Ta có: ( ) ( ) ( ) ( ) 0 2 m f x n n f x m g x h x m  − + >  ≤ < ⇒  + <   , suy ra phương trình vô nghiệm. Vậy x m = là nghiệm của phương trình đã cho Ý tưởng: Bài toán xuất hiện ba căn thức nằm trong một tích, sẽ rất khó để định hình ra hướng giải, ẩn phụ sẽ rất phức tạp. Nhưng nếu xét hai căn thức đầu tiên ta thấy ( ) ( ) 2 2 3 3 x x − − = . Vì thế ta sẽ nghĩ ngay đến chuyện dùng hằng đẳng thức dạng: ( )( ) 2 2 a b a b a b − = − + . Khi đó phương trình đã cho tương đương với:
  • 26. Liên hệ tài liệu word toán SĐT (zalo): 24 Website: ( ) 3 1 1 1 3 1 3 3 3 x x x x x x − + = ⇔ − + = + + + + (i) Với phương trình (i), ta sẽ đi nhẩm một vài giá trị nghiệm đẹp thỏa mãn các yêu cầu là 1 0 x ≥ ≥ và các biểu thức trong căn thức là số chính phương vì thế ta khẳng định nó có nghiệm duy nhất 1 x = , đồng thời 1 lại là miền chặn của biến do đó ta sẽ đi đánh giá phương trình (i). Tức là với 0 1 x ≤ < , ta sẽ chứng minh (i) vô nghiệm như sau: 3 1 3 3 0 1 3 3 x x x x  − + >  ≤ < ⇒  + + <   => VT > VP => (i) vô nghiệm Vậy ta kết luận 1 x = là nghiệm của phương trình đã cho. Bài toán kết thúc. BÀI TẬP TƯƠNG TỰ 1) Giải phương trình: ( )( ) 2 1 2 1 1 x x x + − − − + = Đáp số: 2 x = 2) Giải phương trình: ( )( ) 1 2 3 1 1 x x x + − − − + = Ví dụ 9: Giải phương trình: ( )( ) 4 2 4 2 2 x x x + − − + = Giải: Điều kiện 4 4 x − ≤ ≤ Phương trình đã cho tương đương với: ( ) 4 2 2 4 2 x x x x − + = + + . + Với 0 x = là nghiệm. + Giải: ( ) 4 2 2 4 2 x x − += + + Đặt 4; 4 u x v x = + = − ta thu được ( ) 2 2 2 2 2 2 2 2 2 8 5 8 4 0 8 v u u u u u u v = +  ⇒ + + = ⇔ + − =  + =  ( ) 2 14 ; 2 96 5 5 4 5 25 2 u v x x u l  = =  ⇔ ⇒ + = ⇔ = −  = −   (thỏa mãn) Vậy phương trình có hai nghiệm: 96 0; 25 x x = = − . Nhận xét: Sử dụng phương pháp nhân liên hợp, sau đó đặt ẩn phụ tìm nghiệm của bài toán.
  • 27. Liên hệ tài liệu word toán SĐT (zalo): 25 Website: NHẮC LẠI KIẾN THỨC VÀ PHƯƠNG PHÁP - Hằng đẳng thức ( )( ) a b a b a b a b a b a b − − = − + ⇔ + = − . - Giải phương trình: tổng quát dạng ( ) ( ) f x g x m = + ( ) ( ) ( ) ( ) ( ) ( ) ( ) ( ) ( ) ( ) 2 2 2 2 ; 0 2 4 f x g x m f x g x f x g x m g x m f x g x m mg x  ≥ + ≥    ⇔ ⇔   = + +   − − =       Ý tưởng: Không khó để nhận thấy, phương trình có một nghiệm là 0 x = . Đồng thời vế trái của phương trình có xuất hiện biểu thức 4 2 x + − , dễ thấy rằng ( ) ( )( ) 2 2 4 2 4 2 4 2 x x x x = + − = + − + + . Vì thế, phương trình đã cho tương đương với: ( )( ) ( )( ) 4 2 0 0 4 2 4 2 2 4 2 4 2 4 2 4 2 x x x x x x x x  + − = ⇔ = + − − + = + − + + ⇔  − = + +   Phương trình còn lại có thể giải bằng cách tổng quát nêu ở trên, hoặc có thể giải quyết bằng cách đặt ẩn phụ như sau: ( ) 4 ; 0 4 v x u v u x  = −  ≥  = +   . Ta có hệ phương trình 2 2 8 2 14 2 96 ; 4 5 5 5 25 2 2 v u u v x x v u  + = ⇔ = = ⇒ + = ⇔ = −  = +  . Bài toán kết thúc. BÀI TẬP TƯƠNG TỰ 1) Giải phương trình: ( )( ) 1 1 1 1 2 x x x + − − + = . Đáp số: 24 0; 25 x x = = − 2) Giải phương trình: ( )( ) 9 1 9 1 3 x x x + − − + = Đáp số: 0 x = Ví dụ 10: Giải phương trình: 1 1 4 1 5 2 1 2 x x x x + + = + + + Giải:
  • 28. Liên hệ tài liệu word toán SĐT (zalo): 26 Website: Phương trình tương đương với: 1 1 5 4 1 2 1 2 x x x x − = − − + + . Giả sử: 1 1 1 2 1 2 2 1 2 x x x x x > ⇒ + > + ⇒ < + + => 0 5 4 1 1 VT x x x < ⇒ < − ⇒ < (mâu thuẫn). Giả sử 0 1 x ≤ < , lập luận tương tự thu được 1 x > (mâu thuẫn). Thử lại thu được nghiệm duy nhất 1 x = Nhận xét: Bài toán sử dụng phương pháp đánh giá miền nghiệm để tìm nghiệm của phương trình. Ý tưởng: Quan sát bài toán, ta thấy có vẻ nó hơi rắc rối một chút vì xuất hiện tới bốn căn thức mà căn lại còn ở dưới mẫu thì sẽ rất khó khăn trong việc giải bằng các công cụ mà ta thường làm như ẩn phụ, nâng lũy thừa...Và hướng cuối cùng ta nghĩ đến là đánh giá. Để đánh giá được nó, ta cần tìm nghiệm trước, vẫn là ưu tiên hướng nghiệm nguyên đồng thời cân bằng căn thức với căn thức, phân thức với phân thức ta có: 1 1 1; 4 1 5 1 2 1 2 x x x x x x = ⇔ = + = ⇔ = + + Và bây giờ, ta sẽ đi chứng minh 1 x = là nghiệm duy nhất của phương trình. Hay nói cách khác, với 1 x > hoặc 1 x < nó sẽ vô nghiệm. Biến đổi phương trình đã cho về dạng: ( )( ) 1 1 2 2 1 4 1 5 5 4 1 2 1 2 2 1 2 x x x x x x x x x x + − + + += + ⇔ = − + + + + + * Với 1 x > , ta có 2 2 1 2 2 1 0 5 4 1 5 4 1 0 x x x x x x x x  + < + + − − <   ⇔   > + − + >    , suy ra phương trình vô nghiệm. * Với 1 0 x > ≥ , ta có: 2 2 1 2 2 1 0 5 4 1 5 4 1 0 x x x x x x x x  + > + + − + >   ⇔   < + − + <    , suy ra phương trình vô nghiệm. Vậy 1 x = là nghiệm duy nhất của phương trình. Bài toán kết thúc. BÀI TẬP TƯƠNG TỰ 1) Giải phương trình: 1 1 3 1 4 2 1 2 1 x x x x + + = + + + 2) Giải phương trình: 2 2 1 1 3 1 4 2 1 2 x x x x x x + + = + + + + + Ví dụ 11: Giải phương trình: 2 3 8 3 8 x x += +
  • 29. Liên hệ tài liệu word toán SĐT (zalo): 27 Website: Giải: Điều kiện 2 x ≥ − Đặt ( ) 2 2 ; 0 2 4 a x a b b b x x  = +  ≥ >  = − +   . Ta có 2 2 2 2 2 2 4 2 4 8 a b x x x x + = + + − + = + , suy ra ( )( ) 2 2 2 3 2 0 a b ab a b a b + = ⇔ − − = + TH1: 2 2 1 2 2 4 3 2 0 2 x a b x x x x x x =  = ⇔ + = − + ⇔ − + = ⇔  =  . + TH2: ( ) 2 2 2 4 2 2 4 6 4 0 3 13 a b x x x x x x = ⇔ + = − + ⇔ − − = ⇔ = ± . Nhận xét: Bài toán sử dụng phương pháp đặt ẩn phụ rồi đưa về phương trình đẳng cấp bậc hai, tìm mối quan hệ giữa hai ẩn phụ sau đó nâng lũy thừa tìm nghiệm của phương trình ban đầu. NHẮC LẠI KIẾN THỨC CŨ VÀ PHƯƠNG PHÁP: - Hằng đẳng thức: ( )( ) 2 2 2 2 u v u v u uv v + = + − + - Cách giải phương trình bậc hai dạng: 2 2 . . . 0 a x b xy c y + + = (*). Làm nháp: Chia cả hai vế của phương trình (*) cho 2 y ( vì là làm nháp nên ta cứ coi là 0 y ≠ ), khi đó (*) 2 . . 0 x x a b c y y   ⇔ + + =     . Coi đây là phương trình bậc hai ẩn x t y = (đã biết cách giải) thì nghiệm t tìm được sẽ biểu hiện mối quan hệ giữa , x y . Cụ thể là 1 2 . x t y x t y =   =  Ý tưởng: Bài toán này có thể giải bằng cách lũy thừa hai vế, đưa về phương trình bậc bốn và vấn đề ta gặp phải sẽ là nghiệm của phương trình bậc bốn. Ta đặt dấu hỏi cho nó? Nhưng nếu đế ý 3 8 2 = nên trong căn thức sẽ xuất hiện ngay hằng đẳng thức ( )( ) 3 3 3 2 8 2 2 2 4 x x x x x + = + = + − + . Vậy nên trong căn bậc hai đã xuất hiện hai tích. Do đó để xuất hiện phương trình đẳng cấp thì ta cần làm công việc sau, đó là đồng nhất hệ số của biểu thức dạng: ( ) ( ) 2 2 1 1 2 4 2 8 2 0 2 4 2 8 a a a x x b x x b a b a b =  =   − + + + = + ⇔ − = ⇔   =   + =  . Đến đây, ta viết lại phương trình ban đầu như sau: ( ) ( ) ( )( ) 2 3 2 2 8 3 8 2 4 2 2 3 2 4 2 x x x x x x x x + = + ⇔ − + + + = − + + (i)
  • 30. Liên hệ tài liệu word toán SĐT (zalo): 28 Website: * Đặt 2 2 4 0 2 0 u x x v x  = − + >   = + ≥   nên phương trình trên trở thành: (i) ( )( ) 2 2 3 2 0 2 0 2 u v u uv v u v u v u v =  ⇔ − + = ⇔ − − = ⇔  =  * Hoặc nhận thấy ( ) 2 2 2 4 1 3 0 x x x − + = + + ≠ nên phương trình (i) tương đương với: 2 2 2 2 2. 3 1 0 2 4 2 4 x x x x x x + + − + = − + − + . Có thể đặt căn thức là ẩn phụ để Giải phương trình: bậc hai. * Hai hướng trên đều cho ta: TH1. Với 2 2 2 1 2 4 2 2 3 2 0 x x x x x x x x ≥ − =   − + = + ⇔ ⇔   = − + =   . TH2. Với 2 2 2 2 4 2 2 3 13 6 4 0 x x x x x x x ≥ −  − + = + ⇔ ⇔ = ±  − − =  . Bài toán kết thúc. BÀI TẬP TƯƠNG TỰ 1) Giải phương trình: 2 3 2 3 5 2 6 4 x x x x x + − = + − + . Đáp số: 1 13 2 x − + = 2) Giải phương trình: 2 4 2 3 3 1 1 3 x x x x − + = − + + Đáp số: 1 x = II. BÀI TẬP Bài 1: Giải phương trình: 3 1 2 3 x x + + − =. Giải Điều kiện: 1 2 3 x − ≤ ≤ . Phương trình đã cho tương đương với ( )( ) 2 3 2 3 1 2 9 x x x + + + − = 2 2 2 3 0 3 5 2 3 3 5 2 6 9 x x x x x x x x − >  ⇔ − + + = − ⇔  − + + = − + 
  • 31. Liên hệ tài liệu word toán SĐT (zalo): 29 Website: 2 1 4 11 7 0 7 4 x x x x =   ⇔ − + = ⇔  =  Đối chiếu với điều kiện ta được được nghiệm: 7 1; 4 x x = = . Nhật xét: Bài toán sử dụng phương pháp nâng lũy thừa (bình phương) hai vế tìm nghiệm của phương trình. NHẮC LẠI KIẾN THỨC VÀ PHƯƠNG PHÁP: - Phương trình dạng ( ) ( ) ( ) ( ) ( ) 2 2 0 f x g x m f x g x m + = > + = ( ) ( ) ( ) ( ) ( ) ( ) ( ) ( ) 2 2 2 2 f x g x f x g x m f x g x m f x g x ⇔ + + = ⇔ = − − ( ) ( ) ( ) ( ) ( ) ( ) 2 1 2 2 2 4 m f x g x x x x x f x g x m f x g x  ≥ + =   ⇔ ⇒   =   = − −      - Phương trình trên có cách giải khác như sau: ( ) ( ) ( ) ( ) f x g x m f x m g x + = ⇔ = − ( ) ( ) ( ) ( ) ( ) ( ) ( ) ( ) 2 2 2 2 m f x m f x f x m m g x g x m g x g x m f x ≥ ≥     ⇔ ⇔   = − + = + −     ( ) ( ) ( ) ( ) ( ) ( ) ( ) 2 1 2 2 2 2 ; 4 m f x g x m f x x x x x m g x g x m f x  ≥ + ≥ =   ⇔ ⇒   = = + −    Ý tưởng: Đây là một bài phương trình cơ bản, dạng toán một vế chưa hai căn thức vế còn lại là một hằng số thì phương pháp nâng lũy thừa hai vế là một phương pháp tối ưu nhất. Bài toán kết thúc. BÀI TẬP TƯƠNG TỰ 1) Giải phương trình: 3 1 1 8 x x + + + = Đáp số: 8 x = 2) Giải phương trình: 7 4 1 3 x x + − + = Đáp số: 3 x = . Bài 2: Giải phương trình: ( )( ) 2 1 1 2 2 1 8 x x x + + − + − =
  • 32. Liên hệ tài liệu word toán SĐT (zalo): 30 Website: Giải Điều kiện: 1 1 x − ≤ ≤ Đặt: 1 1 t x x = + + − ( )( ) 2 2 1 2 1 1 1 2 2 1 t x x x x x ⇒ = + + + − + − = + − Khi đó phương trình đã cho trở thành: 2 . 8 t t = 3 2 2 8 2 2 2 1 4 1 1 0 t t x x x ⇔ = ⇔ = ⇔ + − = ⇔ − = ⇔ = Vậy phương trình đã cho có nghiệm duy nhất là 0 x = . Nhận xét: Bài toán sử dụng phương pháp đặt ẩn phụ giải phương trình NHẮC LẠI KIẾN THỨC VÀ PHƯƠNG PHÁP: - Hằng đẳng thức quen thuộc: ( ) 2 2 2 2 u v u uv v + = + + . - Với ( ) [ ; ] f x a b ∈ − , đặt ( ) ( ) t a f x b f x = + + − , khi đó ( ) ( ) ( ) ( ) ( ) ( ) ( ) ( ) 2 2 2 2 t a b t a b a f x b f x a f x b f x − − = + + + − ⇔ + − = Ý tưởng: Nhận thấy ở hai căn thức, ta có tổng bình phương của chúng là một hằng số, mặt khác tích của chúng có liên quan đến biểu thức còn lại trong phương trình. - Ta có: ( ) ( ) ( )( ) 2 2 2 1 1 1 1 2; 2 1 2 1 1 x x x x x x x + + − = + + − = − = + − - Do đó: ( ) ( ) ( )( ) 2 2 2 2 2 1 1 1 2 1 1 x x x x x + − = + + − + + − ( ) 2 1 1 . x x = + + − - Đặt 1 1 t x x = + + − , phương trình đã cho trở thành: 3 8 t = 2 2 2 2 2 1 4 1 1 0 t x x x ⇔ = ⇔ + − = ⇔ − = ⇔ = Bài toán kết thúc. BÀI TẬP TƯƠNG TỰ 1) Giải phương trình: ( ) 2 6 2 4 3 2 2 x x x + − = + + − Đáp số: 2 x = ± 2) Giải phương trình: ( )( ) 7 2 2 4 3 2 2 4 4 3 x x x x + − + = − + + Đáp số: . 5 4 3 4 x ± = −
  • 33. Liên hệ tài liệu word toán SĐT (zalo): 31 Website: Bài 3: Giải phương trình: ( ) 2 2 2 2 4 2 3 x x x x + + − + − = − Giải: Điều kiện 2 x ≥ . Đặt: 2 2 0 t x x = + + − > 2 2 2 2 2 2 4 2 2 4 t x x x x x ⇒ = + + − + − =+ − Phương trình đã cho tương đương: 2 2 2 2 2 4 6 x x x x + + − + + − = 2 6 2 t t t ⇔ + − ⇔ = hoặc 3 t = − (loại). Với 2 t = ta có 2 2 2 x x + + − = Do điều kiện 2 x ≥ , ta có: 2 2 4 0 2 x x + + − ≥ + = Suy ra phương trình có nghiệm duy nhất 2 x = . Nhận xét: Bài toán sử dụng phương pháp đặt ẩn phụ, đưa phương trình ban đầu về phương trình bậc hai tìm ẩn, sau đó dùng phương pháp nâng lũy thừa tìm nghiệm của phương trình ban đầu. * NHẮC LẠI KIẾN THỨC VÀ PHƯƠNG PHÁP - Cách giải phương trình bậc hai tổng quát: 2 . . 0 a t b t c + + = - Hằng đẳng thức: ( )( ) 2 2 a b a b a b − = − + và ( ) 2 2 2 2 a b a ab b + = + + - Phương trình có dạng: ( ) ( ) f x g x m + = , với m là số thực dương thì có hai cách nâng lũy thừa như sau: Cách 1. Bình phương hai vế của phương trình, ta có: ( ) ( ) ( ) ( ) ( ) ( ) ( ) ( ) 2 0; 0 2 . f x g x f x g x m f x g x f x g x m ≥ ≥   + = ⇔  + + =   ( ) ( ) ( ) ( ) ( ) ( ) 1 2 2 2 0; 0 . 4 . f x g x x x x x f x g x m f x g x ≥ ≥  =   ⇔ ⇒   =   = − −      Cách 2. Chuyển ( ) g x sang VP rồi bình phương, ta có: ( ) ( ) ( ) ( ) ( ) ( ) 2 2 m g x f x m g x f x m m g x g x  ≥  = − ⇔  = − +  
  • 34. Liên hệ tài liệu word toán SĐT (zalo): 32 Website: ( ) ( ) ( ) ( ) ( ) ( ) ( ) ( ) 2 2 2 2 . 2 4 m g x m g x m g x m f x g x m g x m f x g x   ≥ ≥   ⇔ ⇔     = − + = − +       Chú ý: - Nếu ( ) ( ) f x g x k − = ; k là hằng số thì ta có thể sử dụng cách liên hợp như sau: ( ) ( ) f x g x m + = (i) ( ) ( ) ( ) ( ) ( ) ( ) ( ) ( ) ( ) f x g x f x g x m f x g x ⇔ + − = − ( ) ( ) ( ) ( ) ( ) ( ) ( ) k f x g x m f x g x f x g x m ⇔ − = − ⇔ − = (ii) Lấy (i) + (ii), ta được ( ) ( ) 2 2 4 k k f x m f x m m m   = + ⇔ = +     . - Nếu c a b ≥ ≥ và x c ≥ suy ra x a x b c a c b + + + ≥ + + + . Dấu “=” xảy ra khi và chỉ khi x c = . Ý tưởng: Bài toán xuất hiện ba căn thức bậc hai ở VT là: 2, 2 x x + − và 2 4 x − , áp dụng hằng đẳng thức ( )( ) 2 2 a b a b a b − = − + dễ thấy được rằng 2 4 2. 2 x x x − = − + , hay nói cách khác căn thức cuối cùng chính là tích của hai căn thức còn lại. Đồng thời nếu chuyển ( ) 2 3 x − từ VP sang VT thì sẽ xuất hiện 2x , mà ( ) ( ) 2 2 2 2 2 2 2 x x x x x = + + − = + + − do đó VT của phương trình ban đầu có: ( ) ( ) 2 2 2 2 2. 2. 2 2 2 6 0 x x x x x x + + − + − + + + + − − = ( ) 2 2 2 2 2 6 0 x x x x ⇔ + + − + + + − − = - Đặt 2 2 0 t x x = + + − > thì phương trình (*) được viết lại thành: ( )( ) 2 0 0 2 2 3 0 6 0 t t t t t t t > >    ⇔ ⇔ =   − − = + − =    - Với 2 t = suy ra 2 2 2 2 2 2 2 4 4 x x x x x ≥   + + − = ⇔  + − =   2 2 2 4 2 x x x x ≥   ⇔ ⇔ =  − = −   Đến đây có thể đánh giá như lời giải là: 2 2 2 4 0 2 2 x x x x ≥ ⇒ + + − ≥ + = ⇒ = . - Vì ( ) 2 2 4 x x + − − = nên giải phương trình 2 2 2 x x + + − = theo chú ý như sau: ( ) 2 2 2 4 2 2 2 x x x x + + − = ⇔ = + + −
  • 35. Liên hệ tài liệu word toán SĐT (zalo): 33 Website: 2 2 2 2 2 4 2 x x x x ⇔ + − = ⇒ + = ⇔ = Bài toán kết thức. BÀI TẬP TƯƠNG TỰ 1) Giải phương trình: 2 2 1 2 1 2 1 3 2 x x x x + + − + − = − Đáp số: phương trình vô nghiệm thực. 2) Giải phương trình: 2 2 2 5 2 2 5 1 5 2 x x x x x + + + − = − + + Đáp số: 17 16 x = Bài 4: Giải phương trình: 2 3 1 3 1 1 x x x x + + − = + + − Giải: Điều kiện 1 1 x − ≤ ≤ . Phương trình tương đương với: ( ) 2 2 2 1 1 1 1 2 1 x x x x x + + + − = + + − + + ( ) ( ) 1 1 2 1 2 1 1 1 x x x x x x ⇔ + + − + + = + + + + − ( )( ) 1 1 2 1 1 0 x x x ⇔ + + − − + − = + Giải 2 2 1 1 2 2 2 1 4 1 1 0 x x x x x + + − = ⇔ + − = ⇔ − = ⇔ = + Giải 1 1 0 x x + = ⇔ = Đáp số 0 x = Nhận xét: Bài toán sử dụng phương pháp nhóm nhân tử chung, sau đó nâng lũy thừa bậc hai để tìm nghiệm của phương trình. * NHẮC LẠI KIẾN THỨC VÀ PHƯƠNG PHÁP - Giải phương trình: ( ) ( ) ( ) ( ) 0 . 0 0 f x f x g x g x =  = ⇔  =   . - Giải phương trình: ( ) ( ) a f x a f x b − + + = ( ) ( ) ( ) ( ) ( ) 2 2 2 2 2 2 2 . 2 2 4 2 a f x a a f x a b a a f x a f x b a ≥ ≥ −  ≥ ≥ −    ⇔ = ⇔     + − − =−      
  • 36. Liên hệ tài liệu word toán SĐT (zalo): 34 Website: Ý tưởng: Bài toán xuất hiện ba căn thức, nhưng có điều đặc biệt ở đây là căn thức còn lại là tích của hai căn thức kia. Mặt khác 2 1 , 1 x x − − có sự đồng nhất hệ số, do đó ta sẽ nhóm hai căn này lại nên ta được nhân tử chung như sau: ( ) 2 1 1 1 1 1 x x x x − − − = − + − . Và ta mong muốn biểu thức 3 3 1 x x + − + sẽ phân tích được biểu thức có chứa 1 1 x + − . Thật vậy nếu coi ( ) 3 3 1 h x x x = + − + là một phương trình bậc hai ẩn 1 x + ta sẽ thấy: ( ) ( )( ) 1 3 1 2 1 1 1 2 h x x x x x = + − + + = + − + − . Chính vì thế bài toán của ta được giải quyết như sau: 2 3 1 3 1 1 x x x x + + − = + + − ( )( ) ( ) 1 1 1 2 1 1 1 0 x x x x ⇔ + − + − + − + − = ( )( ) 1 1 1 1 1 1 2 0 1 1 2 x x x x x x  + = ⇔ + − + + − − = ⇔  + + − =   . Phần còn lại chỉ là việc bình phương các phương trình và tìm nghiệm như ở trên đã nêu. Ta được nghiệm của phương trình là 0 x = Bài toán kết thúc. BÀI TẬP TƯƠNG TỰ 1) Giải phương trình: 2 4 4 3 2 2 x x x x + + − = + + − Đáp số: 1 x = 2) Giải phương trình: 2 2 3 1 4 3 2 1 1 2 x x x x + + − = + + − Đáp số: 0 x = Bài 5: Giải phương trình: 3 3 1 1 x x x + + + = − Giải: Phương trình tương đương với: 2 3 2 3 1 2 2 x x x + + + = − . Đặt 2 2 3 2 2 3 1 u x v u x v x  = +  ⇒ − = −  = +   với ; 0 u v > , ta được ( ) ( )( ) 2 2 2 2 0 u v v u v y v u + = − ⇔ + − − = 2 3 1 3 2 3 1 7 4 3 v u x x x x x ⇔ = + ⇔ + = + + ⇔ + = + + + ( ) 2 3 2 6 4 3 2 3 3 4 3 6 9 x x x x x x x x ≥   ⇔ − = + ⇔ + = − ⇔  + = − +  
  • 37. Liên hệ tài liệu word toán SĐT (zalo): 35 Website: 2 3 5 28 5 28 10 3 0 3 x x x x x x ≥   = ±  ⇔ ⇔ ⇔ = +   − − = ≥    Nhận xét: Bài toán sử dụng phương pháp đặt ẩn phụ để đưa phương trình từ phức tạp về đơn giản hóa, sau đó dùng phương pháp nâng lũy thừa để tìm nghiệm của phương trình. * NHẮC LẠI KIẾN THỨC CŨ VÀ PHƯƠNG PHÁP: - Hằng đẳng thức cơ bản: ( )( ) 2 2 a b a b a b − = − + . - Cách giải phương trình vô tỷ dạng: ( ) ( ) f x g x m = + ( ) ( ) ( ) ( ) ( ) ( ) ( ) ( ) ( ) ( ) 2 2 ; 0 ; 0 2 f x g x f x g x f x g x m m g x f x g x m  ≥ ≥    ⇔ ⇔   − − = = +     ( ) ( ) ( ) ( ) ( ) 1 2 2 2 2 ; 0 . 4 f x g x x x x x f x g x m m g x ≥  =   ⇔ ⇔   =   − − =     Ý tưởng: Một bài toán đẹp, quan sát VT của phương trình có xuất hiện hai căn bậc hai riêng biệt đồng thời trong căn chứa các biểu thức bậc nhất, cũng như VP của phương trình cũng là một biểu thức bậc nhất, nên vậy ta có thể nâng lũy thừa để đưa phương trình ban đầu về phương trình bậc bốn. Nhưng nếu tinh ý một chút, ta có ( ) ( ) ( ) 3 1 3 2 2 2 1 x x x x + − + = − = − , Do vậy đặt 3 3 1 u x v x  = +   = +   với ; 0 u v ≥ , suy ra: ( ) 2 2 2 2 2 1 v u x x − = − = − . Khi đó, phương trình đã cho tương đương với: ( ) ( )( ) 2 2 2 2 0 v u v u v u v u − = + ⇔ + − − = (i) - Vì ; 0 u v ≥ nên phương trình (i) 0 2 u v v u = =  ⇔  = +  , với 0 u v = = phương trình vô nghiệm nên ta chỉ cần giải phương trình: 2 v u = + . - Với 2 v u = + , ta có: 3 1 3 2 3 1 7 4 3 x x x x x + = + + ⇔ + = + + + ( ) 2 3 2 6 4 3 2 3 3 4 3 6 9 x x x x x x x x ≥   ⇔ − = + ⇔ + = − ⇔  + = − +   2 3 5 28 5 28 10 3 0 3 x x x x x x ≥   = ±  ⇔ ⇔ ⇔ = ±   − − = ≥    Bài toán kết thúc. BẢI TẬP TƯƠNG TỰ
  • 38. Liên hệ tài liệu word toán SĐT (zalo): 36 Website: 1) Giải phương trình: 4 1 2 2 2 1 x x x + + + = − Đáp số: 2 5 x= + 2) Giải phương trình: 3 5 2 1 4 x x x + + + = − Đáp số: 5 87 8 x − = Bài 6: Giải phương trình: ( ) 2 3 7 2 1 x x x x + + = + Giải: Điều kiện 0 x > Phương trình tương đương: ( ) 2 3 2 1 7 x x x x + + = + Chia hai vế cho 0 x ≠ , ta được: 1 3 7 3 1 3 4 3 3 2 2 1 2 1 0 2 0 x x x x x x x x x x x x x x x x          + + = + ⇔ + − + + + = ⇔ + − + − =                      + Giải: 2 1 3 3 2 4 4 3 0 3 x x x x x x x x =  + = ⇔ + = ⇔ − + = ⇔  =  + Giải: 2 2 3 2 3 4 3 4 0 x x x x x x x x + = ⇔ + = ⇔ + − = ( )( ) 2 1 4 0 1 x x x x ⇔ − + + = ⇔ = Đáp số 1; 3 x x = = . Nhận xét: Bài toán sử dụng phương pháp ẩn phụ không hoàn toàn, sau đó nâng lũy thừa tìm nghiệm của phương trình ban đầu. * NHẮC LẠI KIẾN THỨC VÀ PHƯƠNG PHÁP - Phương pháp đặt ẩn phụ không hoàn toàn: xét một phương trình bậc hai có dạng ( ) ( ) 2 . . 0 mf x t ng x t k + + =(*), trong đó t là ẩn phụ được biểu diễn dưới dạng ( ) t h x = . Khi đó, ta có: ( ) ( ) 2 4 t ng x kmf x = −     Δ , với t Δ bắt buộc là một số chính phương. Do đó, tìm được nghiệm của (*), đó là
  • 39. Liên hệ tài liệu word toán SĐT (zalo): 37 Website: ( ) ( ) ( ) ( ) ( ) ( ) ; t t ng x ng x t h x t h x mf x mf x − + − − = = = = Δ Δ - Cách giải phương trình: ( ) ( ) ( ) ( ) ( ) ( ) 2 ; 0 . f x g x f x g x f x g x ≥   = ⇔  =   . Ý tưởng: Trước hết, ta cần quy đồng mẫu số bài toán, như vậy ta sẽ được phương trình có dạng ( ) ( ) ( ) . f x g x h x = và nếu nâng lũy thừa hai vế, ta sẽ thu được một phương trình bậc 5. Và phương trình bậc 5 nếu không có nghiệm nguyên thì sẽ rất khó để giải quyết. Vậy nên ta cần nghĩ đến hướng tư duy khác, đó là bài toán xuất hiện căn thức 3 x x + nên ta mong muốn sẽ tạo được lượng 2 3 k x x   +       để có thể đưa về phương trình bậc hai, sau đó đặt 3 t x x = + để sử dụng phương pháp ẩn phụ không an toàn. - Ta có: ( ) ( ) 2 2 3 7 3 2 1 7 2 1 x x x x x x x x + + = ⇔ + + = + + Chia cả hai vế của phương trình cho x, ta có: 1 3 7 3 1 3 4 2 1 2 1 0 x x x x x x x x x x x     + + = + ⇔ + − + + + =         (*) - Đặt 3 0 t x x = + > , khi đó ta có ( ) 2 1 4 * 2 1 0 t t x x   ⇔ − + + =     . Có: 2 2 1 4 1 1 1 t x x x     = + − = −         Δ nên suy ra được ( ) ( ) 3 1 1 2 2 1 1 2 1 1 3 2 1 1 x i t t x x x t t x x ii x x x x   + = = = + + −      ⇔ ⇔    =  =+ + − +  + =     - Giải (i), ta có (i) 3 1 3 4 4 3 0 3 x x x x x x =  ⇔ + = ⇔ − + = ⇔  =  - Giải (ii), ta có (ii) 3 2 3 4 3 4 0 1 x x x x x x ⇔ + = ⇔ + − = ⇔ = Bài toán kết thức. BÀI TẬP TƯƠNG TỰ: 1) Giải phương trình: ( ) ( )( ) 3 4 12 28 x x x x + − + = −
  • 40. Liên hệ tài liệu word toán SĐT (zalo): 38 Website: Đáp số: ( ) 4 2 1 ; 31 3 x x = − = − 2) Giải phương trình: 3 2 2 2 x x x x − = − − Đáp số: 1 5 1 65 ; . 2 8 x x + − = = Ví dụ 6: Giải phương trình: ( )( ) 9 2012 6 2012 9 6 . x x x x + + += + + + Giải: Điều kiện 6 x ≥ − Phương trình đã cho tương đương với: ( )( ) 9 2012 6 1 0 x x + − + − = + Giải ( ) 2 9 2012 0 2012 9 4048135 x x + − = ⇔ = − = + Giải 6 1 0 5 x x + − = ⇔ = − Vậy phương trình có hai nghiệm: 4048135; 5 x x = = − Nhận xét: Bài toán sử dụng phương pháp nhóm nhân tử chung và nâng lũy thừa tìm nghiệm của phương trình. * NHẮC LẠI KIẾN THỨC VÀ PHƯƠNG PHÁP: - Các giải phương trình dạng: ( ) ( ) ( ) ( ) ( ) ( ) ( ) ( ) 2 2 . 0 . f x m f x m f x m g x n g x n g x n  =  =  − − = ⇔ ⇔   =  =   Ý tưởng: Bài toán cho hết sức đơn giản, với sự xuất hiện của hai căn thức ( )( ) 9; 6; 9 6 x x x x + + + + nên không khó để nhóm được nhân tử chung như sau: ( )( ) 9 2012 6 2012 9 6 x x x x + + += + + + ( )( ) 9 9 6 2012 6 2012 0 x x x x ⇔ + − + + + + − = ( ) ( ) 9 1 6 2012 1 6 0 x x x ⇔ + − + − − + = ( )( ) 1 6 9 2012 0 x x ⇔ − + + − = Bài toán kết thúc. BÀI TẬP TƯƠNG TỰ
  • 41. Liên hệ tài liệu word toán SĐT (zalo): 39 Website: 1) Giải phương trình: 2 2 6 7 2 13 42 x x x x + + + = + + + Đáp số: 3; 5 x x = − = − 2) Giải phương trình: 2 4 2 3 2 7 12 x x x x + + + = + + + Đáp số: 0; 2 x x = = − Bài 8: Giải phương trình: 8 2 2 1 x x + − − = Giải: Điều kiện 8 2 x − ≤ ≤ + Nếu 1 2 8 9 3 x x < ≤ ⇒ + > = 2 2 1 1 8 2 2 3 2 1 x x x − < − = ⇒ + − − > − = ⇒ loại. + Nếu 8 1 x − ≤ < , tương tự ta có: 8 2 2 9 2 1 1 x x + − − < − ==> loại Với 1 x = , thỏa mãn phương trình. Đáp số 1 x = . Nhận xét: Bài toán sử dụng phương pháp nhẩm nghiệm và đánh giá theo miền nghiệm để chứng minh nó có nghiệm duy nhất. Ý tưởng: Đây một bài toán đơn giản nhưng đòi hỏi đi theo phương pháp đánh giá thì cần đoán trước nghiệm của phương trình. Đầu tiên ta sẽ ưu tiên nghiệm nguyên trước, với nghiệm nguyên thì các biểu thức chứa căn phải là một số chính phương, tức là 2 2 8 2 x k x h  + = ∈   − = ∈     . Với điều kiện chặn của [ ] 8; 2 x ∈ − ] thì ta sẽ thử một vài giá trị nguyên của x và thấy rằng tại 1 x = thỏa mãn phương trình. Công việc còn lại là ta sẽ đi chứng minh nó là nghiệm duy nhất. Nó là nghiệm duy nhất nếu với [ 8; 1) (1; 2] x ∈ − ∪ thì phương trình bài cho vô nghiệm. Đi xét từng trường hợp, ta có: - Với [ 8; 1) x ∈ − suy ra 8 2 2 9 2 1 1 x x + − − < − =. - Với (1; 2] x ∈ suy ra 8 2 2 9 2 1 1 x x + − − > − =. Cả hai trường hợp trên đều chứng minh 1 x = là nghiệm duy nhất của phương trình. Bài toán kết thúc.
  • 42. Liên hệ tài liệu word toán SĐT (zalo): 40 Website: BÀI TẬP TƯƠNG TỰ 1) Giải phương trình: 7 8 3 4 x x x + − = + Đáp số: x = 1 2) Giải phương trình: 9 2 1 1 x x + − − = Đáp số: x = 0 Bài 9: Giải phương trình: ( )( ) 3 1 1 1 x x x + − − + = Giải: Điều kiện 0 1 x ≤ ≤ Phương trình tương đương với: ( ) ( ) 3 1 1 1 3 1 1 3 3 x x x x x x − + = ⇔ − + = + + + + Nếu ( ) 0 1 3 1 1 3 x x ≤ ≤ ⇒ − + > , đồng thời 3 1 4 3 x x + + < + =, suy ra VT > VP (loại). Thử lại ta thấy 1 x = là nghiệm. Nhận xét: Bài toán kết hợp giữa phương pháp nhân liên hợp và phương pháp đáng giá để tìm nghiệm của phương trình. * NHẮC LẠI KIẾN THỨC VÀ PHƯƠNG PHÁP - Biểu thức liên hợp: ( )( ) x m x x m x x m x + − = + − + + m x m x x m x + − = + + với 0; 0 x x m ≥ + ≥ . - Đánh giá: ( ) ( ) ( ) m f x n g x h x − + = + với ( ) 0 2 m f x n m ≥ ≥    =   . Ta có: ( ) ( ) ( ) ( ) 0 2 m f x n n f x m g x h x m  − + >  ≤ < ⇒  + <   , suy ra phương trình vô nghiệm. Vậy x m = là nghiệm của phương trình đã cho Ý tưởng: Bài toán xuất hiện ba căn thức nằm trong một tích, sẽ rất khó để định hình ra hướng giải, ẩn phụ sẽ rất phức tạp. Nhưng nếu xét hai căn thức đầu tiên ta thấy ( ) ( ) 2 2 3 3 x x − − = . Vì thế ta sẽ nghĩ ngay đến chuyện dùng hằng đẳng thức dạng: ( )( ) 2 2 a b a b a b − = − + . Khi đó phương trình đã cho tương đương với:
  • 43. Liên hệ tài liệu word toán SĐT (zalo): 41 Website: ( ) 3 1 1 1 3 1 3 3 3 x x x x x x − + = ⇔ − + = + + + + (i) Với phương trình (i), ta sẽ đi nhẩm một vài giá trị nghiệm đẹp thỏa mãn các yêu cầu là 1 0 x ≥ ≥ và các biểu thức trong căn thức là số chính phương vì thế ta khẳng định nó có nghiệm duy nhất 1 x = , đồng thời 1 lại là miền chặn của biến do đó ta sẽ đi đánh giá phương trình (i). Tức là với 0 1 x ≤ < , ta sẽ chứng minh (I) vô nghiệm như sau: 3 1 3 3 0 1 3 3 x x x x  − + >  ≤ < ⇒  + + <   => VT > VP => (i) vô nghiệm Vậy ta kết luận 1 x = là nghiệm của phương trình đã cho. Bài toán kết thúc. BÀI TẬP TƯƠNG TỰ 1) Giải phương trình: ( )( ) 2 1 2 1 1 x x x + − − − + = Đáp số: 2 x = 2) Giải phương trình: ( )( ) 1 2 3 1 1 x x x + − − − + = Ví dụ 9: Giải phương trình: ( )( ) 4 2 4 2 2 x x x + − − + = Giải: Điều kiện 4 4 x − ≤ ≤ Phương trình đã cho tương đương với: ( ) 4 2 2 4 2 x x x x − + = + + . + Với 0 x = là nghiệm. + Giải: ( ) 4 2 2 4 2 x x − += + + Đặt 4; 4 u x v x = + = − ta thu được ( ) 2 2 2 2 2 2 2 2 2 8 5 8 4 0 8 v u u u u u u v = +  ⇒ + + = ⇔ + − =  + =  ( ) 2 14 ; 2 96 5 5 4 5 25 2 u v x x u l  = =  ⇔ ⇒ + = ⇔ = −  = −   (thỏa mãn) Vậy phương trình có hai nghiệm: 96 0; 25 x x = = − . Nhận xét: Sử dụng phương pháp nhân liên hợp, sau đó đặt ẩn phụ tìm nghiệm của bài toán.
  • 44. Liên hệ tài liệu word toán SĐT (zalo): 42 Website: NHẮC LẠI KIẾN THỨC VÀ PHƯƠNG PHÁP - Hằng đẳng thức ( )( ) a b a b a b a b a b a b − − = − + ⇔ + = − . - Giải phương trình tổng quát dạng ( ) ( ) f x g x m = + ( ) ( ) ( ) ( ) ( ) ( ) ( ) ( ) ( ) ( ) 2 2 2 2 ; 0 2 4 f x g x m f x g x f x g x m g x m f x g x m mg x  ≥ + ≥    ⇔ ⇔   = + +   − − =       Ý tưởng: Không khó để nhận thấy, phương trình có một nghiệm là 0 x = . Đồng thời vế trái của phương trình có xuất hiện biểu thức 4 2 x + − , dễ thấy rằng ( ) ( )( ) 2 2 4 2 4 2 4 2 x x x x = + − = + − + + . Vì thế, phương trình đã cho tương đương với: ( )( ) ( )( ) 4 2 0 0 4 2 4 2 2 4 2 4 2 4 2 4 2 x x x x x x x x  + − = ⇔ = + − − + = + − + + ⇔  − = + +   Phương trình còn lại có thể giải bằng cách tổng quát nêu ở trên, hoặc có thể giải quyết bằng cách đặt ẩn phụ như sau: ( ) 4 ; 0 4 v x u v u x  = −  ≥  = +   . Ta có hệ phương trình 2 2 8 2 14 2 96 ; 4 5 5 5 25 2 2 v u u v x x v u  + = ⇔ = = ⇒ + = ⇔ = −  = +  . Bài toán kết thúc. BÀI TẬP TƯƠNG TỰ 1) Giải phương trình: ( )( ) 1 1 1 1 2 x x x + − − + = . Đáp số: 24 0; 25 x x = = − 2) Giải phương trình: ( )( ) 9 1 9 1 3 x x x + − − + = Đáp số: 0 x = Ví dụ 10: Giải phương trình: 1 1 4 1 5 2 1 2 x x x x + + = + + + Giải:
  • 45. Liên hệ tài liệu word toán SĐT (zalo): 43 Website: Phương trình tương đương với: 1 1 5 4 1 2 1 2 x x x x − = − − + + . Giả sử: 1 1 1 2 1 2 2 1 2 x x x x x > ⇒ + > + ⇒ < + + => 0 5 4 1 1 VT x x x < ⇒ < − ⇒ < (mâu thuẫn). Giả sử 0 1 x ≤ < , lập luận tương tự thu được 1 x > (mâu thuẫn). Thử lại thu được nghiệm duy nhất 1 x = Nhận xét: Bài toán sử dụng phương pháp đánh giá miền nghiệm để tìm nghiệm của phương trình. Ý tưởng: Quan sát bài toán, ta thấy có vẻ nó hơi rắc rối một chút vì xuất hiện tới bốn căn thức mà căn lại còn ở dưới mẫu thì sẽ rất khó khăn trong việc giải bằng các công cụ mà ta thường làm như ẩn phụ, nâng lũy thừa...Và hướng cuối cùng ta nghĩ đến là đánh giá. Để đánh giá được nó, ta cần tìm nghiệm trước, vẫn là ưu tiên hướng nghiệm nguyên đồng thời cân bằng căn thức với căn thức, phân thức với phân thức ta có: 1 1 1; 4 1 5 1 2 1 2 x x x x x x = ⇔ = + = ⇔ = + + Và bây giờ, ta sẽ đi chứng minh 1 x = là nghiệm duy nhất của phương trình. Hay nói cách khác, với 1 x > hoặc 1 x < nó sẽ vô nghiệm. Biến đổi phương trình đã cho về dạng: ( )( ) 1 1 2 2 1 4 1 5 5 4 1 2 1 2 2 1 2 x x x x x x x x x x + − + + += + ⇔ = − + + + + + * Với 1 x > , ta có 2 2 1 2 2 1 0 5 4 1 5 4 1 0 x x x x x x x x  + < + + − − <   ⇔   > + − + >    , suy ra phương trình vô nghiệm. * Với 1 0 x > ≥ , ta có: 2 2 1 2 2 1 0 5 4 1 5 4 1 0 x x x x x x x x  + > + + − + >   ⇔   < + − + <    , suy ra phương trình vô nghiệm. Vậy 1 x = là nghiệm duy nhất của phương trình. Bài toán kết thúc. BÀI TẬP TƯƠNG TỰ 1) Giải phương trình: 1 1 3 1 4 2 1 2 1 x x x x + + = + + + 2) Giải phương trình: 2 2 1 1 3 1 4 2 1 2 x x x x x x + + = + + + + + Bài 12: Giải phương trình: 2 3 8 3 8 x x += +
  • 46. Liên hệ tài liệu word toán SĐT (zalo): 44 Website: Giải: Điều kiện 2 x ≥ − Đặt ( ) 2 2 ; 0 2 4 a x a b b b x x  = +  ≥ >  = − +   . Ta có 2 2 2 2 2 2 4 2 4 8 a b x x x x + = + + − + = + , suy ra ( )( ) 2 2 2 3 2 0 a b ab a b a b + = ⇔ − − = + TH1: 2 2 1 2 2 4 3 2 0 2 x a b x x x x x x =  = ⇔ + = − + ⇔ − + = ⇔  =  . + TH2: ( ) 2 2 2 4 2 2 4 6 4 0 3 13 a b x x x x x x = ⇔ + = − + ⇔ − − = ⇔ = ± . Nhận xét: Bài toán sử dụng phương pháp đặt ẩn phụ rồi đưa về phương trình đẳng cấp bậc hai, tìm mối quan hệ giữa hai ẩn phụ sau đó nâng lũy thừa tìm nghiệm của phương trình ban đầu. NHẮC LẠI KIẾN THỨC CŨ VÀ PHƯƠNG PHÁP: - Hằng đẳng thức: ( )( ) 2 2 2 2 u v u v u uv v + = + − + - Cách giải phương trình bậc hai dạng: 2 2 . . . 0 a x b xy c y + + = (*). Làm nháp: Chia cả hai vế của phương trình (*) cho 2 y ( vì là làm nháp nên ta cứ coi là 0 y ≠ ), khi đó (*) 2 . . 0 x x a b c y y   ⇔ + + =     . Coi đây là phương trình bậc hai ẩn x t y = (đã biết cách giải) thì nghiệm t tìm được sẽ biểu hiện mối quan hệ giữa , x y . Cụ thể là 1 2 . x t y x t y =   =  Ý tưởng: Bài toán này có thể giải bằng cách lũy thừa hai vế, đưa về phương trình bậc bốn và vấn đề ta gặp phải sẽ là nghiệm của phương trình bậc bốn. Ta đặt dấu hỏi cho nó? Nhưng nếu đế ý 3 8 2 = nên trong căn thức sẽ xuất hiện ngay hằng đẳng thức ( )( ) 3 3 3 2 8 2 2 2 4 x x x x x + = + = + − + . Vậy nên trong căn bậc hai đã xuất hiện hai tích. Do đó để xuất hiện phương trình đẳng cấp thì ta cần làm công việc sau, đó là đồng nhất hệ số của biểu thức dạng: ( ) ( ) 2 2 1 1 2 4 2 8 2 0 2 4 2 8 a a a x x b x x b a b a b =  =   − + + + = + ⇔ − = ⇔   =   + =  . Đến đây, ta viết lại phương trình ban đầu như sau: ( ) ( ) ( )( ) 2 3 2 2 8 3 8 2 4 2 2 3 2 4 2 x x x x x x x x + = + ⇔ − + + + = − + + (i)
  • 47. Liên hệ tài liệu word toán SĐT (zalo): 45 Website: * Đặt 2 2 4 0 2 0 u x x v x  = − + >   = + ≥   nên phương trình trên trở thành: (i) ( )( ) 2 2 3 2 0 2 0 2 u v u uv v u v u v u v =  ⇔ − + = ⇔ − − = ⇔  =  * Hoặc nhận thấy ( ) 2 2 2 4 1 3 0 x x x − + = + + ≠ nên phương trình (i) tương đương với: 2 2 2 2 2. 3 1 0 2 4 2 4 x x x x x x + + − + = − + − + . Có thể đặt căn thức là ẩn phụ để Giải phương trình: bậc hai. * Hai hướng trên đều cho ta: TH1. Với 2 2 2 1 2 4 2 2 3 2 0 x x x x x x x x ≥ − =   − + = + ⇔ ⇔   = − + =   . TH2. Với 2 2 2 2 4 2 2 3 13 6 4 0 x x x x x x x ≥ −  − + = + ⇔ ⇔ = ±  − − =  . Bài toán kết thúc. BÀI TẬP TƯƠNG TỰ 1) Giải phương trình: 2 3 2 3 5 2 6 4 x x x x x + − = + − + . Đáp số: 1 13 2 x − + = 2) Giải phương trình: 2 4 2 3 3 1 1 3 x x x x − + = − + + Đáp số: 1 x = .
  • 48. Liên hệ tài liệu word toán SĐT (zalo): 1 Website: Chuyên đề 2. HỆ PHƯƠNG TRÌNH I. TÓM TẮT LÝ THUYẾT A. HỆ PHƯƠNG TRÌNH ĐỐI XỨNG LOẠI I Hệ phương trình loại I theo ẩn x và y: Là hệ phương trình mà khi ta đổi vai trò của các ẩn x và y thì hệ phương trình vẫn không thay đổi. Dạng hệ phương trình ( ) ( ) , 0 , 0 f x y g x y  =   =   với ( ) ( ) ( ) ( ) , y, , y, f x y f x g x y g x  =   =   . Đặt S x y P xy  = +  =  . Hệ phương trình ở dạng thu gọn ( ) ( ) , 0 , 0 f S P g S P  =   =   , điều kiện 2 4 S P ≥ . (với S là tổng hai nghiệm và P là tích hai nghiệm).  Phương pháp giải: Bước 1: Đặt S x y P xy  = +  =  và x, y chính là nghiệm của phương trình. Điều kiện 2 4 S P ≥ . Bước 2: Xác định S và P. Khi đó S và P là nghiệm của phương trình bậc hai: 2 X 0 X S P − + =. Bước 3: Giải phương trình: bậc hai theo ẩn X. Bước 4: Suy ra giá trị x, y. Ví dụ 1: Giải hệ phương trình: 2 2 3 2 2 3 5 6 x y x y x x y xy y  − + − =   − − + =   . Giải: Ta có hệ phương trình tương đương ( ) ( ) ( )( ) 2 2 2 2 5 6 x y x y x y x y  − + − =   − − =   . Đặt 2 2 a x y b x y = −   = −   . Hệ phương trình trở thành: 5 6 a b ab  + =  =  .
  • 49. Liên hệ tài liệu word toán SĐT (zalo): 2 Website: Suy ra a và b là hai nghiệm của phương trình: 2 2 5X 6 0 3 X X X  = − + = ⇔  =  . Với 2 2 11 2 2 2 6 3 3 7 3 3 6 x a x y x y b x y x y y    =   = + = − =     ⇒ ⇔ ⇔     = − =      − = = −     . Với 2 2 7 3 3 3 4 2 2 1 2 2 4 x a x y x y b x y x y y     =     = − = + =     ⇒ ⇔ ⇔     = − =     − = = −         . Ví dụ 2: Giải hệ phương trình : 2 2 4 2 x xy y x xy y  + + =   + + =   . Giải: Đặt ( ) 2 4 S x y S P P xy  = + ≥  =  . Hệ phương trình trở thành: 2 4 2 S P S P  − =   + =   . ( ) 2 2 2 2 2 2 2 2 4 6 0 0 3 P S P S P S S S S S S S P S    = − = −   = − =   ⇔ ⇔ ⇔ ⇔  =     − − = + − = =      = −     hoặc 3 5 S P  = −  =  . Với 2 2 2 0 0 0 S x y x P xy y    = + = =  ⇒ ⇔    = = =     hoặc 0 2 x y  =  =  . Với ( ) 2 3 3 3 3 3 5 5 5 3 5 0 x y S x y x y y y P xy y y    =− −  =− + =− =− −    ⇒ ⇔ ⇔     − − = = = + + =        2 3 3 11 0 2 4 x y y  =− −  ⇔   + + =      (hệ phương trình vô nghiệm). Vậy hệ phương trình có nghiệm: ( ) ( ) ( ) ; 2;0 , 0;2 x y = . Bài tập tương tự: 1) Giải hệ phương trình 2 2 19 84 xy x y x y xy  + + =  + =  . Đáp số: ( ) ( ) ( ) ( ) ( ) ; 6 42;6 42 , 6 42;6 42 , 3;4 , 4;3 x y = + − − + .
  • 50. Liên hệ tài liệu word toán SĐT (zalo): 3 Website: 2) Giải hệ phương trình  + =   + + =   2 2 6 5 x y xy xy x y . Đáp số: ( ) ( ) ( ) ; 1;2 , 2;1 x y = . 3) Giải hệ phương trình 2 2 2 3 y x xy  − =   = −   . Đáp số: ( ) ( ) ( ) ; 1; 3 , 1; 3 x y =− − . 4) Giải hệ phương trình 4 4 5 97 x y x y  + =  + =  . Đáp số: ( ) ( ) ( ) ; 2;3 , 3;2 x y = . 5) Giải hệ phương trình 2 2 1 3 x xy y x y xy  + + =   − − =   . Đáp số: ( ) ( ) ; 1; 1 x y= − . B. HỆ PHƯƠNG TRÌNH ĐỐI XỨNG LOẠI II Hệ phương trình đối xứng loại II theo ẩn x và y là hệ phương trình mà khi ta đổi vai trò của x cho y thì hai phương trình của hệ sẽ hoán đổi cho nhau. Dạng phương trình ( ) ( ) , 0 ; 0 f x y f y x  =   =   .  Phương pháp giải: Bước 1: Cộng hoặc trừ hai vế của hai phương trình để đưa hệ phương trình về phương trình tích và lập hệ phương trình: Đưa về dạng ( ) ( ) ( ) , , 0 , 0 f x y f y x f x y  − =   =   hoặc ( ) ( ) ( ) , , 0 , 0 f x y f y x f x y  + =   =   ( ) ( ) ( ) . , 0 , 0 x y x y f x y f x y  = ⇔ − = ⇔  =   . Bước 2: Giải hệ phương trình vừa lập được. Bước 3: Xét nghiệm của hệ phương trình là nghiệm của từng phương trình trong hệ ở bước 1. Ví dụ 1: Giải hệ phương trình: ( ) ( ) 2 2 1 3 1 1 3 2 x y y x  + =   + =   Giải: Lấy (1) trừ vế theo vế cho (2), ta được: ( ) ( ) 2 2 1 1 3 3 x y y x + − + = −
  • 51. Liên hệ tài liệu word toán SĐT (zalo): 4 Website: ( ) ( )( ) ( ) 2 2 3 3 0 x y y x x y x y x y ⇔ − = − ⇔ − + + − = ( )( ) 3 0 3 0 3 x y x y x y x y x y x y   = = ⇔ − + + = ⇔ ⇔   + + = = − −    . Với x y = thế vào (1), ta được: 2 1 3 y y + = 2 3 5 3 5 2 2 3 1 0 3 5 3 5 2 2 y x y y y x  + + = ⇒ =   ⇔ − + = ⇔  − − = ⇒ =   . Với 3 x y =− − thế vào (2), ta được ( ) 2 1 3 3 y y + = − − 2 2 3 31 3 10 0 0 2 4 y y y   ⇔ + + = ⇔ + + =     (vô nghiệm). Vậy hệ phương trình có nghiệm: ( ) 3 5 3 5 3 5 3 5 , , , , 2 2 2 2 x y     + + − − =             . Ví dụ 2: Giải hệ phương trình: ( ) ( ) 2 2 2 2 2 2 1 2 2 2 x y x y y x y x  − = +   − = +   Giải: Lấy (1) trừ vế theo vế cho (2), ta được: ( ) ( ) ( ) ( ) 2 2 2 2 2 2 2 2 x y y x x y y x − − − = + − + ( ) ( )( ) ( ) 2 2 3 3 3 1 x y x y x y x y x y x y x y  = ⇔ − = − ⇔ − + = − ⇔  + =   . Với x y = thế vào (1) ta được: 2 0 0 3x 3 3 x y x x y  = ⇒ = − = ⇔  = − ⇒ = −  . Với ( ) 1 3 3 1 3 y x y x − + = ⇔ = thế vào (2), ta được 2 2 1 3 1 3 2 2 3 3 y y y y   − − − = +     2 2 1 5 1 19 0 0 3 9 6 36 y y y   ⇔ − + = ⇔ − + =     (vô nghiệm). Vậy hệ phương trình có nghiệm: ( ) ( ) ( ) ; 0;0 , 3; 3 x y = − − . Bài tập tương tự: 1) Giải hệ phương trình 3 3 2 2 x x y y y x  = +   = +  
  • 52. Liên hệ tài liệu word toán SĐT (zalo): 5 Website: Đáp số: ( ) ( ) ( ) ( ) ; 0;0 , 3; 3 , 3; 3 x y = − − 2) Giải hệ phương trình 2 2 2 2 2 2 x y y xy x  + =   + =   . Đáp số: ( ) ( ) ; 1; 1 x y = − − 3) Giải hệ phương trình  + + =   + + =   2 2 1 1 x xy y x xy y . Đáp số: ( ) ( ) ( ) 1 1 ; 1; 1 , ; , ;2 2 2 x y t t   = − −     với t R+ ∈ . C. HỆ PHƯƠNG TRÌNH ĐẲNG CẤP BẬC HAI Dạng hệ phương trình ( ) ( ) 2 2 2 2 0 1 0 2 ax by cxy d a x b y c xy d  + + + =   ′ ′ ′ ′ + + + =   .  Phương pháp giải: Bước 1: Xét ( ) 0 0 x y = = , thế vào cả hai phương trình để tìm nghiệm (nếu có). Bước 2: Xét ( ) 0 0 x y ≠ ≠ . Đặt ( ) x ty y tx = = . Ta có hệ phương trình tương đương ( ) ( ) 2 2 2 2 2 2 2 2 3 4 at y by cty d a t y b y c ty d  + + = −   ′ ′ ′ ′ + + = −   . Bước 3: Chia (3) cho (4), ta được: 2 2 at b ct d d a t b c t + + = ′ ′ ′ ′ + + . Ta đưa về phương trình bậc nhất hai ẩn t rồi tiến hành giải. Ví dụ 1: Giải hệ phương trình 2 2 2 2 2 3 13 4 2 6 x xy y x xy y  − + =   + − = −   . Giải: Xét 0 x = , ta có hệ phương trình 2 2 3 13 2 6 y y  =   − = −   (vô nghiệm). Xét 0 x ≠ , đặt y tx = . Hệ phương trình trở thành: ( ) ( ) 2 2 2 2 2 2 2 2 2 2 2 2 2 3 13 (1) 2 3 13 4 2 6 1 4 2 6 (2) x t t x tx t x x tx t x x t t   − + = − + =   ⇔   + − = − + − = −     .
  • 53. Liên hệ tài liệu word toán SĐT (zalo): 6 Website: Chia (1) cho (2) theo vế, ta được: 2 2 2 3 13 6 1 4 2 t t t t − + = − + − 2 25 4 8 46 25 0 1 2 t t t t  =  ⇔ − − = ⇔   = −   . Với 25 4 t = thế vào (1), ta được: 2 2 2 25 25 1807 2 3. 13 13 4 4 16 x x       − + = ⇔ =         2 4 25 16 139 139 139 4 25 139 139 x y x x y  = ⇒ =   ⇔ = ⇔  = − ⇒ = −   . Với 1 2 t = − thế vào (1), ta được:. 2 2 2 1 1 13 2 3. 13 13 2 2 4 x x         − − + − = ⇔ =             2 2 1 4 2 1 x y x x y  = ⇒ = − ⇔ = ⇔  = − ⇒ =  . Vậy hệ phương trình có nghiệm: ( ) ( ) ( ) 4 25 4 25 ; ; , ; , 2; 1 , 2;1 139 139 139 139 x y     = − − − −         . Ví dụ 2: Giải hệ phương trình: 2 2 2 2 3 1 3 3 13 x xy y x xy y  − + = −   − + =   Giải: Xét 0 x = , ta được hệ phương trình 2 2 1 3 13 y y  = −   =   (vô nghiệm). Xét 0 x ≠ , đặt y tx = ta được hệ phương trình ( ) ( ) 2 2 2 2 2 2 2 2 3 1 1 3 3 13 2 x tx t x x tx t x  − + = −   − + =   . Chia (1) cho (2) theo vế, ta được 2 2 1 3 1 13 3 3 t t t t − + = − − + . 2 2 2 5 2 0 1 2 t t t t  =  ⇔ − + = ⇔  =   .
  • 54. Liên hệ tài liệu word toán SĐT (zalo): 7 Website: Với 2 t = thế vào (1), ta được: 2 1 2 1 1 2 x y x x y  = ⇒ = − =− ⇔  =− ⇒ =−  . Với 1 2 t = thế vào (1), ta được: 2 2 1 1 1 4 2 1 x y x x y  = ⇒ = − =− ⇔  = − ⇒ = −  . Vậy hệ phương trình có nghiệm: ( ) ( ) ( ) ( ) ( ) ; 1;2 , 1; 2 , 2;1 , 2; 1 x y = − − − − . Bài tập tương tự: 1) Giải hệ phương trình 2 2 2 2 3 2 2 7 6 3 8 x xy y x xy y  − + =   + − = −   . Đáp số: ( ) ( ) ; 1;1 x y = − , ( ) 1; 1 − , 5 31 ; 241 241       , 5 31 ; 241 241   − −     . 2) Giải hệ phương trình: 2 2 2 2 3 5 4 3 9 11 8 6 x xy y x xy y  − − = −   + − =   . 3) Giải hệ phương trình: 2 2 2 2 3 2 11 2 3 17 x xy y x xy y  + + =   + + =   . 4) Giải hệ phương trình: 2 2 2 2 3 8 4 0 5 7 6 0 x xy y x xy y  − + =   − − =   . 5) Giải hệ phương trình: 2 2 2 3 2 160 3 2 8 x xy x xy y  − =   − − =   . II. BÀI TẬP Bài 1: Giải hệ phương trình: 1 1 9 2 1 3 1 1 4 2 x y x y x xy y xy  + + + =       + + = +       Giải: Hệ phương trình tương đương với: 1 1 9 2 1 3 1 1 1 2 4 2 x y y x x x y y y x      + + + =                  + + = + + −             Đặt 1 1 u x y v y x  = +     = +  
  • 55. Liên hệ tài liệu word toán SĐT (zalo): 8 Website: Hệ phương trình trở thành: 9 9 2 2 9 3 9 1 3 2 4 2 2 4 2 v u u v u u u u uv   = − + =     ⇔       + = − + = −         Suy ra: 2 2 9 3 9 9 3 0 4 2 2 4 u u u u u + = − ⇔ − + = . 2 1 3 3 3 2 0 2 2 1 3 3 x u y u v y x   + =     =   ⇔ − = ⇒ ⇔         = + =     2 3 1 1 1 3 2 3 3 3 2 0 2 2 2 2 1 3 2 1 y xy y x y y x x y y y y xy x y x   + = = ⇒ =   ⇔ ⇒ = ⇒ = ⇒ + = ⇔ − + = ⇒    + = = ⇒ =    Hệ phương trình có nghiệm: ( ) ( ) 1 ; ;1 , 1;2 2 x y   =     . Nhận xét: Bài toán sử dụng phương pháp đặt hai ẩn phụ, đưa về hệ phương trình bậc hai cơ bản giải bằng phương pháp thế. Sau đó từ nghiệm ẩn phụ suy ngược lại nghiệm của hệ phương trình. Ý tưởng: Hình thức bài toán khá phức tạp vì sự xuất hiện của phân thức, quan sát ta thấy ở hai phương trình của hệ đều xuất hiện biểu thức 1 x y + . Ta sẽ nghĩ đến chuyện thế 1 9 1 2 x y y x   + = − +     xuống phương trình hai nhưng còn đại lượng 1 xy xy + chưa biết xử lý như thế nào. Có lẽ tác giả đã gợi mở theo con đường đặt ẩn phụ, nếu đặt 1 u x y = + ; 1 v y x = + thì bây giờ ta chỉ cần biểu diễn 1 xy xy + qua u, v thì hệ phương trình đã cho sẽ được giải quyết. Ta có 1 1 1 1 u x uy xy y v y vx xy x  = + ⇔ = +     = + ⇔ = +   . ( ) 2 1 1 2 uvxy xy uv xy xy ⇒ = + ⇔ = + + . Khi đó, hệ phương trình đã cho tương đương với: 9 2 1 3 2 4 2 u v u uv  + =     + = −   Hệ phương trình trên là hệ phương trình cơ bản, hoàn toàn giải quyết được bằng phương pháp thế.
  • 56. Liên hệ tài liệu word toán SĐT (zalo): 9 Website: Bài toán kết thúc. Bài tập tương tự: 1) Giải hệ phương trình ( ) 2 2 2 2 2 1 2 1 3 x y xy x y x  + =   + + =   . Đáp số: ( ) ( ) ( ) 7 5 7 5 ; 1;1 , 1; 1 , ; , ; 4 4 7 7 x y     = − − − −             . 2) Giải hệ phương trình ( ) ( ) ( )( ) 2 2 2 2 1 1 27 1 1 10 x y xy x y xy  + + =   + + =   . Đáp số: ( ) ( ) 1 1 ; ;2 3 , 2;2 3 , 2 3; 2 2 x y     = ± ± −         . Bài 2: Giải hệ phương trình: 2 2 2 2 1 2 4 x xy y x xy y  − + =   + + =   Giải: Giải hệ phương trình: 2 2 2 2 1 2 4 x xy y x xy y  − + =   + + =   . Thế 2 2 1 x xy y = − + xuống vế phải phương trình thứ hai của hệ phương trình, ta có: ( ) 2 2 2 2 2 4 x xy y x xy y + + = − + ( )( ) 2 2 3 5 2 0 3 2 0 . 3 2 x y x xy y x y x y x y  = ⇔ − + = ⇔ − − = ⇔  =  Với y x = , thế vào phương trình thứ nhất trong hệ, ta được 2 1 1 1 1 1 x y x x y  = ⇒ = = ⇔  =− ⇒ =−  . Với 3 3 2 2 x x y y = ⇔ = , thế vào phương trình thứ nhất trong hệ ta được: 2 2 3 7 7 7 1 4 2 3 7 7 x y x x y  = ⇒ =   = ⇔  = − ⇒ = −   . Vậy hệ phương trình đã cho có bốn nghiệm kể trên. Nhận xét: Bài toán sử dụng phương pháp thế và tách ghép phương trình đẳng cấp bậc hai tìm nhân tử chung. Nhắc lại kiến thức và phương pháp:
  • 57. Liên hệ tài liệu word toán SĐT (zalo): 10 Website: • Phương trình đẳng cấp bậc hai có dạng: 2 2 . . . 0 a x b xy c y + + = . • Nhóm nhân tử chung, đưa về dạng: ( )( ) 2 2 . . . 0 . . 0 x my a x b xy c y x m y x n y x ny  = + + =⇔ − − =⇔  =  . Hai trường hợp này, thế vào một trong hai phương trình còn lại của hệ, sẽ được phương trình có dạng 2 2 x t x t y u y u  =⇔ = ±   = ⇔ = ±  Chú ý: Nếu , 0 t u < thì phương trình sẽ vô nghiệm. Ý tưởng: Nhận thấy ở vế trái của mỗi phương trình đều có dạng của phương trình đẳng cấp bậc hai 2 2 . . . a x b xy c y + + , nếu thế một trong hai phương trình còn lại ta cũng sẽ thu được một phương trình bậc đẳng cấp bậc hai bằng 0. Từ đó tìm mối liên hệ giữa x, y. Thế ngược lại phương trình thứ nhất của hệ. Tìm nghiệm. • Ở vế phải phương trình thứ hai có 4 4.1 = mà từ phương trình một 2 2 1 x xy y = − + . Vậy nên phương trình hai trở thành: ( ) 2 2 2 2 2 4 x xy y x xy y + + = − + ( )( ) 2 2 3 5 2 0 3 2 0 x xy y x y x y ⇔ − + = ⇔ − − = . • Có thể dễ dàng giải phương trình theo cách đồng nhất hệ số ở bước làm ra nháp như sau: ( )( ) ( ) 2 2 2 2 3 5 2 3 3 x xy y x my x ny x mxy nxy mny − + = − − = − − + Đồng nhất hệ số hai phương trình, ta có 3 3 5 3 2 m n mn  + =  =  Đây là phương trình hai ẩn, đã biết cách giải theo Vi-ét, tìm được 2 1; 3 m n = = . Bài toán kết thúc Bài tập tương tự: 1) Giải hệ phương trình 2 2 2 2 7 3 5 5 x xy y x xy y  + + =   − − =   . Đáp số: ( ) ( ) ( ) 5 4 5 4 ; 2;1 , 2; 1 , ; , ; 3 3 3 3 x y     = − − − −         2) Giải hệ phương trình  + + =   − + =   3 2 3 2 2 3 3 3 3 x x y y x xy y . Đáp số: ( ) ( ) ( ) ( ) 3 ; 1;1 , 2; 1 , 3;0 x y = − .
  • 58. Liên hệ tài liệu word toán SĐT (zalo): 11 Website: Bài 3: Giải hệ phương trình 2 2 2 2 1 2 3 2 x xy y x xy y x y  − + =   + − − − = −   . Giải Cộng từng vế hai phương trình ta có: 2 2 3 1 x xy x y + − − = − ( ) ( ) 2 2 1 0 x x y x x y ⇔ + − − + + = ( )( ) 1 2 1 0 x x y ⇔ − + − = TH1: 2 1 0 0 x y y y = ⇒ − = ⇒ = hoặc 1 y = (thỏa mãn) TH2: 2 1 1 2 x y y x + = ⇒ = − suy ra ( ) ( ) 2 2 2 0 1 1 2 1 2 1 7 5 0 5 3 7 7 x y x x x x x x x y  = ⇒ =  − − + − = ⇔ − = ⇔  = ⇒ = −   . Đáp số: ( ) ( ) ( ) ( ) 5 5 ; 1;0 , 1;1 , 0;1 , ; 7 7 x y   = −     . Nhận xét: Bài toán sử dụng phương pháp hằng số biến thiên tìm ra được một phương trình biểu diễn mối liên hệ giữa hai biến và từ đó thế ngược lại một trong hai phương trình, tìm nghiệm của hệ. Ý tưởng: Đây là hệ phương trình bậc hai, trước hết ta sẽ đi tìm nhân tử ở từng phương trình trong hệ, nếu công việc này thất bại. Ta sẽ nghĩ đến việc kết hợp cả hai phương trình. Và điều tối ưu ta nghĩ tới sẽ là xét đenta theo ẩn x hoặc y từng phương trình (bạn đọc tự làm) khi đó không tìm được nhân tử x; y. Chính vì thế, còn hướng duy nhất đó là kết hợp hai phương trình của hệ, giả sử tồn tại k R ∈ thỏa mãn phương trình: ( ) − + − + + − − − + = 2 2 2 2 1 2 3 2 0 k x xy y x xy y x y ( ) ( ) ( ) ( ) 2 2 1 2 3 1 2 0 k x k y x k y y k i   ⇔ + + − − + − − + − =   . Và ta coi (i) là phương trình bậc hai ẩn x đồng thời khi xét x ∆ nó phải là một số chính phương. Ta có: ( ) ( ) ( ) 2 2 2 3 4 1 1 2 x k y k k y y k     ∆ = − − − + − − + −     ( ) ( ) 2 2 2 5 4 10 8 4 4 1 k k y k y k k = − − − + − + . Để x ∆ là số chính phương khi hệ số 2 y phải là một số chính phương, tức là ta đi giải phương trình: nghiệm nguyên 2 2 5 4 k k m − =. Không khó để ta thấy rằng 1 1 k m = ⇒ = thỏa mãn. Hay nói cách khác: ( ) 2 1 x y ∆ = − . Khi đó phương trình (i):
  • 59. Liên hệ tài liệu word toán SĐT (zalo): 12 Website: ( ) 2 3 1 1 2 3 1 0 3 1 2 2 y y x x y x y y y x y  − + − = =  ⇔ + − + − = ⇔  − − +  = = −   Việc còn lại là thế 1 x = hoặc 2 x y + = vào phương trình một để tìm nghiệm của hệ phương trình Bài toán kết thúc. Bài tập tương tự: 1) Giải hệ phương trình 3 3 2 2 2 3 9 4 x y y x y x y  − = +   + = −   . Đáp số: ( ) 1 13 5 13 ; ; 2 2 x y   ± − ± =       . 2) Giải hệ phương trình 2 2 2 2 2 3 0 3 1 0 x xy y x xy y y  + + + =   + + + =   . Đáp số: ( ) ( ) 1 5 ; 3 2 2;1 2 , 3 5; 2 x y   ± = − ± − ±        . Bài 4: Giải hệ phương trình: ( )( ) 2 2 3 2 2 2 x y x x y xy  + =   = + −   Giải Từ hệ phương trình đã cho ta có ( )( ) 3 2 2 3 3 2x x y x y xy x y x y = + + − = + ⇔ = . Do đó hệ đã cho tương đương với 2 2 1 2 x y x y x y  = ⇔ = = ±  + =  . Vậy hệ phương trình có nghiệm ( ) ( ) ( ) ; 1;1 , 1; 1 x y = − − . Nhận xét: Bài toán sử dụng phương pháp thế để đưa về phương trình đẳng cấp có mối liên hệ giữa các biến sau đó thế ngược lại một trong hai phương trình của hệ ban đầu để tìm nghiệm. Nhắc lại kiến thức và phương pháp: • Hằng đẳng thức dạng: ( )( ) 3 3 2 2 a b a b a ab b + = + − + và ( )( ) 3 3 2 2 a b a b a ab b − = − + + . • Phương trình dạng: ( )( ) 3 3 3 3 2 2 0 0 x y x y x y x xy y = ⇔ − = ⇔ − + + = . Vì 2 2 2 2 3 0 2 4 y y x xy y x   + + = + + >     .
  • 60. Liên hệ tài liệu word toán SĐT (zalo): 13 Website: Ý tưởng: Cả hai phương trình đều xuất hiện hằng số 2. Đồng thời ở phương trình thứ hai có dáng dấp là một phương trình đẳng cấp bậc ba vì vế trái chứa 3 2x còn vế phải chứa tích của một hàm số bậc nhất và một hàm số bậc hai vì vậy ta sẽ thực hiện phép thế 2 2 2 x y = + vào vế phải của phương trình hai, ta sẽ có: ( )( ) 3 2 2 2x x y x y xy = + + − (*). Với phương trình này (*) nếu bạn nào tinh ý sẽ phát hiện được hằng đẳng thức ( )( ) + = + − + 3 3 2 2 x y x y x xy y . Vì thế (*) ⇔ = + ⇔ = 3 3 3 2x x y x y . Không thì khai triển tích ra ta cũng sẽ có được điều đó. Việc còn lại chỉ là thế x y = ngược lại phương trình thứ nhất trong hệ và tìm nghiệm. Bài toán kết thúc. Bài tập tương tự: 1) Giải hệ phương trình ( )( ) 2 3 4 2 2 x y x x y x  + =   = + −   . Đáp số: ( ) ( ) 5 ; 1;1 , 1; 3 x y   = −     . 2) Giải hệ phương trình ( )( ) 2 2 3 3 4 2 2 x y x x y xy  + =   = + −   . Đáp số: ( ) ( ) ( ) ; 1;1 , 1; 1 x y = − − . Bài 5: Giải hệ phương trình 3 3 1 7 7 x y y x xy xy y x  + = + − +   + − =   Giải Cộng hai phương trình của hệ ta thu được ( ) ( ) 3 3 3 3 3 6 8 2 3 2 0 x y xy x y xy + + = ⇔ + + − − − = ( )( ) 2 2 2 4 2 2 0 x y x y xy y x ⇔ + − + + − + + = Ta luôn có: ( ) ( ) ( ) 2 2 2 2 2 2 x y xy y x + + − ≥ + − + − đẳng thức xảy ra khi và chỉ khi 2 x y = = − . Vậy nếu 2 2 4 2 2 0 x y xy y x + + − + + = ta suy ra 2 x y = = − (loại) vì không thỏa mãn phương trình 7 7 xy y x + − =. Vậy thu được hệ 2 2 7 7 x y y x xy y x  + = ⇒ = −  + − =  .
  • 61. Liên hệ tài liệu word toán SĐT (zalo): 14 Website: Suy ra ( ) 2 1 1 7 2 2 2 7 7 12 5 0 5 9 7 7 x y x x x x x x y  = ⇒ =  − + − = ⇔ − + = ⇔  = ⇒ =   . Nhận xét: Bài toán sử dụng phương pháp thế (hay cộng vế) để ra được phương trình có mối liên hệ giữa các biến. Sau đó thế ngược lại tìm nghiệm của hệ phương trình. Nhắc lại kiến thức và phương pháp: • Tổng các đại lượng không âm: ( ) ( ) ( ) 2 2 2 2 2 2 0 a b b c a c a b c ab bc ca − + − + − ≥ ⇔ + + ≥ + + . • Đẳng thức: ( )( ) 3 3 3 2 2 2 3 a b c abc a b c a b c ab bc ca + + − = + + + + − − − . Ý tưởng: Cả hai phương trình của hệ, đều xuất hiện nhân tử x y − vì thế ta sẽ nghĩ đến chuyện thế x y − từ phương trình một vào phương trình hai (hoặc ngược lại), do đó ta có được 3 3 6 8 0 x y xy + + − = (i). Đến đấy ta mong muốn sẽ biểu diễn mối quan hệ giữa x, y, quan sát phương trình (i), ta thấy rằng ( ) 3 2 8 − = − và ( ) 6 3 2 xy xy = − do đó nếu đặt 2 z = − thì (i) 3 3 3 3 0 x y z xyz ⇔ + + − = . Một biểu thức đối xứng rất đẹp, bằng cách nhóm nhân tử, ta có: ( ) ( ) 3 3 3 3 3 3 0 3 0 x y z xyz x y z xy x y z + + − = ⇔ + + − + + = . ( )( ) ( ) 2 2 2 2 3 0 x y z x xy y xz yz z xy x y z ⇔ + + + + − − + − + + =. ( )( ) 2 2 2 0 (*) x y z x y z xy yz xz ⇔ + + + + − − − = . Dễ thấy ( ) ( ) ( ) 2 2 2 2 2 2 1 2 x y z xy yz xz x y y z z x   + + − − − = − + − + −     do đó, phương trình (*)  + + = ⇔  = =  0 x y z x y z . Công việc còn lại là thay 2 z = − suy ra 2 2 x y x y  + =  = = −  . Nhưng 2 x y = = − loại vì không thỏa mãn phương trình hai trong hệ. Với 2 x y + = thay xuống phương trình hai, ta tìm được nghiệm của hệ phương trình là ( ) ( ) 5 9 ; 1;1 , ; 7 7 x y   =     . Bài toán kết thúc. Bài tập tương tự: 1) Giải hệ phương trình 3 3 2 2 1 3 2 3 0 x y xy x y x y  + + =   + − + − =   . Đáp số: ( ) ( ) 1 33 5 33 1 33 5 33 ; 1;1 , ; , ; 4 4 4 4 x y     − − + + − − =             .